Tại sao Giả thuyết 1 và 2 không thể suy ra từ bài toán 7‹ Bài toán 10 .15/ Hãy nêu ví dụ trò chơi kết thúc của hai người chơi trong đó chỉ có 1 chu trình và không có cân bằng thuần nhất [r]
(1)5 magazine
Chuẩn Euclid
Ngô Bảo Châu
Cân Nash
Vladimir Gurvich
Luật Benford ứng dụng thú vị
Trần Nam Dũng & Đặng Nguyễn Đức Tiến
Ứng dụng dãy số toán phương trình hàm
Đỗ Minh Khoa & Võ Quốc Bá Cẩn
(2)(3)LỜI NGỎ CHO EPSILON SỐ 5
Ban Biên tập Epsilon
Ý tưởng tạp chí Epsilon khởi nguồn vào khoảng cuối năm 2014, đến hành trình gần năm Nhìn lại suốt chặng đường đó, chúng tơi ln nhận sức sống Epsilon gắn liền với ủng hộ đóng góp độc giả tác giả
Để đáp lại thịnh tình đơng đảo độc giả, Epsilon số có nội dung hấp dẫn với nhiều viết thể loại chuyên mục khác Ngoài chuyên mục định kỳ lịch sử toán học, vấn đề cổ điển đại có viết thú vị khác
Phần mở đầu Epsilon số viết Chuẩn Euclid Ngơ Bảo Châu – tóm lược phần đầu giảng trường hè Lý thuyết số từ cổ điển đến đại
Tiếp theo đó:
Về xấp xỉ Diophantine: phần trước, có câu trả lời cho câu hỏi "Các số hữu tỉ xấp xỉ số vơ tỉ tốt đến nào?" số này, lần Lý Ngọc Tuệ giới thiệu với vấn đề thú vị khả xấp xỉ véc tơ trênRnbằng véc tơ hữu tỉQn
Về nhịp cầu kết nối toán cao cấp toán sơ cấp có bài: Từ Euclid đến Lobachevsky Nguyễn Ngọc Giang Cân Nash Vladirmir Gurvich Phần viết Chứng minh tiến William P Thurston qua lời dịch Nguyễn Dzuy Khánh đề cập đến câu hỏi thú vị quan trọng "Điều khích lệ người nghiên cứu toán học?"
Về trung gian lý thuyết ứng dụng viết luật Benford ứng dụng thú vị Trần Nam Dũng & Đặng Nguyễn Đức Tiến
Ngồi mảng tốn sơ cấp, phong phú chủ đề hình học với viết "Điều kiện ngoại tiếp tứ giác không lồi ứng dụng" Đỗ Thanh Sơn, viết cơng thức tính khoảng cách tâm đường tròn Euler tâm đường tròn Apollonius Trịnh Xuân Minh "Tổng quát hóa hình vơ địch Nga 2005" Trần Quang Hùng Phan Anh Quân Phần giải tích đại số có "Áp dụng dãy số vào giải phương trình bất phương trình hàm" Đỗ Minh Khoa Võ Quốc Bá Cẩn "Giải tích toán cực trị" Trần Nam Dũng
Phần số học tổ hợp có "Thặng dư bậc hai moduloM" Nguyễn Hồng Lữ "Phân hoạch tập số tự nhiên thành hai tập hợp có tổng nhau" Nguyễn Văn Lợi, Nguyễn Hải Đăng Nguyễn Thành Khang, “Tối ưu tổ hộp” Gil Kalai
(4)Đặc biệt số này, giới thiệu số đề thi (cùng lời giải bình luận) chọn đội tuyển số trường số tỉnh cho VMO 2016
Cuối phần kết Epsilon số Ma trận ngẫu nhiên Vũ Hà Văn – nơi thông báo hàng loạt giả thuyết chinh phục Vũ đồng nghiệp anh
Hy vọng rằng, Epsilon nhận ủng hộ độc giả, đóng góp bạn ln động lực để người thực tiếp tục đường dài phía trước
(5)MỤC LỤC
Ban Biên tập Epsilon
Lời ngỏ cho Epsilon số
Ngô Bảo Châu
Chuẩn Euclid
Lý Ngọc Tuệ
Xấp xỉ Diophantine trênRn- Phần 2: Quy tắc Dirichlet hình học số 15
Vladimir Gurvich
Cân Nash 25
Nguyễn Ngọc Giang
Mở rộng toán hình học Euclid thành tốn hình học cầu hình học Lobachevsky - Một phương phức sáng tạo toán 33
William P Thurston
Về chứng minh tiến toán học (tiếp theo) 53
Trần Nam Dũng, Đặng Nguyễn Đức Tiến
Luật Benford ứng dụng thú vị 61
Đỗ Thanh Sơn
Điều kiện ngoại tiếp tứ giác không lồi ứng dụng 69
Trịnh Xuân Minh
Khoảng cách tâm đường tròn Euler tâm đường tròn Apollonius 75
Trần Quang Hùng, Phan Anh Quân
Tổng qt tốn thi vơ địch Nga năm 2005 81
Đỗ Minh Khoa, Võ Quốc Bá Cẩn
(6)Trần Nam Dũng
Giải tích tốn cực trị 109
Nguyễn Hồng Lữ
Thặng dư bậc hai moduloM 131
Nguyễn Văn Lợi, Nguyễn Hải Đăng, Nguyễn Thành Khang
Về phân hoạch tập số tự nhiên thành hai tập hợp có tổng phần tử 151
Gil Kalai
Tối ưu tổ hợp I: Các toán tối ưu hệ tập hợp 167
Ban Biêp tập Epsilon
Bài toán hay - Lời giải đẹp 173
Trần Nam Dũng
Đơi điều hình học phi Euclid 177
Ban Biên tập Epsilon
Giới thiệu số đề thi chọn đội tuyển mơn Tốn năm 2015 - 2016 181
Trần Nam Dũng
Các vấn đề cổ điển đại 209
Vũ Hà Văn
(7)CHUẨN EUCLID
Ngô Bảo Châu (Viện nghiên cứu cao cấp toán - VIASM)
Bài viết trích từ giảng "Lý thuyết số từ cổ điển đến đại" Viện nghiên cứu cao cấp toán, hè 2015 Nội dung viết tập trung vào khái niệm chuẩn Euclid vành số nguyên, vành số nguyên Gauss hệ số học
1 Nguyên lý trật tự tập số tự nhiên
Đối tượng nghiên cứu lý thuyết số tập số tự nhiênND f0; 1; 2; : : :g Vì số tự nhiên dường quen thuộc, có ý thức thân số tự nhiên cần định nghĩa, xây dựng, cách chặt chẽ Xây dựng số tự nhiên cách chặt chẽ vấn đề trung tâm hóc búa sở tốn học Ở ta không đề cập đến vấn đề cách có hệ thống mà điểm lại số tính chất tập số tự nhiên mà ta công nhận tiên đề
Tập số tự nhiênNchứa hai phần tử0và1 Tập trang bị phép cộng.x; y/7!xCy Quan hệ thứ tựx < y, cho bởix < ykhi tn tiz Nviz Ô0sao choxCz Dy,
l mt quan hệ thứ tự tuyến tính theo nghĩa với mọix ¤ y, hoặcx < y hoặcy < x TậpN,
với quan hệ thứ tự tuyến tính, thoả mãn nguyên lý trật tự (well-ordering principle): tập không rỗngS Nđều chứa phần tử cực tiểu i.e tồn tạia S x với
x 2S Nguyên lý trật tự thực chất phiên nguyên lý quy nạp quen thuộc
2 Ước chung lớn nhất
Phép chia có dư Euclid phép toán số học Sự tồn phép toán dựa vào khẳng định: choa; bl hai s nguyờn vibÔ0, ú tn ti nhấtq; r 2Zvới
aDbqCrI (2.1)
sao chor thoả mãn0r <jbj
Để chứng minh khẳng định này, ta xét tậpS tất số tự nhiênx Nsao chox a modb
(ta theo quy ước0là số tự nhiên) TậpS chứaacho nên tập khơng rỗng Theo nguyên lý trật tự,S chứa phần tử cực tiểu mà ta ký hiệu làr Ta chứng minhr <jbjbằng phản chứng Thật vậy, nếur jbjthìr jbjsẽ phần tử củaS, nhỏ hẳnr mâu thuẫn với giả thiếtr phần tử cực tiểu củaS Vìr a mod b, tồn nhấtq Zsao cho
phương tình (2.1) thoả mãn
Với cặp số nguyêna; b 2Z, ước chung lớn gcd.a; b/là số nguyên dươngd lớn
(8)Thực nhiều lần phép chia có dư Euclid phương pháp hiệu để tính ước chung lớn NếuaDbqCr phương trình (2.1), ta dễ dàng kiểm tra gcd.a; b/D gcd.b; r/ Thay tính ước chúng lớn củaa D a0 vàb D b0, ta cần tính ước chung lớn củab D a1 vàr D b1với0 < r <jbj Tiếp tục vậy, ta có cặp số nguyên a0; b0/,.a1; b1/, cho
gcd.a0; b0/Dgcd.a1; b1/D
vớijb0j > b1 > b2 > > Dãy số bắt buộc phải dừng thời điểm đó, giả sử dừng ở.an; bn/ Ta khơng thực phép chia Euclid số chia khơng, có nghĩa làbnD0 Hiển nhiên nếubnD0thì ta có gcd.an; bn/Dan,
gcd.a; b/D Dgcd.an; bn/Dan:
Giải thuật trình bày gọi thuật tốn Euclid Nhìn từ góc độ thực hành, thuật tốn Euclid giải thuật hiệu để tính ước chung lớn hai số ngun lớn Nhìn từ góc độ lý thuyết, thuật toán Euclid kéo theo định lý sau, thường gọi định lý Bezout:
Định lý 2.1 Với số nguyêna; b2 Z, ước chung lớn nhấtd Dgcd.a; b/biểu diễn dưới
dạngd DxaCybvớix; y 2Z.
Thật vậy, theo quy nạp, tất sốa0; b0; a1; b1; : : : ; an; bn D 0xuất thuật tốn Euclid trình bày biểu diễn dạngxaCybvà đăc biệtd Dancó dạng Có chứng minh khác sử dụng nguyên lý trật tự thay cho thuật toán Euclid Xét tậpS số nguyên dươngncó dạngn DxaCybvớix; y 2Z Tập có phần tử cực tiểu ký hiệu làe2 S Ta cần chứng minh rằngd De
Vìdjavàdjbcho nêndjnvới mọin2 S Vì vậydje Để chứng minhd De, ta cần chứng minh thânecũng ước chung củaavàb Giả sử thế, chẳng hạn nhưekhơng ước củaa Khi thực phép chia có dư Euclid củaachia choe ta có aDqeCr với0 < r < e Hiển nhiênr Svì có dạngxaCybcho nên điểu mâu thuẫn với tính cực tiểu củae Vì vậyephải ước chung củaavàb
Định lý 2.2 Nếudjabvàgcd.d; a/D1thìdjb.
Nếu gcd.d; a/D1thì tồn tạix; y 2Zsao choxd CyaD1 Khi đóbD.xd Cya/bhiển
nhiên bội củad
Định lý 2.3 (Định lý số học) Mọi số tự nhiênn > 1có thể phân tích cách duy nhất thành tích số nguyên tố.
(9)Tạp chí Epsilon, Số 05, 10/2015
3 Vành có chuẩn Euclid
Vì định lý số học thực công cụ để giải tốn số học, ta muốn tìm cách mở rộng số vành giao hoán khác Để mở rộng lập luận trình bày phần trước vànhR, ta cần trang bị choR chuẩn Euclid Giả sửRlà miền nguyên (integral domain),K trường thương (fraction field) củaR, chuẩn Euclid củaRlà đồng cấu nhóm
j j WK !RC (3.1)
từ nhóm phần tử khả nghịch trongK vào nhóm nhân số dương thực cho với mọia2Rta cójaj N,
với mọix 2K, tồn tạia2Rsao chojx aj<
Với giả thiết này, phép chia có dư Euclid vànhRln tồn tại, dầu khơng nhất: vi mia; b 2R viB Ô0, tn tiq; r Rvớijrj<jbjsao cho phương trình (2.1) thoả mãn Thật vậy, ta chọnq Rsao chojab qj< 1, đór Da bq thoả mãn jrj<jbj
Định lý 3.1 Trong vành EuclidR, ideal ideal chính.
Ta cần chứng minh idealI Rđều chứa phần tử sinha I Theo nguyên lý trật tự tồn tạia2I cho với mọix 2I ta cójaj jxj Để chứng minh rằngalà phần tử sinh, ta chứng minh phần tửx I bội củaa Nếu không thế, tồn q; r Rsao chox DqaCr vớijrj<jajvà điều mâu thuẫn với tính cực tiểu củajaj Trong miền nguyênRmà ideal ideal chính, ta định nghĩa gcd.a; b/của hai phần tửa; b R Xét idealI D fxaCybjx; y Zgsinh bởia; b R Ideal
chứa phần tử sinhd ta đặtd Dgcd.a; b/ Vì hai phần tử sinh củad; d02 I sai khác đơn vịc R, có nghĩa làd D cd0, gcd.a; b/ là phần tử củaR
được xác định với sai khác phần tửc R
Tương tự trường hợp vành số nguyên, chứng minh phần tửn2 R phân tích thành tích phần tử nguyên tố; phân tích với sai khác phần tử khả nghịch củaR(một phần tửa 2R coi nguyên tố phân tích aDbc athành tích hai phần tửb; c2R, hoặcb hoặccphải phần tử khả nghịch)
NgồiZ, ví dụ vành Euclid vành đa thứckŒt một biếnt trườngk Vành số nguyên Gauss ví dụ thú vị khác:
R D fxCiyjx; y 2Zg: (3.2)
vớii thoả mãn phương trìnhi2 D Trường thương củaR
K D fxCiyjx; y 2Qg: (3.3)
Ta chọn chuẩn cho bởijxCiyj Dx2Cy2 Dễ thấy với a2 R ta cójaj Nvà với
x 2K, tổng quát hơn, với mọix2 C, tồn tạia2K, chojx aj< Thật
(10)cách tớixkhơng q1=p2 Vì ta chứng minh khẳng định mạnh hơn: với mọix 2C,
luôn tồn tạia2K, chojx aj 1=2
Như vành số nguyên GaussRlà vành Euclid, ideal ideal Các phần tử R phân tích cách thành tích phần tử nguyên tố với sai khác RD f˙1;˙ig
Lập luận tương tự, ta chứng minh vành
R2 D fxCyp 2jx; y2 Zg (3.4)
và
R3 D fxCy1C p
3
2 jx; y 2Zg (3.5)
cũng có chuẩn Euclid Thật vậy, xem phần tửR2(hoặcR3) mắt lưới mặt phẳng phức, lưới đủ dầy đề cho số phứcz 2C, tồn mắt
lướia2R2(hoặca2R3) chojz aj< Lập luận khơng cịn với
R5 D fxCyp 5jx; y2 Zg (3.6)
vì lướiR5quá thưa mặt phẳng phức: ta đảm bảo với mọiz C, tồn
a2R5sao chojz aj< Không chuẩn thông thườngjxCyp 5j Dx2C5y2của R5không phải chuẩn Euclid, màR5khơng thể có chuẩn Euclid tính chất phân tích thừa số nguyên tố khơng thoả mãn trongR5 Thật ta có đẳng thức
6D2:3D.1Cp 5/.1 p 5/ (3.7)
trong đó2; 3; 1Cp 5; p 5là phần tử nguyên tố củaR5 Như vậy6có hai cách phân tích khác thành tích phần tử ngun tố
Nói chung vành giao hốn có chuẩn Euclid, có, chúng đem đến cho ta số kết số học đơn giản thú vị
4 Tổng hai bình phương
Diophantus sách Arithmetica nhận xét số nguyên có dạng 4nC3vớin Z
không thể viết thành tổng hai bình phương Nói cách khác, với mọin Z, phương
trình
x2Cy2 D4nC3 (4.1)
khơng có nghiệmx; y 2Z
Phương trình thực khơng có nghiệm vànhZ=4Zcác lớp đồng dư modulo Thật
vậy nếux số chẵn, ta cóx mod 4, cịn số lẻ, ta cóx1 mod Vì trường hợpx2Cy2chỉ đồng dư với0; 1hoặc2modulo4 Như vậy, phương trình (4.1) khơng có nghiệmx; y2 Z=4Zvà khơng thể có nghiệm trongZ
(11)Tạp chí Epsilon, Số 05, 10/2015
Định lý 4.1 Một số tự nhiên n viết thành tổng hai bình phươngn D x2Cy2 với
x; y 2Zkhi phân tíchnthành tích thừa số nguyên tố
nD2epe1 : : : p
er
r (4.2)
vớip1; : : : ; pr là số nguyên tố lẻ đôi khác nhau, thừa số nguyên tốpi đồng dư với 3 modulo 4, có số mũei chẵn.
Trước hết ta nhận xét hai số nguyên biểu diễn dạng tổng hai bình phương tích chúng Nếun1 Dx12Cy12vàn2 Dx22Cy22thì ta có
n1n2 D.x1x2 y1y2/2C.x1y2Cx2y1/2: (4.3) Trong trường hợp x1; x2; y1; y2 số nguyên, z1 D x1 C iy1 z2 D x2 Ciy2 số nguyên Gauss tương ứng, ta có n1 D jz1j, n2 D jz2j n1n2 D jz1z2j với z1z2 D.x1x2 y1y2/Ci.x1y2Cx2y1/ Nhờ vào nhận xét này, định lý Fermat tổng hai bình phương quy hai khẳng định liên quan đến số nguyên tố lẻ
Định lý 4.2 Nếup là số nguyên tố lẻ vớip mod 4vàx; y2 Zsao chox2Cy20
mod pthì cảxvày đều bội củap.
Định lý 4.3 Nếu p là số nguyên tố lẻ với p mod thì tồn tạix; y Z sao cho
x2Cy2 Dp.
Sử dụng Định lý 4.2, ta chứng minh nếunlà tổng hai bình phương mà phân tích thành tích thừa số ngun tố cơng thức (4.2), thừa số nguyên tốpi đồng dư với modulo số thừa số nguyên tốp1; : : : ; pr có số mũei chẵn Ngược lại thừa số nguyên tốpi đồng dư với modulo có số mũei chẵn ta sử dụng Định lý 4.3 để chứng minh rằngncó thể biểu diễn thành tổng hai bình phương
Định lý 4.2 toán đồng dư Ta thực chất làm việc trườngFp lớp đồng dư moduop Giả sửy khơng bội củap, lớp đồng dư củaylà phần tử khác không, khả nghịch, củaFp Khi phương trìnhx2Cy2 modp kéo theo tồn củaz mod psao cho
z2 mod p (4.4)
Theo Định lý Fermat nhỏ, ta có đồng dưzp 1 mod p Đồng dư (4.4) kéo theo
1/p21 mod p: (4.5)
Vì đồng dư khơng với số ngun tốp mod 4, trường hợp quan hệ đồng dưx2Cy2 0 mod psẽ kéo theopjxvàpjy.
Bước chứng minh Định lý 4.3 nhật xét trường hợpp mod 4phương trình đồng dư (4.4) có nghiệm Có hai phương cách để chứng minh khẳng định Phương cách thứ sử dụng định lý Wilson:
.p 1/Š mod p: (4.6)
(12)Để suy từ việc tồn nghiệm phương trình đồng dư (4.4) việc phương trìnhp Dx2Cy2 có nghiệm ngun, ta cần dùng đến chuẩn Euclid vành số nguyên GaussR DZŒ
p 1 Choz số nguyên choz2C10 modp Nếu cần thiết thayzbằngz Cpta giả thiết
z2C160 mod p2: (4.7)
Xét ước chung lớn củazCi vàp trongR VìR vành Euclid, ước chung lớn tồn với sai khác phần tử củaRD f˙1;˙ig, ta có
gcd.zCi; p/DxCiy (4.8)
Khi ta có
x2Cy2 D jxCiyj Dgcd.jz Cij; p/Dgcd.z2C1; p2/Dp: (4.9) Như phương trình x2Cy2 D p có nghiệm ngun với số nguyên tốp vớip mod
5 Về phương trình có dạng n D x2 C dy2
Phương trìnhn D x2C2y2 có thể giải phương pháp hồn tồn tương tự với phương trìnhn D x2Cy2 Các số phức có dạngxCyp 2vớix; y Ztạo nên miền
nguyên ký hiệu làR D ZŒ
p
2 Trường thương củaR trườngK số phức có dạng xCyp 2vớix; y 2Q Ta có ánh xạ chuẩnK !Qcho
z DxCyp 27! jxCyp 2j Dx2C2y2: (5.1) Vì chuẩn đồng cấu nhóm: với mọiz1; z2 2Kta cójz1z2j D jz1jjz2jcho nên, bản,
giải phương trìnhnDx2C2y2quy trường hợpnlà số nguyên tố
Định lý 5.1
1 Nếu p là số nguyên tố đồng dư với modulo 8, phương trình đồng dư
xC2y20 mod pkéo theopjxvàpjy.
2 Nếup là số nguyên tố đồng dư với modulo 8, phương trìnhx2C2y2Dp
có nghiệm nguyên.
3 Số tự nhiênnvới phân tích thành tích thừa số ngun tố dạng(4.2)có thể biểu diễn ở dạngnDx2Cpy2khi thừa số nguyên tốpi, đồng dư với modulo 8, có số mũei là số chẵn.
Khẳng định thứ ba dễ dàng suy từ hai khẳng định đầu
(13)Tạp chí Epsilon, Số 05, 10/2015
Khẳng định thứ hai chứng minh hoàn toàn tương tự với toánp Dx2Cy2, dựa vào tồn chuẩn Euclid trênQŒp 2và tồn ước chung lớn vành
Vì chuẩn thơng thường trênQŒp 3vãn chuẩn Euclid tốnnDx2C3y2 giải phương pháp hoàn toàn tương tự Tuy số tự nhiênd tổng quát, đặc biệt vớid D5, chuẩn thông thường trênQŒ
p
d khơng cịn chuẩn Euclid Tệ nữa, vành phần tử nguyên củaQŒp d không vành chính, có ideal khơng thể sinh phần tử Vì lập luận khẳng định thứ hai sử dụng khái niệm ước chung lớn khơng cịn trường hợp
Để khắc phục khó khăn này, trước hết cần nhận thấy ta định nghĩa "ước chung lớn nhất" ideal thay số: ước chung lớn củaavàbchỉ đơn giản ideal sinh bởiavàb Nhận xét xem điểm khởi thuỷ lý thuyết miền Dedekind Trong miền Dedekind, mà điển hình miền phần tử nguyên mở rộng hữu hạn K củaQ, định lý phân tích thành tích thừa số ngun tố khơng cịn Tuy thế, định lý tương tự với số thay ideal, số nguyên tố ideal nguyên tố,
(14)(15)XẤP XỈ DIOPHANTINE TRÊN Rn - PHẦN 2: QUY TẮC DIRICHLET VÀ HÌNH HỌC CỦA
CÁC SỐ
Lý Ngọc Tuệ - Đại học Brandeis, Massachusetts, Mỹ
1 Định lý Dirichlet
Trong phần trước [11], với cơng cụ liên phân số, có câu trả lời cho câu hỏi: "Các số hữu tỉ xấp xỉ số vơ tỉ tốt đến nào?" qua định lý sau Euler: Định lý 1.1 (Euler 1748 [4]) Với số vô tỉx RXQ, tồn vô số số hữu tỉ p
q Qvới q > 0sao cho:
ˇ ˇ ˇ ˇ x p q ˇ ˇ ˇ ˇ <
q2: (1.1)
Tuy nhiên, tận chưa có cách xây dựng liên phân số không gian nhiều chiềuRncó đầy đủ tính chất để trả lời câu hỏi khả xấp xỉ véc tơ trênRnbằng véc tơ hữu tỉQn Phải đến gần 100 năm sau, Định lý 1.1 mở rộng lênRnbởi nhà toán học Peter Gustav Lejeune Dirichlet Kết xem xuất phát điểm cho lý thuyết xấp xỉ Diophantine phát triển Vì nên Định lý 1.1 thường gọi Định lý Dirichlet (trênR)
Trên không gian véc tơRn, giá trị tuyệt đối trênRtrong bất đẳng thức (1.1) thay sup norm:
xE
WDmaxfjx1j; :::;jxnjg vớixE D.x1; :::; xn/2 Rn: Lưu ý sup norm tương đương với Euclidean norm:
xE 2 WD p E x Ex D
q
x12Cx22C:::Cxn2
vẫn thường dùng để định nghĩa khoảng cách trênRnnhư sau: xE 2 xE p nxE
2: Định lý Dirichlet choRncó thể phát biểu sau:
Định lý 1.2 (Dirichlet 1842 [3]) Với véc tơ xE Rn XQn, tồn vô số véc tơ hữu tỉ
E p q D p1 q ; p2
q ; :::; pn
q
2 QnvipE2Znvq 2Z,q Ô0, cho:
E
x pE q < jqj1C1n
(16)Dirichlet chứng minh Định lý 1.2 thông qua Định lý sau:
Định lý 1.3 (Dirichlet 1842 [3]) Với mọiQ1và với mọixE Rn, tồn tạipE2Znvàq Z,
0 <jqj Qnsao cho:
qxE pE <
1
Q: (1.3)
Chứng minh Định lý 1.2 dựa vào Định lý 1.3. Với mỗiQ1cố định, áp dụng Định lý 1.3, ta tìm đượcpE2Znvàq 2Z,0 <jqj Qnsao cho:
E
x pE q D jqj
qxE pE <
1 Qjqj
1 jqj1Cn1
:
VìxE …Qn,xE pE
q Ô0, nờn viQ
0 > 0sao cho
1 Q0 <
E
x pE q ; E
p0vàq0tìm theo Định lý 1.3 tương ứng vớiQ0thỏa mãn điều kiện:
E
x pE
0 q0 < Q0jq0j
1 Q0 <
E
x pE q :
Điều dẫn đến:
E p0 q0 Ô
E p q:
Vỡ vy, khiQ! 1, ta có vơ số pE
q khác thỏa mãn (1.2)
Lưu ý 1.4 Định lý 1.3 gọi làĐịnh lý Dirichlet mạnhvà Định lý 1.2 gọi
Định lý Dirichlet yếu
Để chứng minh Định lý 1.3, Dirichlet sử dụng quy tắc nhốt thỏ vào chuồng (Dirichlet gọi
Nguyên tắc ngăn kéo - Schubfachprinzip), hay gọi nguyên lý Dirichlet sau:
Nguyên lý Dirichlet. Nếu cókcon thỏ bị nhốt tronglcái chuồng, vàk > l, có chuồng có thỏ
Lưu ý 1.5 Nguyên tắc biết đến nhà toán học trước Dirichlet (ss [8]), báo Dirichlet lần nguyên tắc áp dụng vào chứng minh kết quan trọng toán, nên gắn với tên ơng
Để minh họa ý tưởng chính, chứng minh Định lý 1.3 cho trường hợpnD1như sau:
Chứng minh Định lý 1.3 vớinD1. Với số thực x R, sử dụng ký hiệu phần
nguyên phần thập phân củaxnhư sau:
(17)Tạp chí Epsilon, Số 05, 10/2015
Khơng tính tổng qt, ta giả sử nhưQlà số nguyên dương (thayQbởibQcnếu cần), chia đoạnŒ0; 1/ra thànhQđoạn:
0; Q ; 1 Q; Q ; :::; Q 1 Q ;
;
mỗi đoạn có độ dài Q
XétQC1số thực0;fxg;f2xg; :::;fQxg VìQC1 > Q, theo Nguyên lý Dirichlet, tồn đoạn
a Q;
aC1 Q
,0a < Qv0q1; q2 Q,q1 Ôq2sao cho:
fq1xg;fq2xg
a Q;
aC1 Q
:
Vậy đặtp1 D bq1xc,p2D bq2xc, ta có được:
j.q1x p1/ q2x p2/j D jfq1xg fq2xgj < Q: Và (1.3) thỏa mãn vớiqDq1 q2 vàp Dp1 p2
Chứng minh dễ dàng mở rộng chon1bất kỳ sau:
Chứng minh Định lý 1.3 vớin1. Tương tự trên, ta giả sử rằngQ > 0là số nguyên dương Chia hình hộp vngŒ0; 1/nra thànhQnhình hộp vng nhỏ có độ dài cạnh
Q: a1
Q;
a1C1 Q
::: an
Q;
anC1 Q
với0a1; :::; an < Q: (1.4) Và xétQnC1véc tơ dạng:
0; fx1g; :::;fxng/; f2x1g; :::;f2xng/; :::; fQnx1g; :::;fQnxng/:
Theo Nguyên lý Dirichlet, ta tìm véc tơ nằm trong hộp vuông nhỏ (1.4) Và lập luận tương tự trên, ta tìm đượcpE2 Znvàq2 Zvới0 <jqj Qn
sao cho:
qxEC Ep <
1 Q:
Bài tập 1.6 Gọi Mm;n.R/là tập ma trậnmdòngncột với hệ số thực Định lý 1.2 mở rộng raMm;n.R/thành dạng mệnh đề sau: Nếu ma trậnA2 Mm;n.R/thỏa mãnAqE… Zmvới mọiqE2 ZnX f0g, tồn vụ s.p;E q/E 2ZmZnviqEÔ0v
AqE pE < qE :
(18)2 Hình học số Minkowski
Cũng trênR, tính tối ưu hàmjqj 1Cn1/trong Định lý 1.2 chứng minh bởi tồn véc tơxE xấp xỉ kémđược định nghĩa tính chất sau: tồn tạic > 0sao cho với véc tơ hữu tỉ pE
q 2Q n
,
E
x pE q
> c jqj1C1n
: (2.1)
Tuy nhiên không giống trường hợpR, khin > 1, khơng có cơng cụ liên phân số để mơ tả qua chứng minh tồn véc tơ xấp xỉ Tập véc tơ xấp xỉ trênRnlà đối tượng nghiên cứu quan trọng lý thuyết xấp xỉ Diophantine Chúng tơi có viết riêng tập số báo sau
Cũng khơng có cơng cụ liên phân số hồn thiện không gian nhiều chiều, phải sử dụng công cụ khác để cải thiện số1trong Định lý 1.2 Công cụ mà giới thiệu phần cịn lại Hình học số (Geometry of Numbers) Minkowski Hình học số (Geometry of Numbers) phát triển vào cuối kỷ 19, đầu kỷ 20 nhà toán học Hermann Minkowski [7] nhằm đưa đại số tuyến tính hình học vào giải số vấn đề lý thuyết số đại số Hình học số Minkowski nhanh chóng tìm ứng dụng xấp xỉ Diophantine, trở thành công cụ vô quan trọng
Một số tài liệu tham khảo cho Hình học số: Cassels [2], Siegel [10], Gruber & Lekkerkerker [5]
2.1 Vật lồi (Convex Body)
Một đối tượng nghiên cứu Hình học số tậplồitrongRnđược định nghĩa sau: Tập hợpE Rnđược gọi tậplồinếu với điểm bất kỳx;E yE 2E bất
kỳ, đoạn thẳng nốixEvàyEcũng nằm trongE: E
x;yE2E )txEC.1 t /yE 2E với mọi0t 1: E gọi làđối xứng tâmnếu như:
E
x 2E ) xE 2E: Bài tập 2.1 Phân loại tất tập lồi trênR
Ví dụ 2.2 (i) Tập˚
.x; y/ 2R2Wx2Cy2 tập lồi trênR2
(ii) Tập˚.x; y/2 R2Wx2Cy2D1 tập lồi trênR2
(iii) Tập (
E
x 2RnW
n X
iD1
jxij )
là tập lồi trênRn
(iv) Tập (
E
x 2RnW
n Y
iD1
jxij< )
(19)Tạp chí Epsilon, Số 05, 10/2015 Bài tập 2.3 Chứng minh ví dụ 2.2
Với tậpE Rn, ký hiệuE hàm đặc trưng củaE:
E.x/E WD (
1 ;xE 2E ;xE …E
và vol.E/là thể tích trênRncủaE (độ đo Lebesgue củaE): vol.E/D
Z
Rn
E.x/d.E x/:E
Định lý sau Minkowski, kết Hình học số, cho ta biết điều kiện đủ để tập lồi có chứa điểm có tọa độ nguyên:
Định lý 2.4 (Định lý hình lồi Minkowski ) GọiE Rnlà tập lồi, đối xứng tâm bị
chặn trênRn Nếu như: (i) vol.E/ > 2n,
(ii) vol.E/D2nvàE compact,
thìE có chứa điểm tọa độ nguyên khác 0: E \ZnX f0g Ô ;:
chng minh nh lý 2.4, ta cần đến Quy tắc Blichfeldt Hình học số (Định lý 2.6) Bổ đề sau:
Bổ đề 2.5 Giả sử nhưf x/E hàm khả tích khơng âm trênRnvới: Z
Rn
f x/d.E x/ <E 1:
Tồn tạiyE2Rnsao cho:
X
E
p2Zn
f yEC Ep/ Z
Rn
f x/d.E x/:E
Chứng minh Nếu chuỗi vế bên trái khơng bị chặn theoyEthì kết luận Bổ đề hiển nhiên Giả sử chuỗi vế bên trái bị chặn theoy, theo Định lý hội tụ mạnh củaE Lebesgue, ta có được:
Z
Rn
f x/d.E x/E D X
E
p2Zn Z
Œ0;1/n
f xEC Ep/d.x/E
D Z
Œ0;1/n X
E
p2Zn
f xEC Ep/d.x/E
vol.Œ0; 1/n/ sup
E
x2Œ0;1/n X
E
p2Zn
f xEC Ep/
D sup
E
x2Œ0;1/n X
E
p2Zn
(20)Nếu như:
Z
Rn
f x/d.E x/ <E sup
E
x2Œ0;1/n X
E
p2Zn
f xEC Ep/
thì ta tìm đượcyE 2Œ0; 1/nsao cho: Z
Rn
f Ex/d.x/E X
E
p2Zn
f yEC Ep/ < sup
E
x2Œ0;1/n X
E
p2Zn
f ExC Ep/:
Còn như:
Z
Rn
f x/d.E x/E D sup
E
x2Œ0;1/n X
E
p2Zn
f xEC Ep/
thì vol @ < : E
y Œ0; 1/n W Z
Rn
f Ex/d.x/E D X
E
p2Zn
f yEC Ep/ =
;
AD1;
nghĩa hầu hếtyE 2Œ0; 1/nthỏa mãn Bổ đề
Định lý 2.6 (Blichfeldt 1914 [1]) Nếu nhưE tập đo trênRnvới vol.E/ > 1thì tồn véc tơ khác nhaux1;E x2E 2S chox2E x1E 2Zn
Chứng minh Áp dụng Bổ đề 2.5 vớif DE, ta tìm đượcyE2Rnsao cho:
X
E
p2Zn
E.yEC Ep/ Z
Rn
E.x/d.E x/E Dvol.E/ > 1:
Vì vậy, tồn p1;E p2E Zn khác cho yE C Ep1;yE C Ep2 S Đặt x1E D Ey C Ep1,
E
x2 D EyC Ep2, ta có véc tơ thỏa mãn Định lý
Chứng minh Định lý Vật lồi Minkowski 2.4. Đầu tiên ta chứng minh cho trường hợp vol.E/ > 2n ĐặtS D˚xE W2xE 2E , thể tích củaS là:
vol.S /D
2nvol.E/ > 1:
Vì theo Định lý 2.6, ta tìm đượcx1;E x2E S khác chox1E x2E Zn VìS
cũng đối xứng tâm, x2E 2S, vìS tập lồi:
tx1E C.1 t / x2E 2S với mọi0t 1: Vớit D
2,
1 2x1E
1
2x2E 2S: Theo định nghĩa tậpS:
2
1 2x1E
1 2x2E
D Ex1 x2E 2E:
(21)Tạp chí Epsilon, Số 05, 10/2015
Với trường hợp vol.E/ D2nvàE compact, xét dãyEk D
1C k
E D
E x W k
1CkxE 2E
vol.Ek/ > 2n với k, nên ta áp dụng trường hợp cho Ek để có dãypkE Zn\Ek Vì tậpEk bị chặn đều, dãy pkE dãy bị chặn, nên theo Định lý
Bolzano-Weierstrass, tồn dãy hội tụ VìZnlà tập rời rạc vàE D
1
\
kD1
Ek, dãy hội tụ củapk cho ta véc tơ tọa độ nguyên trongE
Lưu ý 2.7 Điều kiện thể tích Định lý 2.4 tối ưu qua ví dụ sau: tậpE D˚ E
x 2RnWxE < tập lồi, đối xứng tâm tích bằng2n, nhưngE\ZnD f0g
2.2 Dạng tuyến tính (Linear Forms)
Xét hệ bất phương trình tuyến tínhnẩnnbất phương trình sau: ja1;1x1 C ::: C a1;nxnj < c1
::
: : :: ::: ::: jan 1;1x1 C ::: C an 1;nxnj < cn
jan;1x1 C ::: C an;nxnj cn
(2.2)
Áp dụng kết tập lồi cho phép ta tìm nghiệm ngun khơng hiển nhiên cho hệ bất phương trình tuyến tính trên:
Định lý 2.8 (Định lý Dạng tuyến tính Minkowski ) Giả sử ma trậnAD ai;j
1i;jn cójdet.A/j D 1,c1; c2; :::; cn> 0và
n Y
iD1
ci Thì hệ bất phương trình tuyến tính (2.2) có
nhất nghiệm ngun khác
Chứng minh Với mỗi1i n, gọiAi D.ai;1; ai;2; :::; ai;n/véc tơ dòng thứi ma trậnA Và vớikD1; 2; :::, xét hình bình hành nhiều chiều sau:
Ek WD
E
x2Rn WˇˇAi Exˇˇ< ci với1i n 1; ˇ
ˇAn Exˇˇ< cnC k
:
Bài tập 2.9 Chứng minh tậpEk tập lồi, đối xứng tâm, tích: vol.Ek/D2nc1c2:::cn
cnC k
> 2n:
Theo Định lý 2.4, ta tìm dãy véc tơ tọa độ nguyênzkE Ek khác Lập luận chứng minh Định lý 2.4 ta có véc tơ tọa độ ngun cần tìm
Chúng ta áp dụng Định lý Dạng tuyến tính để có chứng minh khác cho Định lý Dirichlet:
Chứng minh khác cho Định lý 1.3. Với véc tơxE 2Rn, xét ma trậnA2MnC1;nC1.R/như
sau: AD B B B B B @
1 : : : x1 : : : x2
::
: ::: : :: ::: ::: 0 : : : xn 0 : : :
1 C C C C C A D B B B B B @ A1 A2 :: : An AnC1
(22)Với mỗiQ1, áp dụng Định lý 2.8, ta tìm véc tơ tọa độ nguyên: E z D B B B @ p1 :: : pn q C C C A
2 ZnC1X0
sao cho
jqxi pj DˇˇAi Ezˇˇ<
Q với1i n jqj D ˇ
ˇAnC1 Ez ˇ
ˇQn:
Ta ch cn phi chng minh rngq Ô0 Gi s nhq D0, vỡzEÔ0, nờn tn tipi Ô0 iu ú dn đến:
1 jpij DˇˇAi Ezˇˇ<
Q (Vơ lý) Vậy ta có véc tơpED.p1; :::; pn/2Znvàq 2Z,q Ô0cn tỡm
2.3 Ci thin hng s Định lý Dirichlet trên Rn
Định lý 2.10 (Minkowski 1910) Với véc tơ xE RnXQn, tồn vô số véc tơ hữu tỉ
E p q D p1 q ; p2
q ; :::; pn
q
2 QnvipE2Znvq 2Z,q Ô0, cho:
E
x pE q < Cn jqj1C1n
vớiCnD n
nC1: (2.4)
Lưu ý 2.11 KhinD1, ta có đượcC1D
2, khơng phải số tối ưu p
5 Định lý Hurwitz (xem [11]) Có số kết cho số cho Định lý Dirichlet tốt Định lý 2.10, chẳng hạn Blitchfeldt [1] thay thếCnbằng:
n
nC1 1C
n 1 nC1
nC3! n :
Tuy nhiên số tối ưu cho Định lý Dirichlet trênRnvớin2vẫn câu hỏi mở quan trọng lý thuyết xấp xỉ Dirichlet Hình học số
Để chứng minh Định lý 2.10, với mỗiQ > 0vàC > 0, xét tập hợpEQ;C định nghĩa bởi: EQ;C D˚
.y; z/E D.y1; :::; yn; z/2 RnC1 WQ njzj CQyE
C :
Bổ đề 2.12 Với mỗiQ > 0vàC > 0,EQ;C tập compact, lồi, đối xứng tâm, tích:
vol.EQ;C/D 2C / nC1 nC1 :
(23)Tạp chí Epsilon, Số 05, 10/2015
Bài tập 2.13 Chứng minh rằngf hàm tuyến tính, với định thức 1, song ánh giữaEQ;C vàE1;C
Vậy nênf vàf bảo tồn tính chất compact, lồi đối xứng tâm, ta cần chứng minh trường hợpQD1
Tính compact đối xứng tâm tậpE1;C hiển nhiên Gọi.y; z/E và.yE0; z0/là điểm E1;C, với0t 1, áp dụng bất đẳng thức tam giác, ta có được:
ˇ
ˇt zC.1 t /z0 ˇ ˇC
tyEC.1 t /yE0
jt zj C ˇ
ˇ.1 t /z0 ˇ ˇC
tyE
C
.1 t /yE0 Dt jzj CyE
C.1 t / ˇˇz0 ˇ ˇC
yE0
C Vậyt y; z/E C.1 t /.yE0; z0/2 E1;C
Cuối ta tính thể tích củaE1(cũng mọiEQ):
vol.E1/D Z C
C
Z C jzj
jzj C : : :
Z C jzj
jzj C
dy1: : : dyndz
D2nC1 Z C
0
.C z/ndz
D 2C / nC1 nC1 :
Chứng minh Định lý 2.10. ĐặtC D.nC1/nC11 ma trậnAđược định nghĩa (2.3) Theo Bổ đề 2.12, tậpAEQ;C tập compact, lồi, đối xứng tâm, tích vol.AEQ;C/D 2nC1 Áp dụng Định lý 2.4, ta tìm véc tơ tọa độ nguyên.pQ; qQ/E khác nằm trongAEQ;C, nghĩa là.pQE ; qQ/thỏa mãn:
Q nˇˇqQ ˇ ˇCQ
qQxE pQE C:
Lưu ý với véc tơ tọa độ nguyên.p; q/, tồn hữu hạnE Q > 0thỏa mãn: Q njqj CQqxE pE
DC:
Vậy nên ngoại trừ số đếm cácQ > 0, baQ;pQ; qQE thỏa mãn bất đẳng thức: Q nˇˇqQˇˇCQ
qQxE pQE < C: (2.5) Với bộQ;pQ; qQE thỏa mãn (2.5), áp dụng bất đẳng thức trung bình cộng trung bình nhân cho ta:
ˇ ˇqQ
ˇ ˇ
qQxE pQE n
Dnn Q nˇˇqQ ˇ ˇ Q n
qQxE pQE
n
nn Q nˇ
ˇqQ ˇ ˇCQ
qQxE pQE nC1
!nC1
< nn
C nC1
(24)D n
nC1 n
DCnn;
tương đương với (2.4)
Nếu ta chnQ C thỡqQ Ô 0, vỡ nu nhqQ D 0,pQE ¤ 0và bất đẳng thức (2.5) dẫn đến:
qQxE pQE D
pQ< CQ 1 (vô lý): Lưu ý thêm với mỗi.p; q/, tập:E
˚
Q > W.p; q/E 2AEQ;C D˚
Q > 0WQ njqj CQq xE pEC
bị chặn Vì khiQ ! 1, ta tìm vô số.p; q/E thỏa mãn (2.4) Và với lập luận tương tự chứng minh Định lý 1.2 dựa vào Định lý 1.3, ta tìm vô số véc tơ hữu tỉ pE
q thỏa mãn (2.4)
Tài liệu tham khảo
[1] Blichfeldt, H.,A new principle in the geometry of numbers with some applications, Trans Amer Math Soc.15(1914), pp 227-235
[2] Cassels, J W S.,An introduction to the Geometry of Numbers, Springer (1959)
[3] Dirichlet, L G P.,Verallgemeinerung eines Satzes aus der Lehre von den Kettenbrăuchen nebst einigen Anwendungen auf die Theorie der Zahlen, S B Preuss Akad Wiss (1842), pp 93–95
[4] Euler, L.,Introductio in analysin infinitorum I, (1748)
[5] Gruber, P., Lekkerkerker, C.,Geometry of Numbers, North-Holland Mathematical Library (1987)
[6] Hardy, G., Wright, E M.,An introduction to the theory of numbers,5th ed., Clarendon Press (1979)
[7] Minkowski, H.,Geometrie der Zahlen, Teubner: Leipzig U Berlin (1896 & 1910)
[8] Rittaud, B., Heeffer, A.,The Pigeonhole Principle - Two centuries before Dirichlet, The Mathematical Intelligencer36, Springer (2014), pp 27–29.
[9] Schmidt, W M.,Diophantine approximation, Lectures Notes in Mathematics785, Springer (1980)
(25)CÂN BẰNG NASH
Vladimir Gurvich
Đề đề xuất cho Hội nghị mùa hè, Cuộc thi thành phố từ ngày3đến 11=8=2012:
1 Mở đầu
Điều chung cờ vây, cờ vua, cờ nhảy cờ ca rơ? Tất chúng trị chơi hữu hạn vị trí với thơng tin hồn hảo khơng có vị trí hội Ý cuối có nghĩa “tất các người chơi biết tất thứ” họ biết điều giống Điều không giống chơi hay chơi domino, nơi mà người chơi quân đối thủ Trong cờ cá ngựa, có vị trí hội, ta tung xúc sắc
Từ “vị trí” có nghĩa có bước từ vị trí đến vị trí khác
Từ “hữu hạn” có nghĩa có hữu hạn vị trí Bất kỳ cấu hình viên sỏi vị trí cờ vây Có nhiều (nhưng hữu hạn) vị trí Một ván cờ vị trí ban đầu kết thúc vị trí chung (mà ta gọi vị trí kết thúc) Ví dụ, nước chiếu bí vị trí kết thúc cờ vua Điều quan trọng phải nhận thấy vị trí lặp lặp lại ván đấu có vịng lặp Tất trị chơi nói trị chơi hai người có tổng khơng (zero-sum)
Từ “zero-sum” có nghĩa chiến thắng người thất bại người lại Và người số điểm số người điểm số Một cặp chiến lược tối ưu chúng tạo thành trạng thái cân bằng, nghĩa kết tương ứng cải thiện hai đối thủ Tuy nhiên, thứ trở nên phức tạp nhiều có nhiều hai người chơi (hoặc hai người chơi trị chơi khơng phải zero-sum) Có thể xảy người chơi đảm bảo kết kém, khó khăn để chiến đấu chống lại liên minh tất đấu thủ khác
Bài tập 1.1 Có10que diêm và3người chơi xoay vịng bốc que diêm, lần bốc1; 2; 3; 4;hoặc5que Người bốc que diêm cuối phải rửa bát Chứng minh hai người bất kỳ, thỏa thuận với nhau, buộc đấu thủ thứ ba rửa bát Giả sử rửa bát tính 2;trong người lạiC1:Tổng ván bằng0: Ta muốn xác định trạng thái cân trường hợp này, nghĩa là, đề xuất3chiến lược cho người chơi thay đổi chiến lược mình, người khơng lợi hai đối thủ giữ chiến lược cũ họ Khái niệm giới thiệu John Nash vào năm1950và gọi cân Nash
(26)Nash mơ tả quy luật mà việc tuân thủ thỏa thuận mà không cần chế áp buộc từ bên
Bài tập 1.3 “Gặp gỡ siêu thị:” Hai (hoặc ba) người bị lạc siêu thị mà khơng có sóng điện thoại di động Họ gặp trong3cửa vào, người lựa chọn cửa để cách độc lập lựa chọn người lại Nếu tất gặp nhau, người nhận đượcC1;nếu không người 1:Cân Nash trị chơi gì?
Lưu ý 1.Trị chơi cho hình thức bình thường (tất người chơi đưa lựa chọn lúc mà lựa chọn Sau người chơi lựa chọn xong xác định thắng hay thua)
Lưu ý 2.Một số siêu thị treo bảng “Nếu lạc gặp quầy tính tiền số1” Bài tập 1.4 Có hay khơng cân Nash trị chơi ca-rơ bàn cờ kích thước33: Hãy mơ tả chúng
Đầu tiên, chứng tỏ rằng trường hợp khơng lặp (trong khơng có vị trì lặp lại ) Nash - giải Chúng ta gán cho trò chơi đồ thị có hướng, mà đỉnh vị trí cạnh có hướng (cung) nước Các vị trí mà từ khơng di chuyển gọi vị trí kết thúc Ta gán cho vị trí kết thúc số điểm mà người chơi nhận vào vị trí Các vị trí lại chia người chơi, vị trí ta biết người chơi từ vị trí Giả sử từ vị tríP nước dẫn đến vị trí kết thúc, người chơi chọn nước thuận lợi cho vị tríT:Điểm củaT chuyển vềP:Và bây giờP trở nên xác định vị trí kết thúc Cách phân tích từ cuối làm cho vị trí xác định, có vị trí Chiến lược tối ưu nước người chơi đến vị trí mà điểm số lớn
Bài tập 1.5 Chứng minh với trò chơi khơng lặp chiến lược tối ưu nói tạo thành trạng thái cân Nash
Bây xây dựng đồ thị trò chơi cờ vua Chúng ta hiểu vị trí khơng cách quân cờ, mà thứ tự nước (đến lượt bên đi) Ngồi ta cần biết có quyền nhập thành, ăn tốt qua đường hay khơng, hay vị trí lặp lại trước chưa Một giải pháp để cung cấp cho vị trí thơng tin cần thiết ta nhớ với lịch sử trước Khi vị trí lặp lại khơng có, đồ thị khơng lặp hai người chơi có chiến thuật tối ưu
Nhưng cách hiểu không thú vị Luật lặp lại ba lần vị trí hiểu vị trí cách khác Cụ thể, xếp quân cờ, thứ tự nước đi, có hay khơng quyền nhập thành, bắt tốt qua đường Khi đồ thị có chu trình có hướng, phân tích từ cuối khơng thể thực Ta làm rõ khái niệm chiến thuật
Ta gọi quy tắc chọn nước xác định vị trí đến lượt người chơi chiến thuật ổn định Ví dụ trị chơi với que diêm chiến thuật tham lam đạo lần bốc nhiều số que diêm
(27)Tạp chí Epsilon, Số 05, 10/2015
Bài tập 1.6 Có hay khơng cân Nash cờ vua? Và bỏ luật hòa vị trí lặp lại3lần hay50nước khơng ăn qn tốt?
Từ đầu đến ta xét trị chơi có kết thúc, mà kết trò chơi (chi trả) xác định vị trí kết thúc vịng lặp Trong số trị chơi, ngồi điều này, người chơi cịn nhận phải trả qua nước đi, kết cuối xác định cho người chơi tổng tất thu chi
Bài tập 1.7 Trên bàn có5que diêm Ba người bốc1hoặc2que diêm Người bốc que diêm cuối thưởng3que diêm Số điểm số que diêm bốc Hãy xây dựng đồ thị trị chơi tìm chiến thuật tối ưu cho tất người chơi
Nếu trị chơi kết thúc vịng lặp ta giả sử vịng lặp diễn vơ hạn lần Khi kết người chơi hữu hạn tổng thu chi bằng0:Trái lại tổng C1hoặc 1:
Bài tập 1.8 Có100tên cướp khát máu cướp nhà băng được1triệu đô-la ngồi chia tiền Đầu tiên, người thứ đề xuất phương án: Tôi chừng này, người thứ hai chừng này, người thứ ba chừng sau cả100người biểu Nếu có nhất1nửa đồng ý đề xuất chấp thuận họ chia tiền theo phương án giải tán Nếu có nửa chống, bọn cướp thủ tiêu người thứ người thứ hai lại đề xuất phương án chia mới, Mỗi tên cướp đạo mong muốn trước tiên sống, sau (nếu mạng sống không bị de dọa) nhiều tiền cuối (nếu mạng sống số tiền không bị ảnh hưởng) – thủ tiêu nhiều tốt (dân xã hội đen mà) Hỏi tiền chia tất tên cướp hành động suy luận hoàn toàn logic (tức hay tìm cân Nash)
2 Dẫn nhập “khơng có định nghĩa chặt chẽ”
Ta xét câu hỏi sau:Những trị chơi vị trí với đầy đủ thơng tin có cân Nash các chiến thuật túy ổn định? Trong số trường hợp câu trả lời biết Ta điểm qua trường hợp vậy, bỏ qua định nghĩa xác
Cân Nash tồn cho lớp sau:
.1/ Các trị chơi khơng lặp: Trong trị chơi này, vị trí khơng thể lặp lại Trong trường hợp tồn cân Thế Cờ vua Cờ vây có lặp lại vị trí
.2/ Các trị chơi đối kháng hai người chơi: Lớp có Cờ vua Cờ vây Nhưng lợi ích hai bên khơng đối kháng sao? Hay số người chơi lớn hơn2thì sao?
.3/ Nếu nước người chơi phụ thuộc vào lịch sử trước đó: Tuy nhiên giới hạn người chơi dùng chiến thuật ổn định Nói cách khác, nước phụ thuộc vào vị trí chọn hồn tồn xác định, khơng có ngẫu nhiên Ví dụ trị chơi có dùng quân xúc sắc bị loại bỏ
(28)thơng tin đầy đủ (ví dụ trị chơi domino) Nhưng ta khơng xét trị chơi vậy, chí cịn khơng định nghĩa chúng
Tóm tắt lại:Ta giới hạn việc xem xét trị chơi với thơng tin đầy đủ, khơng có nước đi ngẫu nhiên với chiến thuật túy ổn định Trong số người chơi lớn hơn2và trong trường hợp2người lợi ích họ khơng thiết phải đối kháng.
Rất đáng ngạc nhiên trường hợp ta chưa có nhiều tiến triển Có số cách tiếp cận, đơn giản cân Nash (ta định nghĩa đây) Mặc dù cơng trình cân Nash đến5giải thưởng Nobel kinh tế có cảm nhận câu hỏi toán học “đơn giản” tự nhiên câu hỏi mở Ở đề xuất hai câu hỏi - giả thuyết Các giả thuyết kiểm chứng, với trợ giúp máy tính với ví dụ đủ lớn (nhưng không lớn) Tôi hy vọng có câu trả lời khẳng định, khơng ngạc nhiên có phản ví dụ Trong giả thuyết có trường hợp riêng tương đối đơn giản mà ta luyện tập Tuy nhiên, trường hợp riêng khác phức tạp có số trường hợp ta chưa biết kết giống trường hợp tổng quát
3 Các định nghĩa bản
Tơi có cảm nhận định nghĩa hiển nhiên cách trực giác Tuy nhiên, cách phát biểu hình thức làm “sợ” Nếu vậy, bỏ qua việc đọc phần lần đọc sử dụng từ điển sổ tay tra cứu
3.1 Đồ thị trò chơi, vị trí nước đi
Cho đồ thị có hướng hữu hạnG D.VI E/:Mỗi đỉnhv 2V vị trí trị chơi cạnh có hướnge D.v; v0/là nước vị trív:Các vị tríVT V;mà khơng có nước đi, gọi vị trí kết thúc Ta chọn vị trí khởi đầuv0 2V VT:
Mỗi vị trí khơng kết thúcv 2V nVT ta cho tương ứng với người chơii I 2I D f1; 2; : : : ; ng;
người chọn nước vị trív:Ta nóiikiểm sốt v viếti D.v/:Nói cách khác, ánh xạ WV nVT !I phân phối vị trí khơng kết thúc cho người chơi Bộ bafG; ; v0gđược gọi cấu trúc vị trí
3.2 Chiến thuật tình huống.
Chiến thuật xi người chơi i I kế hoạch cho phép chọn nước e D v; v0/ vị trív 1.i /;điểm kiểm sốt bởii;nói cách khác ánh xạxi cho tương ứng vị trí v 1.i /một nước đie D v; v0/từv:Đây chiến thuật túy ổn định Như nói trước đây, chiến thuật khác ta không xét đến khơng định nghĩa
(29)Tạp chí Epsilon, Số 05, 10/2015
thúc vị trí kết thúcv 2VT bị lặp, tức xuất chu trình có hướngC mà sau lặp lại vơ hạn (ván đấup.x/khơng thể khỏiC;vì tất chiến thuật ổn định) Như thu ánh xạg WX !P mà tình huốngx 2X cho tương ứng với ván đấup 2P:Các ánh xạ gọi thể thức chơi
3.3 Hàm lượng giá
Mỗi người chơii 2I sau nước đie 2E trả khoảng phíc.i; e/2R:Số thực gọi giá địa phương (nếu nhưc.I; e/ < 0;thìikhơng trả mà ngược lại nhậnˇˇc.I; e/ˇˇ/: Cấu trúc vị trí tốn địa phương xác định trị chơi hình thức vị trí Giá hiệu ván đấup Dp.x/được định nghĩa cho người chơii 2I sau Nếup kết thúc vị trív 2VT giá nóc.i; p/DX
e2p
c.i; e/là cộng tính, tức tổng giá tất nước củap:Nếupbị lặp, ta cần tính giác.i; C /D X
e2C
c.i; e/của xíchC tương ứng đối vớii:Nếu nhưc.i; C />0;thìc.i; p/D 1vàc.i; p/D 1;nếu nhưc.i; C / < 0:
Định nghĩa tự nhiên, chu trình lặp lại vơ hạn lần, giá địa phương cộng lại Tuy nhiên, ván đấu bị lặp “chu trình0”,c.i; C /D 0;ta đặtc.i; p/D 1: Đây quy ước cho tiện lợi
Thể thức chơi g giá hiệu quảc xác định trò chơi.g; c/ở dạng chuẩn Tất nhiên, mội người chơiiđều cố gắng tối thiếu hối giá hiệu quảc.i; p/của
Trò chơi kết thúc:Nước đie D.v; v0/được gọi nước kết thúc nếuv0 2VT:Chú ý nước kết thúc thuộc vào chu trình Hàm lượng giá (và trò chơi) gọi kết thúc nếuc.i; e/ 0với người chơii với nước khơng kết thúce:Trong trường hợp này, giá trị chơi phụ thuộc vào vị trí kết thúc Nếu ván đấup bị lặp giá 1hoặc 1:
Trị chơi với tổng 0:Hàm lượng giá (và trị chơi) có tổng0nếuX i2I
c.i; e/D0với nướce 2E:Trị chơi cho2người,nD2với tổng0đóng vai trị quan trọng mặt lịch sử lẫn theo chất Mọi trị chơi vớinngười chuyển dễ dàng thành trị chơinC1 người với tổng0:Chỉ cần đưa vào người chơi thứnC1– “ơng tám” (người khơng kiểm sốt vị trí nào) xác định giá địa phương theo công thứcc.nC1; e/D
n X
iD1
c.i; e/:
Trò chơi dạng chuẩn:Định nghĩa chung
Ta giả sửI D f1; 2; : : : ; nglà tập hợp người chơi,Xi – tập hữu hạn chiến thuật người chơi thứi I vàX DX1X2 Xnlà tích Đề-các chúng, tức tập hợp tình
Tiếp theo, giả sửP tập hợp kết trò chơi (trong trường hợp ván đấu) Mọi ánh xạg WX !P gọi thể thức chơi
(30)Cân Nask điểm yên ngựa:Tình huốngx D x1; x2; : : : ; xn/X1 Xn D X gọi cân Nash thay đổi chiến thuật người chơii 2I (nhưng người) không đem lại lợi ích cho anh ta, tức không làm giảm giá Một hình hình thức viết này:c.I; g.x//c.I; g.x0//với người chơii I tình huốngx0 2X mà tọa độ (chiến thuật) giống củaxngoại trừ tọa độI;nói cách khác, cóxi0 khácxi:
Khái niệm đưa nởi John Nash năm1950:Trong trường hợp trò chơi2người với tổng0cân Nash có tên điểm yên ngựa Khái niệm đời từ200năm trước Khác với điểm yên ngựa, khái niệm cân Nash nhạy cảm trích Thơng thường hai người chơi thay đổi chiến thuật lúc hai đoán Hơn thế, điều xảy với cảnngười chơi Tình cân (trong chiến thuật túy) khơng tồn nói chung Và tồn chín có nhiều Hơn nữ, khơng cân mà tốn cân có nhiều Điểm n ngựa khơng có đa số điểm yếu Tuy nhiên, đả phá Nash mục tiêu (chúng ta nhờ về5giải Nobel)
Cân Nash nhất:Tình huốngx 2X gọi cân Nash cân khơng với vị trí khởi đầu chov02 V;mà với vị trí kết thúcv00 2V khác
4 Các toán giả thiết
Chúng ta quan tâm đến định lý tồn cân Nash (tức giải theo Nash) trị chơi vị trí định nghĩa (độ phức tạp toán đánh giá số điểm viết ngoặc)
Giả thuyết 1.500/:Có phải trị chơi vị trí cho2người giải theo Nash? Bài tốn 1.10/:Hãy chứng tỏ rằng, khơng tính tổng qt ta giả sử khơng có “chu trình0”, nói chặt chẽ chu trình có hướng với tổng giá địa phương bằng0:Nói khác, không tỗng quát giả sử rằngX
e2C
c.i; e/Ô0vi mi chu trỡnh cú hngC v vi mi người chơii 2I D f1; 2g:
Nhắc lại giá hiệu ván đấu có lặp bằngC1hoặc 1:
Đây giả thuyết hoàn toàn Vladimir Udalov viết chương trình khẳng định giả thuyết cho nhiều đồ thị có hướng với10 18đỉnh
Bài tốn 2.25/:Giả thuyết1khơng tổng qt lên cho trường hợp3người chơi Hãy xây dựng ví dụ
Đối với trò chơi cho2người với tổng bằng0giả thuyết chứng minh phức tạp Hơn thế, trường hợp đưa vào giá hiệu hữu hạn ván đấup;kết thúc “chu trình0”C;sao cho điểm yên ngựa tồn (nhắc lại theo định nghĩa c.i; p/D C1trong trường hợp này) Tuy nhiên tái định nghĩa không đơn giản Bài tốn 3.70/:Hãy thử tìm định nghĩa chứng minh tính giải Chứng tỏ “cố gắng đơn giản” khơng qua cửa Ví dụ đặtc.i; p/D0hayc.i; p/DX
e2p
c.i; e/
(31)Tạp chí Epsilon, Số 05, 10/2015
Giả thuyết 500/:Có phải trị chơi vị trí với nngười chơi, giá địa phương khơng âm giải theo Nash?
Bài toán 3a.5/:Chứng minh ta cần xét trường hợp giá địa phương lớn hơn0:Giả thuyết chưa chứng minh trường hợp “rất riêng”
Giả thuyết 2a.300/:Thanh tốn kết thúc Trong giá hiệu ván đấu lặp người chơi bằngC1:
Giả thuyết 2b.400/:Thanh tốn kết thúc Trong chu trình tạo kết quả,NHƯNGkhông thiết kết tệ cho người chơi Thay điều này, ta giả sử người chơi đề xếp tất vị trí kết thúc kết chu trình cách tùy ý
Giả thuyết 2c.300/:Trường hợp hai người chơinD2:Trong trường hợp ta gom hai Mệnh đề giả thuyết1và2lại
Bài toán 4.100/:Chứng minh trường hợp hai người chơi hàm giá kết thúc Giả thuyết2vẫn
Kết suy từ định lý cũ tơi, năm1975:
Theo định nghĩa, thể thức trị chơi cho n người g W X1 X2 Xn ! P giải theo Nash trò chơi tương ứng.g; c/có cân Nash với hàm lượng giá c WI P !R:Ở đâyc.i; p/– giá kết quảp 2P cho người chơii 2I:
Trong trường hợp2người chơiI D f1; 2gbên cạnh với định nghĩa tổng quát tính giải ta xét tính chất yếu hơn: Thể thức chơi hai ngườigđược gọi giải đới kháng giải lớp trò chơi với tổng0:Cuối cùnggđược gọi là˙1giải giải lớp trị chơi2người với tổng0;trong hàm lượng giá nhận2giá trị:C1hay
1:
Bài tốn 5.100/:Chứng minh tất ba tính chất (giải được, giải đối kháng giải được˙1/là tương đương
Sự tương đương hai tính chất cuối chứng minh nhiều năm trước đây, vào năm 1973; cịn trước nữa, Edmonds, J.; Fulkerson chứng minh Ed-monds, J.; Fulkerson, D R 1970/; “Bottleneck extrema”, Journal of Combinatorial Theory 8W3 1970/ 299 306:Rất đáng tiếc, kết luận tốn5khơng tổng qt hóa lên cho trường hợp trị chơi3người Ta phát biểu điều chặt chẽ Mọi thể thức chơi nngười, I D f1; ; 2; : : : ; ngcó thể cho tương ứng vớinthể thức chơi cho2người, người chơii chơi vớiI n figvớii 2I:
Bài tốn 5a.50/:Hãy nêu ví dụ thể thức chơi với3người chơi, không giải theo Nash, cho3thể thức chơi2người tương ứng với giải
Bài tốn 5b.20/Hãy nêu ví dụ ngược lại, trị chơi3người giải theo Nash thể thức chơi2người tương ứng với khơng giải
Bài tốn 6.20/:Hãy chứng tỏ tốn4có thể đưa tốn5:
Bài toán 7.15/:Chứng minh cân Nash tồn đồ thịGkhơng lặp (khơng có chu trình có hướng)
(32)Kết thu bở Harold Kun.1952/và David Geil.1953/ngay sau Nash đưa khái niệm cân
Bài toán 7a.20/:Chứng minh với trị chơi khơng lặp cân Nash tồn trường hợp cho phép có vị trí may mắn (trong cho phân phối xác suất) Tất nhiên, để giải hai ta được20điểm tối đa, không phải35:
Bài toán 8.40/:Chứng minh cân Nash (điểm n ngựa) tồn cho trị chơi vị trí hai người với tổng0:
Nói riêng kết áp dụng cho cờ vua cờ vây Kết Ernst Zermelo báo cáo Đại hội toán học giới lần thứ5vào năm1912:Báo cáo ơng có tên “Về ứng dụng lý thuyết tập hợp vào cờ vua”
Tôi lưu ý trường hợp này, kết mở rộng, cho phép vị trí may mắn Tuy nhiên điều đưa xa phía trị chơi ngẫu nhiên Vì sau ta để hướng qua bên, dành cho lần khác
Bài tốn 9.10/:Ta quy ước kết thúc trị chơi từ lần vị trí lặp lại Và kết trị chơi chu trình thu Hãy chứng minh trường hợp đồ thị hữu hạn thay hữu hạn (trong khơng khơng có chu trình có hướng, mà nói chung khơng có chu trình) Tại Giả thuyết1và2khơng thể suy từ tốn7‹ Bài tốn 10.15/Hãy nêu ví dụ trị chơi kết thúc hai người chơi có1chu trình khơng có cân (chu trình khơng thiết phải kết tệ cho hai đối thủ Mỗi người họ xếp vị trí kết thúc chu trình cách tùy ý) Bài tốn 11.100/Hãy nêu ví dụ tro chơi kết thúc hai người chơi khơng có cân có chu trình, kết tệ cho hai đối thủ
Bài tốn 12.25/Hãy nêu ví dụ tương tự trị chơi kết thúc3người chơi: Trong khơng có cần có chu trình kết tê cho cả3người chơi
(33)MỞ RỘNG CÁC BÀI TỐN HÌNH HỌC EUCLID THÀNH CÁC BÀI TỐN HÌNH HỌC CẦU VÀ HÌNH HỌC LOBACHEVSKY - MỘT PHƯƠNG THỨC SÁNG TẠO
CÁC BÀI TỐN MỚI
Nguyễn Ngọc Giang – TP Hồ Chí Minh
TĨM TẮT
Sáng tạo tốn ln niềm đam mê đích tới nhà tốn học Tuy nhiên câu hỏi ln đặt là, làm để phát toán mới? Để trả lời câu hỏi này, cần đến phương pháp phát triển mở rộng toán Ở bậc đại học, học phương pháp thế, phương pháp afin-xạ ảnh Tuy nhiên, phương pháp afin-xạ ảnh khơng phải phương pháp Có phương pháp hay hấp dẫn phương pháp afin-xạ ảnh, phương pháp mở rộng tốn hình học Euclid1thành tốn hình học cầu hình học Lobachevsky Nội dung phương pháp tìm chứng minh tốn tổng quát hình học Euclid hình học cầu hình học Lobachevsky Trong báo tìm hiểu tốn, khái niệm, tính chất so sánh chúng ba thứ hình học Đặc biệt, tốn, khái niệm, tính chất nhìn ”con mắt” Euclid nên dễ hiểu, dễ tiếp nhận
1 So sánh hình học Euclid, hình học cầu hình học Lobachevsky
Trong hình học cầu, bán kính cầuRcho ta biết điều, bán kínhRcàng lớn hình học phạm vi gần hình học Euclid Vì bán kính mặt cầuRcịn gọi bán kính cong Người ta chứng minh
R2 độ cong toàn phần không đổi mặt cầu R2 độ cong toàn phần mặt phẳng Lobachevsky Ta thêm dấu trừ để khác biệt với hình học Euclid Hình học Lobachevsky diễn theo hướng ngược với hình học cầu so với hình học Euclid Hình học Euclid (hai chiều) hình học mặt phẳng có độ cong tồn phần khơng Như vậy, hình học Euclid trường hợp giới hạn hình học mặt cầu (khi R ! 1/và giới hạn hình học mặt cong có độ cong tồn phần âm khơng đổi
R2 (khiR ! 1/
Ta quy ước khái niệm thông thường đường thẳng, tam giác, tiếp tuyến, đường tròn, cung
1
(34)tròn mà khơng nói thêmcó nghĩa làcác khái niệm hình học Euclid.Ta quy ước khái niệm đường thẳng, đường trịn hình học Lobachevsky có thêm kí hiệuLđi kèm Ví dụ đường thẳngL A,L AB có nghĩa đường thẳng quaA, đường thẳngAB hình học Lobachevsky, đường trịnL OIOA/là đường trịn tâmObán kínhOAtrong hình học Lobachevsky Đường thẳng, đường trịn, hình học cầu có thêm kí hiệuS kèm Ví dụ đường thẳngS ABcó nghĩa đường thẳngABtrong hình học cầu Ta quy ước sinAB
R , tan AB
R sin S AB
R !
;tan S AB R
!
; sinhAB R ;tanh
AB
R sinh L AB
R !
;tanh L AB R
!
Ta quy ước mục 1.1, 2.1, 3.1, , n.1, khái niệm, định lí hình học Euclid; mục 1.2, 2.2, 3.2, , n.2, khái niệm hình học cầu; mục 1.3, 2.3, 3.3, , n.3, mục hình học Lobachevsky Sau mục so sánh khái niệm, tính chất, hệ thức, định lí cách dựng đối tượng ba thứ hình học Euclid, cầu Lobachevsky [4]:
1.1 Điểm
1.2 Điểm nằm mặt cầu
1.3 Điểm nằm phía trục-xcho trước
2.1 Điểm vô tận (trong mặt phẳng Euclid mở rộng) 2.2 Khơng có tương ứng
2.3 Điểm thuộc trục-x 3.1 Khơng có tương ứng 3.2 Khơng có tương ứng 3.3 Điểm nằm phía trục-x 4.1 Đường thẳngAB
4.2 Đường tròn lớn quaA; B giao mặt phẳng.OAB/với mặt cầu đường thẳng S AB:
4.3 Nửa đường tròn có tâm trục-xđi quaA; B đường thẳngL AB Cách dựng sau: - Dựng đường trung trực đoạnAB cắt trục-xtạiO:Nửa đường tròn.OIOA/đi quaA; B nửa đường trịn cần dựng
(35)Tạp chí Epsilon, Số 05, 10/2015
5.1 Đoạn thẳngAB
5.2 CungABd(cung nhỏ) đoạn thẳngS AB:
5.3 CungABdcủa nửa đường trịn có tâm trục-xđi quaA; B đoạn thẳngL AB 6.1 Độ dài đoạn thẳngAB
6.2 Độ dài cungdABlà độ dài đoạn thẳngS AB:
6.3 - Dựng cungABd nửa đường trịn có tâm trục-xđi quaA; Bcắt trục-xtại hai điểm vô tậnP; Q:
- Đo độ dài đoạn thẳngAP ; AQ; BP ; BQ:
- Gọitỉ số képcủa.AB; PQ/là.AB; PQ/D AP =AQ BP =BQ: - Đặtd D jln.AB; PQ/jthìd độ dài đoạn thẳngL AB:
7.1 Định lí: Có đường thẳng qua điểm song song với đường thẳng cho trước
7.2 Khơng có đường thẳng song song hình học cầu Hai đường thẳng ln cắt 7.3 - Có hai đường thẳng qua điểm song song với đường thẳng cho trước - Hai đường thẳng cắt nhau, song song phân kì
- Có vơ số đường thẳng qua điểm khơng có điểm chung với đường thẳng cho trước 8.1 Độ lớn gócACB[
8.2 - Cho hai cung trònCA;d CBdthuộc đường tròn lớn mặt cầu
(36)8.3 - Dựng hai cung tròndCA;CBd hai đoạn thẳngL CA; L CB:(Xem 5.3) - Dựng hai tiếp tuyếnCA0; CB0với hai cung tròn tạiC BB0?CB0; AA0?CA0/ - Độ lớn gócA\0CB0chính độ lớnL ACB[.
9.1 Đường phân giácC C0của gócACB[ 9.2 - Dựng gócS ACB[ làA\0CB0.
- Dựng phân giácC C0của gócA\0CB0.
- Dựng đường tròn lớn.OCD/ quaC tiếp xúc với C C0 C CD phân giác góc S ACB:[
9.3 - Dựng gócL ACB[ gócA\0CB0vớiCA0; CB0được dựng 8.3.
(37)Tạp chí Epsilon, Số 05, 10/2015 - Dựng đường thẳngd?C C0: -d cắt trục-xtạiO0:
- Nửa đường tròn.O0IO0C /chính đường phân giácC C0củaL ACB[
10.1 Đường thẳng vng góc với đường thẳng cho trước điểm nằm đường thẳng 10.2 - Đường tròn lớn qua hai điểmB; C đường thẳngS BC:
- GọiAlà điểm nằm đường thẳngS BC:
- QuaO dựng đường thẳngd vng góc với mặt phẳng chứa đường tròn lớn qua hai điểmB; C: - Mặt phẳng.A; d /cắt mặt cầu theo giao tuyến đường tròn lớn quaA:Đường trịn đường thẳngL Ađi quaAvà vng góc vớiBC:
(38)- GọiO tâm đường tròn nằm trục-x qua hai điểmA; B: - NốiOA:
- GọiO0là điểm trục-x choO0A?OA:
- Dựng đường trịn.O0IO0A/thì nửa đường trịn trục-x quaAlà đường thẳngL Acần dựng
11.1 Đường thẳng vng góc với đường thẳng cho trước điểm khơng nằm đường thẳng
11.2 - Đường trịn lớn qua hai điểmB; C đường thẳngS BC: - GọiAlà điểm nằm đường thẳngS BC:
- QuaO dựng đường thẳngd vng góc với mặt phẳng chứa đường tròn lớn qua hai điểmB; C: - Mặt phẳng.A; d /cắt mặt cầu theo giao tuyến đường tròn lớn quaA:Đường trịn đường thẳngS Ađi quaAvà vng góc vớiS BC:
11.3 - Dựng đường trịn.O/đi qua hai điểmA; B có tâmO trục-x Nửa đường trịn phía trục-xnày đường thẳngL AB:
(39)Tạp chí Epsilon, Số 05, 10/2015
- Dựng quaO đường vng góc vớiOC cắt nửa đường trịn phía trên.OIOC /tạiF: -OC; OF cắt đường thẳngL AB tạiD; E:
- Dựng quaE đường thẳng song song vớiDF cắtOC tạiG: - GọiO0là giao đường trung trực đoạnC Gvới trục-x:
- Nửa đường trịn.O0IO0C /phía trục-x đường thẳngL C quaC vng góc vớiL ABcần dựng
12.1 Trung điểmM đoạn thẳngCD 12.2 - Cho đoạn thẳngS CD
- GọiM0là trung điểm đoạn thẳngCD:
- TiaOM0cắt đoạn thẳngS CD tạiM thìM trung điểm đoạn thẳngS CD: 12.3 - Gọi đường tròn qua hai điểmC; D có tâm nằm trục-xlà.O/:Nửa đường trịn phía chứaC; Dchính đường thẳngL CD:
- Đường thẳngCDcắt trục-xtạiO0: - GọiH trung điểm củaOO0:
- Đường trịn.HIHO/cắt đường thẳngL CDtạiM thìM trung điểm cần dựng
13.1 Trung trực đoạn thẳngCD
13.2 - Dựng trung điểmM đoạn thẳngS CDnhư cách dựng 12.2 - Dựng đường thẳng quaM vng góc vớiS CD tạiM cách dựng 10.2 13.3 - Dựng trung điểmM đoạn thẳngL CD cách dựng 12.3
(40)14.1 Ảnh đối xứngA0của điểmAqua đường thẳng quaM vng góc vớiAM cho trước 14.2 - Dựng đường thẳngS AM;S mvng góc vớiS AM tạiM:
- Dựng đường thẳng quaAvng góc vớiOM cắt đường thẳngS AM tạiA0 Thế thìA0là điểm cho đoạn thẳngS AM; S A0M nhau, nghĩa làS AM ŠS A0M:
14.3 - Dựng đường thẳngL AM:
- Dựng đường thẳngL mquaM vng góc với đường thẳngL AM cách dựng 10.3 Đường thẳngL mlà nửa đường trịn có tâm trục-x làO:
- Dựng đường thẳngOAcắt đường thẳngL AM điểmA0:Thế thìA0là điểm cần dựng
15.1 Đường trịn tâmO bán kínhOP
(41)Tạp chí Epsilon, Số 05, 10/2015
15.3 Cho điểmO điểmP:Dựng đường trònL OIOP /: - Dựng đường thẳngl quaO vng góc với trục-x
- DựngP0là ảnh củaP qua đường thẳngL O vng góc với đường thẳngL OP tạiO cách dựng 14.3 Thế thìL OP ŠL OP0:
- Dựng đường trung trực đoạnPP0cắtl tạiO0: - Đường tròn.O0IO0P /là đường tròn cần dựng
16.1 Đường trịn tâmO có bán kínhRbằng đoạn thẳngAB cho trước 16.2 Cho điểmO đoạn thẳngS AB:
- Dựng đường trung trựcS d đoạnS OAnhư cách dựng 13.2
(42)16.3 Cho điểmO, dựng đường trònL OIOP /vớiOP độ dài đoạn thẳngAB cho trước L OP ŠL AB
- Dựng đoạn thẳngL OA:DựngL l đường trung trực đoạn thẳngL OAnhư cách dựng 13.3
- Dựng đường thẳngL mquaB vuông góc với đường thẳngL l điểmM: - GọiP ảnh củaBnằm đường thẳngL mđi quaM:
- Đường trònL OIOP /là đường tròn cần dựng cách dựng 15.3
17.1 Định lí hàm số cơsin:a2 Db2Cc2 2bccosA 17.2 Định lí cơsin-S: cos a
R Dcos b Rcos
c R Csin
b R sin
c
RcosA (Chứng minh: [2]) 17.3 Định lí cơsin-L: cosh a
R Dcosh b R cosh
c
R sinh b Rsinh
c
RcosA(Chứng minh: [1]) 18.1 Định lí hàm số sin: a
si nA D b sinB D
c sinC 18.2 Định lí sin-S: sin
a R sinA D
sinRb sinB D
sinRc sinC 18.3 Định lí sin-L: sinh
a R sinA D
sinhRb sinB D
(43)Tạp chí Epsilon, Số 05, 10/2015
19.1 Định lí Phương tích điểm đường tròn: Nếu từ điểmP ta kẻ hai cát tuyếnPMM0; PN N0tới đường tròn cắt đường tròn cặp điểmM; M0I N; N0thì ta có hệ thức
PM:PM0 D PN:PN0:
19.2 Nếu từ điểmP ta kẻ hai cát tuyếnS PMM0; S PN N0tới đường tròn cắt đường trịn cặp điểmM; M0I N; N0thì ta có hệ thức:
tanPM 2R :tan
PM0
2R Dtan PN
2R :tan PN0
2R :
19.3 - Nếu từ điểmP ta kẻ hai cát tuyếnL PMM0; L PN N0tới đường tròn cắt đường tròn cặp điểmM; M0I N; N0thì ta có hệ thức
tanhPM 2R :tanh
PM0
2R Dtanh PN
2R :tanh PN0
2R : 20.1 Định lí Ménélaus
20.2 Điều kiện cần đủ để ba điểmA0; B0; C0 theo thứ tự nằm ba cạnhS BC; S CA; S AB tam giácS ABC thẳng hàng
sinAR0B sinAR0C
:sin B0C
R sinBR0A
:sin C0A
R sinCR0B D
1:
20.3 Điều kiện cần đủ để ba điểm A0; B0; C0 theo thứ tự nằm ba cạnhL BC; L CA; L ABcủa tam giácL ABC thẳng hàng
sinhAR0B sinhAR0C
:sinh B0C
R sinhBR0A
:sinh C0A
R sinhCR0B D
1:
21.1 Định lí Céva
21.2 Điều kiện cần đủ để ba đường thẳngS AA0; S BB0; S C C0theo thứ tự nối đỉnhA; B; C với điểmA0; B0; C0nằm ba cạnhS BC; S CA; S ABcủa tam giác S ABC đồng quy
sinAR0B sinAR0C :
sinBR0C sinBR0A:
sinCR0A
sinCR0B D 1:
21.3 Điều kiện cần đủ để ba đường thẳngL AA0; L BB0; L C C0theo thứ tự nối đỉnhA; B; C với điểmA0; B0; C0nằm ba cạnhL BC; L CA; L AB tam giác L ABC đồng quy
sinhAR0B sinhAR0C:
sinhBR0C sinhBR0A:
sinhCR0A
sinhCR0B D 1: 22.1 Định lí ba đường cao
22.2 Ba đường cao hình học-S đồng quy
(44)- Nếu hai đường cao cắt điểmO đường cao thứ ba quaO: - Nếu hai đường cao phân kì đường cao thứ ba phân kì với chúng Cả ba nhận chung đường vng góc
- Nếu hai đường cao song song với phía đường cao thứ ba song song với chúng phía
23.1 Định lí ba đường trung tuyến
23.2 Ba đường trung tuyến-S tam giác-S đồng quy 23.3 Ba đường trung tuyến-Lcủa tam giác-Lđồng quy 24.1 Định lí ba đường phân giác
24.2 Ba đường phân giác trong-S tam giác-S đồng quy 24.3 Ba đường phân giác trong-Lcủa tam giác-Lđồng quy
25.1 Định lí hai đường phân giác đường phân giác
25.2 Trong tam giác-S;hai đường phân giác ngoài-S đường phân giác trong-S thứ ba đồng quy
25.3 Trong tam giác-L, hai đường phân giác ngoài-Lvà đường phân giác trong-Lthứ ba đồng quy
26.1 Định lí ba đường trung trực
26.2 Trong tam giác-S, ba đường trung trực-S đồng quy 26.3 Trong tam giác-L, ba đường trung trực-Lđồng quy
2 Dùng hình học cầu chứng minh hình học Lobachevsky
2.1 Phương pháp
Để chứng minh định lí hình học Lobachevsky, ta chứng minh định lí hình học cầu nhờ hàm lượng giác tỉ số a
R; b R;
c
R; v; v; :::vớia; b; c là độ dài đoạn
thẳng cầu Bây giờ, chứng minh ta thay mọiR bằngRi thì ta lại chứng minh khác cho phép ta khẳng định, định lí hình học Lobachevsky đúng.
2.2 Ví dụ minh họa
Bài toán (Định lý Céva-L).
Điều kiện cần đủ để ba đường thẳngL AA0; L BB0; L C C0theo thứ tự nối đỉnh
A; B; C với điểmA0; B0; C0 nằm ba cạnhL BC; L CA; L AB của tam giác
L ABC đồng quy là
sinhAR0B sinhAR0C:
sinhBR0C sinhBR0A:
sinhCR0A
sinhCR0B D 1:
Lời giải.Để chứng minh định lí Céva-Lta chứng minh cho định lí Céva-S Trong chứng minh định lí Céva-S, ta sử dụng hàm lượng giác Sau thayRbởiRi ta chứng minh cho định lí Céva-L
(45)Tạp chí Epsilon, Số 05, 10/2015
Điều kiện cần đủ để ba đường thẳngS AA0; S BB0; S C C0theo thứ tự nối đỉnh
A; B; C với điểm A0; B0; C0nằm ba cạnh S BC; S CA; S AB của tam giác
S ABC đồng quy là
sinAR0B sinAR0C :
sinBR0C sinBR0A:
sinCR0A
sinCR0B D 1: Điều kiện cần: Các trường hợp biểu diễn hình vẽ
Từ hình vẽ xét, bỏ qua việc xét dấu, ta có: sin A0B
R sinBOA\0 D
sinOBR sinOA\0B
sinAR0C sin\COA0 D
sinOCR
sinOA\0B( gócS OA\
0B vàS OA\0C là bù nhau).
Tương tự, ta có: sinA
0B
R D
sinOBR :sin\BOA0
sinOA\0B sin
A0C R D
sinOCR :sinCOA\0
sinOA\0B :
Tiếp theo: sin A0B
R sinAR0C D
sinOBR sinOCR :
sinBOA\0
(46)Các tam giácS OCB0vàS OAB0cho: sinBR0C sinBR0A D
sinOCR sinOAR :
sinCOB\0
sinAOB\0.2/
Các tam giácS AOC0vàS BOC0: sin C0A
R sinCR0B D
sinOAR sinOBR :
sinAOC\0
sinBOC\0.3/
Nhân vế theo vế hệ thức (1), (2) (3), ta có: sinAR0B:sinBR0C:sinCR0A
sinAR0C:sinBR0A:sinCR0B D
sinOBR :sinOCR :sinOAR :sinBOA\0sinCOB\0:sinAOC\0
sinOCR :sinOAR :sinOBR :sinCOA\0sinAOB\0:sinBOC\0:
Nói cách khác: sin A0B
R sinAR0C:
sinBR0C sinBR0A:
sinCR0A
sinCR0B D 1.4/ (bởi gócS BOA
0; S AOB0IS
COB0; S BOC0IS AOC0; S COA0hoặc đôi đôi bù nhau) Hệ thức ta chứng minh trường hợp giá trị tuyệt đối Bây ta cần xét dấu
Trong trường hợp hình vẽ bên trái, tỉ số vế trái (4) âm, nên tích chúng lại âm
Trong trường hợp hai hình vẽ cịn lại, hai tỉ số ba tỉ số vế trái (4) dương tỉ số lại âm nên tích chúng lại âm
Cuối ta viết: sin A0B
R sinAR0C
:sin B0C
R sinBR0A
:sin C0A
R sinCR0B D
1:
Điều kiện đủ:
Giả sử ta có hệ thức: sin A0B
R sinAR0C
:sin B0C
R sinBR0A
:sin C0A
R sinCR0B
D 1.5/
GọiO giao điểm đường thẳngS AA0vàS BB0 Gọi giao điểm củaS COvới đường thẳngS AB làC00
Áp dụng điều kiện cần ta có: sin A0B
R sinAR0C
:sin B0C
R sinBR0A
:sin C00A
R sinCR00B
D 1.6/
Từ (5) (6) ta có: sin C00A
R sinCR00B D
sinCR0A sinCR0B.7/ Từ hệ thức (7) ta có điểmC0; C00trùng
ThayR Ri ta chứng minh cho định lí Céva-L Vậy ta chứng minh định lí Céva-Lbằng cách sử dụng chứng minh định lí Céva-S
3 Dùng hình học Euclid chứng minh hình học Lobachevsky
3.1 Phương pháp
(47)Tạp chí Epsilon, Số 05, 10/2015
3.2 Ví dụ minh họa
Bài tốn (Định lí Pascal Lobachevsky).
Cho điểm A; B; C; D; E; F nằm đường tròn L O/:Giả sửL AB \L DE D XIL BC \L EF DY; L CD\L FADZ:Chứng minh rằngX; Y; Z thẳng hàng.
Lời giải.Để chứng minh định lí Pascal-L, ta sử dụng mơ hình Poincaré để đưa toán Euclid Bây giờ, ta chứng minh toán sau
Bài toán Cho điểm A; B; C; D; E; F nằm đường tròn O/: l là đường thẳng bất kì khơng qua tâm O:.OAB/; ODE/; OBC/; OEF/; OCD/; OFA/ là đường tròn có tâm thuộc l và qua A; BID; EIB; CIE; FIC; DIF; A: Giả sử OAB/ \ ODE/ D X; X0I OBC/\.OEF/DY; Y0I OCD/\.OFA/DZ; Z0:Chứng minh rằngX; X0IY; Y0IZ; Z0
cùng thuộc đường trịn có tâm thuộcl:
Chứng minh Ngô Quang Dương.GọiU; V; W giao điểm củaABvàDE; BC vàEF; CD vàFA:
Dễ thấyXX0là trục đẳng phương của.OAB/và.ODE/
Phương tích củaU tới.OAB/và.ODE/lần lượt làUA:UB vàUD:UE:
DoA; B; D; E đồng viên nênUA:UB D UD:UE suy raX; U; X0thẳng hàng vàUX:UX0 D UA:UB DUD:UE:
Điều dẫn tới phương tích củaU tới.XX0Y Y0/và.O/bằng Tương tự, phương tích củaV tới.XX0Y Y0/; O/bằng nênU V trục đẳng phương của.O/và.XX0Y Y0/: Hoàn toàn tương tựU V W trục đẳng phương của.Y Y0ZZ0/,.ZZ0XX0/với.O/:
(48)Nhận xét
1 Để cho thuận tiện, từ trở sau ta gọi đường trịn.XX0Y Y0ZZ0/là ”đường trịn Pascal” điểm có thứ tựA; B; C; D; E; F:
2 Khi đường tròn.O/nằm trênlvà ta lấy nửa đường tròn.OAB/; ODE/; OBC/, OEF/; OCD/; OFA/; OXX0Y Y0ZZ0/và tương giao chúng (hình vẽ)
thế toán trở thành toán Vậy ta chứng minh tốn Pascal hình học Lobachevsky
Bài tốn (Định lí Steiner-L).
Cho điểmA; B; C; D; E; F thuộc đường trònL O/:Chứng minh đường thẳng
Pascal-L ABCDEF; EDAFBC; CEFBAD đồng quy.
Để chứng minh định lí Steiner-L, ta sử dụng mơ hình Poincaré để đưa tốn Euclid Bây giờ, ta chứng minh toán sau hình học Euclid
Bài tốn Trong mặt phẳng cho điểmA; B; C; D; E; F thuộc đường trịn.O/vàllà đường thẳng khơng qua tâm Chứng minh ”đường trịn Pascal” điểm có thứ tự
ABCDEF; EDAFBC; CEFBADđồng trục.
(49)Tạp chí Epsilon, Số 05, 10/2015
Theo chứng minh toán 4, đường thẳng Pascal củaABCDEF trục đẳng phương (O/với đường tròn Pascal củaABCDEF
Đường thẳng Pascal EDAFBC trục đẳng phương (O/với đường tròn Pascal EDAFBC
Đường thẳng Pascal củaCEFBAD trục đẳng phương (O/với đường tròn Pascal CEFBAD
Theo định lý Steiner, đường thẳng Pascal củaABCDEF; EDAFBC; CEFBADđồng quy Ta gọi điểm đồng quy làS Vậy nênS có phương tích với ”đường trịn Pascal” ”đường trịn Pascal” có tâm thuộcl nên lấymlà đường thẳng quaS vng góc vớil thìmlà trục đẳng phương ”đường trịn Pascal” Điều có nghĩa ”đường tròn Pascal” đồng trục Nhận xét.Khi đường tròn.O/nằm phía đường thẳnglvà ta lấy nửa tất đường tròn (trừ đường tròn.O//như.OAB/; ODE/; OBC/; OEF/; OCD/; OFA/; :::và tương giao nửa đường trịn tốn trở thành tốn Vậy ta chứng minh toán Steiner hình học Lobachevsky
4 Mở rộng tốn từ hình học Euclid thành hình học cầu hình học Lobachevsky
4.1 Phương pháp
(50)4.2 Ví dụ minh họa
Bài tốn (Bài T4/285 – Tạp chí tốn học tuổi trẻ).
Cho tam giác ABC với điểm M nằm tam giác Các tia AM; BM; CM cắt cạnh
BC; CA; AB tương ứng tạiD; E; F:GọiK là giao điểm củaDE vàCM, gọiH là giao điểm củaDF vàBM Chứng minh đường thẳngAD; BK; CH đồng quy.
Ta chứng minh toán sau
Áp dụng định lí Ménélaus cho tam giácAM C (với ba điểm thẳng hàngE; K; D/và tam giác AMB (với ba điểm thẳng hàngF; H; D/ta có
KM KC :
EC EA:
DA DM D1;
HB HM:
DM DA:
FA FB D1:
Suy ra: KM KC D
EA EC:
DM DA;
HB HM D
FB FA:
DA DM.1/
Áp dụng định lí Céva cho tam giácABC với ba đường thẳng đồng quyAD; BE; CF: DC
DB: FB FA:
EA
EC D 1:
Từ đó: DC DB D
FA FB:
EC EA.2/ Từ (1) (2) ta có
KM KC :
HB HM:
DC
DB D 1:
Vậy theo phần đảo định lí Céva,BK; CH; MD đồng quy HayAD; BK; CH đồng quy Bài toán (Mở rộng tốn hình học cầu).
Cho tam giácS ABC với điểmM nằm tam giác Các tiaS AM; S BM; S CM
cắt cạnhS BC; S CA; S AB tương ứng tạiD; E; F:GọiK là giao điểm củaS DE
vàS CM, gọiH là giao điểm củaS DF vàS BM Chứng minh đường thẳng
(51)Tạp chí Epsilon, Số 05, 10/2015
Áp dụng định lí Ménélaus cho tam giácS AM C (với ba điểm thẳng hàngE; K; D/và tam giácS AMB (với ba điểm thẳng hàngF; H; D/ta có
sinKMR sinKCR :
sinE CR sinEAR :
sinDAR sinDMR D1;
sinHBR sinHMR :
sinDMR sinDAR :
sinFAR sinFBR D
1:
Suy ra: sin KM
R sinKCR D
sinEAR sinE CR :
sinDMR sinDAR ;
sinHBR sinHMR D
sinFBR sinFAR
:sin DA
R sinDMR 1/
Áp dụng định lí Céva cho tam giácS ABC với ba đường thẳng đồng quyS AD; S BE; S CF: sin
DC R sinDBR :
sinFBR sinFAR :
sinEAR sinE CR D
1:
Từ đó: sin DC
R sinDBR D
sinFAR sinFBR
:sin E C
R sinEAR
.2/
Từ (1) (2) ta có: sin KM
R sinKCR
:sin HB
R sinHMR
:sin DC
R sinDBR D
1:
Vậy theo phần đảo định lí Céva,S BK; S CH; S MD đồng quy HayS AD; S BK; S CH đồng quy
Bài toán (Mở rộng tốn hình học Lobachevsky)
Cho tam giácL ABC với điểmM nằm tam giác Các tiaL AM; L BM; L CM
cắt cạnhL BC; L CA; L ABtương ứng tạiD; E; F:GọiK là giao điểm củaL DE
vàL CM, gọiH là giao điểm củaL DF vàL BM Chứng minh đường thẳng
(52)Áp dụng định lí Ménélaus cho tam giácL AM C (với ba điểm thẳng hàngE; K; D/và tam giácL AMB (với ba điểm thẳng hàngF; H; D/ta có
sinhKMR sinhKCR
:sinh E C
R sinhEAR
:sinh DA
R sinhDMR D
1; sinh HB
R sinhHMR
:sinh DM
R sinhDAR
:sinh FA
R sinhFBR D
1:
Suy ra:
sinhKMR sinhKCR D
sinhEAR sinhE CR :
sinhDMR sinhDAR ;
sinhHBR sinhHMR D
sinhFBR sinhFAR
:sinh DA
R sinhDMR 1/
Áp dụng định lí Céva cho tam giácL ABC với ba đường thẳng đồng quyL AD; L BE; L CF:
sinhDCR sinhDBR
:sinh FB
R sinhFAR
:sinh EA
R sinhE CR D
1:
Từ đó:
sinhDCR sinhDBR D
sinhFAR sinhFBR
:sinh E C
R sinhEAR
.2/
Từ (1) (2) ta có: sinh KM
R sinhKCR
:sinh HB
R sinhHMR
:sinh DC
R
sinhDBR D 1:
Vậy theo phần đảo định lí Céva,L BK; L CH; L MDđồng quy HayL AD; L BK; L CH đồng quy
5 Kết luận
Chúng ta vừa có số khám phá mở rộng thú vị từ hình học Euclid sang hình học cầu hình học Lobachevsky Phương pháp mở rộng phương pháp phát tốn Chính thế, phương pháp quan trọng phát triển tư Bài viết cần trao đổi thêm? Mong chia sẻ bạn
Tài liệu tham khảo
[1] N.V.Efimov (1980),Higher geometry, Mir Publishers Moscow
[2] P Constan (1941),Cours de Trigonométrie Sphérique, Paris Société D’Éditions [3] Tạp chí Tốn học tuổi trẻ
(53)VỀ CHỨNG MINH VÀ TIẾN BỘ TRONG TOÁN HỌC (tiếp theo)
William P Thurston - Nguyễn Dzuy Khánh dịch
5 Điều khích lệ người nghiên cứu tốn học?
Có niềm vui thực làm toán, việc học cách tư diễn giải, tổ chức đơn giản hóa Bạn cảm thấy niềm vui khám phá toán học mới, tái khám phá toán học cổ xưa, học cách tư từ hay từ tài liệu, tìm cách giải thích mới, thú vị để nhìn nhận cấu trúc tốn học cũ
Động lực nội hướng nghĩ làm tốn hồn tồn lợi ích Điều khơng xác: mơi trường xã hội quan trọng Chúng ta truyền cảm hứng người khác, đánh giá cao người khác thích giúp đỡ người khác giải vấn đề tốn học họ Những điều ưa thích thay đổi tương ứng với người khác Tương tác xã hội xảy qua buổi gặp mặt trực tiếp Nó xảy qua thư thư điện tử điện tử, tiền ấn phẩm báo tạp chí Một hiệu ứng hệ thống tốn học mang tính xã hội cao khuynh hướng nhà toán học theo vấn đề hợp mốt Vì mục đích tạo định lý tốn học có lẽ khơng hiệu quả: tốt hợn có nhà toán học phủ lĩnh vực tri thức đồng Nhưng hầu hết nhà tốn học khơng thích đơn độc, họ gặp vấn đề với việc phấn khích với chủ đề, họ đạt tiến triển cá nhân, họ có đồng nghiệp để chia sẻ phấn khích họ
Ngồi động lực nội động lực mang tính xã hội khơng thức cho việc làm toán, dẫn dắt lý kinh tế địa vị Các nhà toán học, giống nhiều nhà khoa học khác, thực nhiều đánh giá bị đánh giá nhiều Bắt đầu với mức điểm, tiếp tục qua thư tiến cử, định tuyển, định đề bạt, báo cáo thẩm định, lời mời báo cáo, giải thưởng, tham gia vào nhiều đánh giá, hệ thống cạnh tranh khốc liệt
Jaffe Quinn phân tích động lực để làm toán qua hệ thống tiền tệ chung mà nhiều nhà toán học tin tưởng: trả công cho định lý
(54)Thậm chí có quan điểm hẹp hịi mà tạo định lý, tập thể quan trọng Bóng đá coi ẩn dụ Có thể có hai bàn thắng thơi suốt trận đấu, ghi hai cầu thủ Nhưng điều khơng có nghĩa nỗ lực tất thủ khác vô giá trị Chúng ta không đánh giá cầu thủ đội bóng việc họ có tự ghi bàn hay khơng; đánh giá đội bóng qua cách vận hành đội bóng
Trong tốn học, thường xảy chuyện nhóm nhà toán học đạt tiến với tập hợp ý tưởng định Có định lý đường tiến mà chắn chứng minh người này, hay người khác Đơi nhóm nhà tốn học chí tiên đốn định lý tựa Việc khó nhiều so với việc dự đoán xem thực chứng minh định lý, thường có số "người bật" mà có khả cao ghi điểm Tuy nhiên, họ vị trí mà chứng minh định lý nhờ tập hợp nỗ lực toàn đội Toàn đội có chức nữa, việc tiếp thu sử dụng định lý chúng chứng minh Thậm chí người chứng minh tất định lý đường mà khơng cần trợ giúp, chúng bị bỏ phí khơng cịn khác học chúng
Có tượng thú vị liên quan tới người "nổi bật" Thường xảy mức trung bình chứng minh định lý mà chấp nhận rộng rãi kết giá trị Vị họ cộng đồng - thứ bậc họ - tăng đột ngột Khi điều xảy ra, họ nhanh chóng đạt hiệu suất cao nhiều trung tâm ý tưởng nguồn cho định lý Tại sao? Trước tiên, có tăng tiến lớn niềm tự tôn thân, tăng tiến hệ hiệu suất công việc Thứ hai, vị họ lớn mạnh hơn, người gần trung tâm mạng lưới ý tưởng-những người khác nhìn nhận họ nghiêm túc Điểm cuối có lẽ quan trọng nhất, đột phá toán học thường thể cách tư mới, phương cách hiệu tư thường áp dụng vào nhiều tình
Hiện tượng thuyết phục cộng đồng toán học trở nên dần đạt hiệu suất cao nhiều mở rộng tâm hồn vào giá trị thực công việc mà thực Jaffe Quinn đề xuất hệ thống nhận biết vai trò chia nhỏ thành "ức đoán" "chứng minh" Một phép phân chia trì niềm tin hoang đường tiến đo đơn vị cho định lý chuẩn tìm Nghe giống ngụy biện người mà in bảng 10000 số nguyên tố Thứ mà làm hiểu biết người Chúng ta có nhiều cách khác để hiểu nhiều trình khác mà góp phần xây dựng nên hiểu biết Ta thỏa mãn hơn, có hiệu suất cao hơn, hạnh phúc ghi nhận tập trung vào việc
6 Một số kinh nghiệm cá nhân
Bởi viết nảy sinh từ phản chiếu khơng tương thích kinh nghiệm tơi với mô tả Jaffe Quinn, bàn hai kinh nghiệm cá nhân, bao gồm kinh nghiệm mà họ ám tới
(55)Tạp chí Epsilon, Số 05, 10/2015
biết thân tơi tiến trình tốn học, có nhiều điều mà tơi hi vọng làm khác Tơi hi vọng qua việc kể kinh nghiệm cách thẳng thắn tơi cịn nhớ hiểu nó, tơi giúp người khác hiểu tốt tiến trình tốn học học trước từ
Đầu tiên bàn luận ngắn gọn lý thuyết phân lá, chủ đề mà tơi bắt đầu cịn học viên sau đại học (ở khơng quan trọng bạn có biết lý thuyết phân hay khơng) Ở thời điểm đó, lý thuyết phân trở thành trung tâm ý nhà tơpơ hình học, người nghiên cứu hệ động lực nhà hình học vi phân Khá nhanh chóng, tơi chứng minh số định lý theo cách ấn tượng Tôi chứng minh định lý phân loại cho phân lá, đưa điều kiện cần đủ cho đa tạp có phân Tôi chứng minh số định lý đáng ý khác Tôi viết số báo đáng kính nể cơng bố định lý quan trọng Thật khó để có đủ thời gian để viết để trì tơi chứng minh, xây dựng phần lưu trữ
Một tượng thú vị xảy Trong vòng vài năm tháo lui đột ngột ngành bắt đầu diễn Tôi nghe từ nhiều nhà toán học họ đưa hay nhận lời khuyên đừng vào lý thuyết phân lá– họ nói Thurston giải lý thuyết Người ta nói với tơi (khơng phải phàn nàn, mà lời khen) giết chết lĩnh vực Những học viên sau đại học ngừng nghiên cứu lý thuyết phân lá, nhanh chóng, tơi hướng ưa thích sang lĩnh vực khác
Tôi không tin tháo lui xảy lĩnh vực cạn kiệt tiềm tri thức Đã có (và cịn) nhiều câu hỏi thú vị có lẽ tiếp cận Kể từ năm tháng ấy, có phát triển thú vị số nhà tốn học khác đưa ra, người cịn làm việc ngành hay tham gia, có tiến triển quan trọng địa hạt lân cận mà nghĩ chúng tăng tốc cực nhanh nhà toán học tiếp tục theo đuổi lý thuyết phân cách mạnh mẽ Ngày nay, tơi nghĩ có nhà tốn học, người hiểu thứ tiếp cận trạng thái nghệ thuật phân tồn thời điểm đó, có vài phần lý thuyết phân lá, bao gồm phát triển kể từ thời ấy, phát triển mạnh
Tôi tin hai hiệu ứng mang tính sinh thái học quan trọng nhiều việc làm ngã lòng người chủ đề so với việc cạn kiệt nguồn tri thức xảy
(56)Thứ hai vấn đề có ngành dành cho người khác Khi bắt đầu làm việc với lý thuyết phân lá, nghĩ điều mà họ theo đuổi tập hợp định lý mạnh chứng minh mà áp dụng để trả lời vấn đề toán học lớn Nhưng phần câu chuyện mà Hơn tri thức, người muốn có thấu hiểu mang tính cá nhân Và hệ đánh giá đưa chúng ta, họ muốn cần ghi nhận qua định lý
Tôi bỏ qua vài năm để đến với chủ đề mà Jaffe Quinn ám tới, bắt đầu nghiên cứu đa tạp ba chiều quan hệ chúng với hình học hyperbolic (Một lần nữa, khơng có vấn đề bạn khơng biết gì.) Tơi xây dựng vài năm trực quan định với 3-đa tạp hyperbolic, với tập hợp cách xây dựng, ví dụ phép chứng minh (quá trình thực bắt đầu tơi cịn sinh viên, ủng hộ mạnh áp dụng vào lý thuyết phân lá) Sau thời gian, đặt giả thuyết hay ước đoán tất 3-đa tạp có cấu trúc hình học định, giả thuyết cuối biết đến tên gọi giả thuyết hình học hóa Khoảng hai hay ba năm sau đó, tơi chứng minh giả thuyết hình học hóa cho đa tạp Haken Nó định lý khó, tơi dành nỗ lực khổng lồ để nghĩ Khi tơi hồn thiện phép chứng minh, dành nhiều công sức để kiểm tra phép chứng minh, tìm kiếm điểm khó khăn kiểm chứng lần trước thông tin độc lập
Tôi muốn viết chi tiết ý tưởng tơi nói tơi chứng minh định lý Nó có nghĩa tơi có dịng ý tưởng sáng sủa hoàn thiện, bao gồm chi tiết mà đứng vững trước nhiều lần kiểm tra người khác Các nhà tốn học có phong cách tư khác Phong cách đưa tổng qt hóa rộng bất cẩn mang tính định hướng tạo cảm hứng: thiết lập mô hình tư cụ thể, tơi tư điều qua Vì chứng minh tơi đáng tin cậy Tôi chưa gặp vấn đề với việc lưu lại mệnh đề hay đưa chi tiết thứ mà chứng minh Tôi làm tốt việc phát lỗi sai suy luận người khác
Tuy nhiên, đơi có yếu tố bị phát triển mạnh qua việc phiên dịch từ mã hóa tư riêng tơi tới mà truyền tải sang khác Nền tảng giáo dục tốn học độc lập theo phong cách riêng, mà nhiều năm rịng tơi tự học nhiều thứ, tự phát triển hình mẫu tư duyriêng cho việc nên nghĩ toán học Việc thường lợi lớn cho tơi việc tư tốn học, sau dễ dàng tiếp nhận hình mẫu tư chuẩn chia sẻ nhóm nhà tốn học khác Điều có nghĩa số khái niệm mà sử dụng cách tự tự nhiên tư lại xa lạ với hầu hết nhà toán học khác mà tơi nói chuyện Những hình mẫu cấu trúc tư duycủa cá nhân tương tự mặt đặc tính với kiểu mẫu hình mà nhóm nhà tốn học khác chia sẻ - chúng thường hình mẫu khác Ở thời điểm mà tơi hệ thống hóa giả thuyết hình học hóa, hiểu biết tơi hình học hyperbolic ví dụ tốt Một ví dụ ngẫu nhiên hiểu biết không gian tô pô hữu hạn, chủ đề kỳ quặc mà cho mượn ý tưởng tốt vào mớ câu hỏi khơng hồn tồn đáng để phát triển trường hợp có lối diễn đạt loanh quanh luẩn quẩn ngăn trở
(57)Tạp chí Epsilon, Số 05, 10/2015
các xu hướng tôpô 30 năm trước đó, làm người ngạc nhiên Với hầu hết nhà tơpơ vào thời điểm đó, hình học hyperbolic ngành cần phải biết tốn học, có nhóm nhà tốn học khác nhà hình học vi phân, họ khơng hiểu số góc độ định Các nhà tôpô cần tốn chút thời gian để hiểu giả thuyết hình học hóa gì, có tốt, lại xác đáng
Cùng thời điểm đó, tơi bắt đầu viết ghi chép hình học tơpơ 3-đa tạp, dùng cho khóa học sau đại học mà tơi dạy Tơi phát cho số người, lâu trước người khác giới bắt đầu chép lại Danh sách thư điện tử lớn lên tới khoảng 1200 người, người mà gửi ghi chép vài tháng lần Tôi cố gắng trao đổi ý tưởng thực ghi chép Mọi người thực nhiều seminar dựa ghi chép nhận nhiều phản hồi Tràn ngập phản hồi câu “Những ghi chép ông thật đầy cảm hứng đẹp đẽ, tơi phải nói với ơng phải dành tới ba tuần seminar để hiểu chi tiết trongn.n Chắc chắn cần thêm giải thích.”
Tơi dành nhiều lời giới thiệu tới nhóm nhà tốn học ý tưởng nghiên cứu 3-đa tạp từ quan điểm hình học chứng minh giả thuyết hình học hóa cho đa tạp Haken Ban đầu, chủ đề xa lạ với hầu hết người Thật khó để trao đổi sở nằm đầu tơi, khơng phải cộng đồng tốn học Có vài lý thuyết tốn học mà có ảnh hưởng lên đám ý tưởng này: tôpô ba-đa tạp, nhóm Klein, hệ động lực, tơpơ hình học, nhóm rời rạc nhóm Lie ri rc, lý thuyt phõn lỏ, cỏc khụng gian Teichmăuller, đồng phơi giả Anosov, lý thuyết nhóm hình học, hình học hyperbolic Chúng tơi tổ chức workshop Hiệp hội Toán học Mỹ Bowdoin vào năm 1980, nơi nhiều nhà toán học ngành hẹp tôpô số chiều thấp, hệ động lực nhóm Klein tham dự
Đó kinh nghiệm thú vị việc trao đổi văn hóa Câu chuyện đột ngột trở nên rõ ràng phép chứng minh phụ thuộc vào thính giả Chúng ta chứng minh vài thứ bối cảnh xã hội nhắm tới số thính giả định Một vài phần phép chứng minh tơi trao đổi vịng hai phút với nhà tơpơ nhà giải tích cần tới giảng dài để họ bắt đầu hiểu Tương tự vậy, có vài thứ mà nói vịng hai phút cho nhà giải tích mà tốn trước nhà tôpô bắt đầu nhận thức Và có nhiều phần khác phép chứng minh cần hai phút để diễn đạt phần tóm tắt, khơng số thính giả thời điểm có đủ sở trí tuệ để hiểu
Ở thời điểm đó, thực tế khơng có sở ngữ cảnh cho định lý này, phát triển từ việc làm mà ý tưởng khởi nguồn từ tâm trí tơi tới tơi phải nói để khiến trở nên hiểu được, khơng đề cập tới chuyện thính giả phải hi sinh để hiểu nó, ấn tượng
(58)Tương tự vậy, tới sau này, vài người thực vượt qua phần khó sâu sắc ghi chép
Kết nhiều nhà toán học có hiểu biết mà họ thiếu bắt đầu: hiểu biết có hiệu lực khái niệm sở mà tự nhiên với ngành Đã có cịn tiếp tục có nhiều hoạt động tốn học lớn mạnh Bằng cách tập trung vào xây dựng sở, giải thích cơng bố định nghĩa cách thức tư chậm rãi việc phát biểu công bố phép chứng minh tất “định lý” mà biết làm để chứng minh, dành hội cho người để nhận lấy danh tiếng Vẫn hội cho người để khám phá công bố chứng minh khác định lý hình học hóa Những chứng minh giúp phát triển khái niệm toán học mà tự thân thú vị thúc đẩy toán học xa Điều mà nhà toán học muốn cần từ tơi học cách tư thực tế học cách chứng minh giả thuyết hình học hóa cho đa tạp Haken Không chắn chứng minh giả thuyết hình học hóa tổng qt bao hàm việc đẩy xa Một vấn đề người ta cần hay muốn kết chấp nhận xác khơng để học nó, mà cịn từ họ trích dẫn lại hay dựa
Các nhà tốn học thực nhanh chóng chấp nhận phép chứng minh tơi, để bắt đầu trích dẫn hay sử dụng dựa tài liệu sẵn có, kinh nghiệm họ niềm tin chấp thuận ý kiến riêng chuyên gia, người mà sử dụng nhiều thời gian để thảo luận phép chứng minh Định lý ghi lại, qua nguồn công bố, với số người khác tác giả, nên hầu hết người cảm thấy an tồn trích dẫn lại nó; người ngành chắn khơng địi hỏi tơi tính xác nó, hay biểu lộ cho thấy cần thiết chi tiết mà khơng có sẵn
Khơng phải tất phép chứng minh có vai trị giàn ý logic mà xây dựng cho toán học Phép chứng minh đặc thù có lẽ có giá trị logic tạm thời, có giá trị động lực lớn lao việc giúp đỡ ủng hộ nhìn định vào cấu trúc 3-đa tạp Giả thuyết hình học hóa tổng quát giả thuyết Nó chứng minh nhiều trường hợp, ủng hộ nhiều chứng máy tính, chưa chứng minh trường hợp tổng quát Tôi bị thuyết phục phép chứng minh tổng quát khám phá ra; hi vọng từ rất nhiều năm trước Ở điểm này, phép chứng minh trường hợp đặc biệt gần trở nên lỗi thời Trong đó, người muốn hiểu kỹ thuật hình học tốt nên bắt đầu với giả định: “Cho M3 đa tạp mà có nhận phân rã hình học” tổng quát “Cho M3 đa tạp Haken” Những người không muốn sử dụng kỹ thuật hay nghi ngờ lờ Thậm chí định lý đa tạp Haken chứng minh kỹ thuật hình học, việc tìm kiếm kỹ thuật tơpơ để chứng minh có giá trị cao
Trong tình (mà cịn tiếp tục) tơi nghĩ phải cố gắng để loại bỏ hai kết xấu cho mình: khơng lộ tơi khám phá, giữ cho riêng (có lẽ với niềm hi vọng chứng minh giả thuyết Poincare) để giới thiệu lý thuyết kín kẽ khó học mà có người để giữ tồn phát triển
(59)Tạp chí Epsilon, Số 05, 10/2015
Haken mà không phát biểu thật tốt trước cộng đồng toán học Khi tập trung vào phát triển sở thay định lý cấp cao lý thuyết hình học 3-đa tạp, trở nên xa rời chủ đề tiếp tục phát triển không thúc đẩy cách tích cực hiệu ngành này, hay nghiệp người tham gia nghiên cứu (Nhưng với tơi, vài mức độ xa rời gần sản phẩm phụ tránh việc hướng dẫn học viên sau đại học số khác: để thực chuyển hướng nghiên cứu đích thực sang lĩnh vực khác, cần phải gạt bỏ chặn lại nghĩ nặng nề chuyện ấy)
Mặt khác, trở nên bận bịu có hiệu suất cao hơn, nhiều hoạt động khác Hệ thống không tạo thêm thời gian cho người sử dụng để viết nghiên cứu; thay vào làm tràn ngập nhiều hội cho việc làm thêm phản ứng tơi nói “có” với tất yêu cầu hội Tôi dành nhiều nỗ lực vào hoạt động không tạo danh tiếng mà coi trọng coi trọng việc chứng minh định lý: trị tốn học, chỉnh sửa ghi chép thành sách với tiêu chuẩn cao trao đổi, khám phá tính tốn toán học, giáo dục toán học, phát triển hình thức trao đổi tốn học qua Trung tâm Hình học (giống thí nghiệm chúng tôi, video “Not Knot”, đạo MSRI, v.v
Tơi nghĩ tơi làm khơng thể cực đại hóa “danh tiếng” tơi Tơi vị trí mà khơng có nhu cầu lớn phải cạnh tranh để giành nhiều công nhận Thực bắt đầu thấy cảm thấy bị thách thức mạnh mẽ từ thứ khác, bên cạnh việc chứng minh định lý
(60)(61)LUẬT BENFORD
VÀ NHỮNG ỨNG DỤNG THÚ VỊ
Trần Nam Dũng - Đặng Nguyễn Đức Tiến
(Đại học Khoa học Tự nhiên, ĐHQG-TP.HCM - Đại học Trento, Italia)
LỜI GIỚI THIỆU
Luật Benford (Benford Law) hay gọi luật chữ số thứ (First Ditgit Law) luật tiếng toán học giới thiệu nhiều viết diễn đàn số giáo trình tốn học bậc đại học Trong viết Epsilon, muốn giới thiệu với độc giả cách tiếp cận với định luật kỳ lạ thông qua giảng toán học nhà toán học tiếng Vladimir Arnold mà người viết may mắn có dịp nghe trực tiếp Bên cạnh đó, chúng tơi nhân viết giới thiệu với độc giả số ứng dụng thú vị định luật tưởng chừng "vô bổ" này!
1 Câu chuyện nhà toán học Vladimir Arnold
Câu chuyện tôi, Trần Nam Dũng, nghe trực tiếp từVladimir Arnold, nhà toán học tiếng người Nga ơng nói chuyện với học sinh chun tốn Mát-xcơ-va (Moscow, thủ nước Nga nay) Câu chuyện sâu sắc đòi hỏi kiến thức toán học định Vladimir Arnold bắt đầu buổi nói chuyện câu hỏi:“Em cho biết,2100bắt đầu bằng chữ số nào?”
Ái chà, câu lạ Nếu tìm chữ số tận 2100thì dễ, học sinh lớp biết Chữ số tận của2nkhinD1; 2; 3; 4; 5; : : : là2; 4; 8; 6; 2; 4; 8; 6; : : : có nghĩa chúng tuần hoàn với chu kỳ ta có ngay2100 tận
Ta thử tìm xem chữ số của2n.nD0; 1; 2; : : : /có quy luật tuần hồn khơng: 1, 2, 4, 8, 1, 3, 6, 1, 2, 5,1, 2, 4, 8, 1, 3, 6, 1, 2, 5, 1, 2, 4, 8, 1, 3, 6, 1, 2, 5,1, 2, 4, 8, 1, 3, 6, 1, 2, 5
Dường có quy luật, cụ thể dãy số 1, 2, 4, 8, 1, 3, 6, 1, 2, (độ dài 10) lặp lại Như đáp số 1!
Một học sinh giơ tay:“Dạ thưa giáo sư, đáp số ạ!”
“Đúng! Giỏi lắm! Em giải thích sao?”
“Dạ thưa, em quan sát thấy dãy chữ số của2nlà tuần hoàn với chu kỳ 10, từ em tính được2100có chữ số giống20,210,220và ạ.”
(62)“Dạ em chưa chứng minh được, em nghĩ đúng, em thử đến tậnnD40rồi ạ!” “Đúng có nhiều nguyên nhân để em tin dự đốn Em thử đến 40 và thấy quy luật lặp lặp lại Hơn lại báo cho em biết em nói đáp số Tuy nhiên, trong toán học, dự đoán chưa chứng minh dự đốn, cho dù nó được thử triệu hay tỉ Có mệnh đề sai bước thứ triệu lẻ một!” “Nhưng thưa giáo sư, trường hợp phát biểu bạn Kolia hay sai ạ?”Một học sinh nơn nóng hỏi
“Các em thử tính tiếp xem sao!”
“Ơi, sai ạ! Ở hàng chục thứ 5, từ240 đến249, chữ số 1, 2, 4, 8, 1, 3,7, 1, 2, 5.”Xuất chữ số ngoại lai nằm ngồi quy luật
Tiếp tục tính chục tiếp theo, ta được: 50-59: 1, 2, 4,9, 1, 3,7, 1, 2, 5
60-69: 1, 2, 4,9, 1, 3,7, 1, 2, 5 70-79: 1, 2, 4,9, 1, 3,7, 1,3, 6 80-89: 1, 2, 4,9, 1, 3,7, 1,3, 6 90-100: 1, 2, 4,9, 1, 3,7, 1,3, 6, (1)
Như số trệch theo quy luật dự đoán ban đầu ngày nhiều Tuy nhiên, dường chữ số khơng bị lệch quy luật
“Thưa giáo sư, dường chữ số lũy thừa2nvớin 0; 4; 7mod 10 luôn bằng 1”.
“Dự đoán dự đoán! Các em kiên nhẫn tính thêm 10 số nữa!”
Và 10 số là:
100-109: 1, 2, 5, 1, 2, 4, 8, 1, 3,
Như dường quy luật bị lệch pha khẳng định không Chú ý rằng, khác với trường hợp chữ số tận cùng, q trình tính tốn, ta khơng thể “cắt đuôi” hay bỏ đầu mà phải giữ lại tất Vì phải làm việc với số lớn lên đến hàng chục chữ số
“Thưa giáo sư, phải làm nào? Bởi tính tốn ngày phức tạp máy tính chúng em bó tay Chẳng hạn cần tìm chữ số của21990thì làm sao?”
(Lưu ý câu chuyện xảy vào năm 1990)“Liệu có phải 1?”Bạn học sinh hỏi thêm
“Nếu tìm chữ số của210i thì ln cho đếni D30thì sai.2300bắt đầu bằng chữ số 2”
“Thế còn21990?”
(63)Tạp chí Epsilon, Số 05, 10/2015
“Nhưng tìm ạ?”
“Được Rõ ràng ta tiếp tục câu chuyện mà dùng tính tốn túy Ta tính tay như đủ Bây lúc phải suy nghĩ Theo em, điều kiện để sốNcó chữ số đầu tiên làalà gì?”
“Dạ, N có k chữ số điều kiện là:a10k N < aC1/10k ạ!”
“Đúng rồi, tốt! Bây lấylghai vế, ta đượck 1ClgalgN < k 1Clg.aC1/”
“Điều có nghĩa gì? Có nghĩa N có chữ số đầu a ta có bất đẳng thức trên.” “Dạ thưa giáo sư, ta k ạ!”
“Các em thử nghĩ xem, số chữ số số N tính nào?” “Dạ,kD1CŒlgN ạ!”
“Đúng Như có phải N có chữ số a khilga flgNg< lg.aC1/
đúng khơng?”
“Ồ, vìlg.21990/D1990lg.2/D559:049nênflg.21990/g D0:049và ta suy chữ số đầu tiên của21990 là 1!”Một học sinh hồ hởi nói
“Và vậy, cần biếtlg.2/; lg.3/; : : : ; lg.10/là ta tìm chữ số của2nvớin
bất kỳ Vậy xong rồi!”
“Nhưng câu chuyện bắt đầu!”Vladimir Arnold hóm hỉnh nói.“Bây chúng ta thử làm thống kê xem 100 lũy thừa đầu tiên, có lũy thừa bắt đầu chữ số 1, lũy thừa bắt đầu chữ số 2, , lũy thừa bắt đầu chữ số 9.”
Học sinh tiến hành thống kê bảng sau:
Chữ số
# 30 17 13 10 5
Như chữ số xuất nhiều hẳn, sau đến chữ số
“Điều giải thích nhờ vào điều kiện: 2n có chữ số a khifnlg.2/g Œlg.a/; lg.aC1//”
“Và theo định lý Weil phân bố đều, xác suất để điều xảy độ dài của khoảngŒlg.a/; lg.aC1//, tức bằnglgaC1
a .”
“Định lý Weil phân bố định lý ạ?”
“Định lý khẳng định nếu˛là số vô tỷ dãyfn˛gsẽ phân bố đoạnŒ0; 1, điều có nghĩa với khoảng.a; b/thuộcŒ0; 1, xác suất đểfn˛gthuộc.a; b/sẽ bằng
(64)“Như thế, dolg2 > lg
3
2 > > lg 10
9 nên việc chữ số xuất nhiều hợp lý!” Học sinh hiểu phấn khích với điều giáo sư nói
“Câu chuyện tốn học đến kết thúc Nhưng áp dụng quan sát này vào lịch sử địa lý chút Các em nhà lấy atlas ra, tìm số liệu diện tích và dân số nước, sau thống kê xem số diện tích dân số đó, có bao nhiêu số bắt đầu chữ số 1, số bắt đầu chữ số 2, , số bắt đầu bằng chữ số Hãy đưa nhận xét cố gắng giải thích nhận xét góc độ tốn học và lịch sử! Xin cảm ơn em tham gia buổi nói chuyện hơm cách nhiệt tình”.
2 Đơi dịng lịch sử luật Benford
Vladimir Arnold giới thiệu với học sinh ông giảng tuyệt đẹp chất luật Benford, luật lại có tên gọi vậy, đời nào? Trong phần này, chúng tơi giới thiệu đơi dịng lịch sử định luật đáng kinh ngạc
Nhà toán học – thiên văn học người Mỹ - CanadaSimon Newcomb(1835 – 1909) ghi nhận người để ý kiện Chuyện kể rằng, Simon ngạc nhiên thấy tra cứu logarithm trang đầu chứa số bắt đầu nhiều hơn, cịn trang sau số lại có chữ số đầu lớn Simon đặt giả thiết phải người ta gặp số có chữ số đầu chữ số nhỏ nhiều chữ số lớn? Từ giả thiết đó, ơng đề cập đến tượng báo "Ghi chép tần suất sử dụng chữ số khác số tự nhiên" tính xác suất gặp chữ số đầu 1, 2, 3, giảm dần
Đến năm 1938,Frank Benford(1883 - 1948), kỹ sư điện tử vật lý học người Mỹ, nghiên cứu lại tượng sau đặt tên luật theo tên ơng Frank Benford thu thập số liệu thực tế từ diện tích bề mặt 335 sông, 104 số vật lý, 1800 trọng lượng phân tử, 5000 mục từ sổ tay toán học, nhiều nguồn khác Tổng cộng ông thu thập 20.229 số tiến hành thống kê số lần xuất chữ số Trong phân tích mình, ơng tìm có khoảng 30% số bắt đầu với 1, 18% với 2, Định luật lặp lại với tập hợp liệu khác, ví dụ kết trận bóng chày, tỉ lệ tử vong, giá cổ phiếu, địa nhà, hóa đơn tiền điện, Benford khơng thể giải thích lại
Tháng sáu năm 1961, nhà toán học người Mỹ Roger Pinkham lần đưa giải thích chứng minh cho định luật qua báo "On the Distribution of First Significant Digits" từ nhiều lý giải khác khai thác Một cách tiếp cận dễ hiểu cho trường hợp2n giới thiệu phần đầu viết thông qua giảng Vladimir Arnold
Kể từ luật Benford đời đến nay, có 18.000 cơng trình1hoặc trực tiếp gián tiếp liên quan đến định luật thú vị Và kết thúc phần lịch sử này, chúng tơi tóm tắt lại luật Benford công thức đơn giản sau:
1
(65)Tạp chí Epsilon, Số 05, 10/2015
Xác suất xuất chữ số đầu tiênd (d f1; 2; : : : ; 9g) là: P d /Dlg.1C
d/ Giá trị P(d) tính xấp xỉ là:
d
P(d) 30.1% 17.6% 12.5% 9.7% 7.9% 6.7% 5.8% 5.1% 4.6%
3 Những ứng dụng thú vị
Như với định luật Benford, ta biết tập hợp danh sách số liệu lấy từ nguồn thực tế tuân theo dạng định xác suất chữ số Nhưng liệu điều phải bất ngờ thú vị hay có ứng dụng vào sống? Câu trả lời hẳn độc giả dễ dàng đốn ra, luật Benford có nhiều ứng dụng quan trọng, viết này, giới thiệu hai ứng dụng luật Benford: ứng dụng vào kiểm tra số liệu kinh tế ứng dụng vào giám định ảnh số! Nếu ứng dụng ứng dụng kinh điển, nhiều người biết đến ứng dụng thứ hai lạ, tầm hiểu biết người viết lần giới thiệu với độc giả Việt Nam Hãy lấy chiều cao tòa nhà cao giới hay quốc gia đó, thống kê chữ số đầu tiên, lấy độ dài sông giới, thống kê chữ số đầu tiên, dù tính theo mét hay theo dặm, theo inch hay theo foot, tất kết gần với xác suất nêu luật Benford Và số báo cáo tài chính, báo cáo thuế vậy! Vì vậy, vào việc thống kê so sánh độ khác biệt so với luật Benford, người ta phát số liệu liệu có bị chỉnh sửa hay khơng!
Vì luật Benford trái với cảm nhận thông thường nhiều người (cho chữ số có xác suất xuất nhau) nên người làm giả số liệu có xu hướng đưa số có chữ số tuân theo phân bố đều, giả mạo phát so sánh với phân bố luật Benford Một kết nghiên cứu Jialan Wang (đương thời giáo sư đại học Washington, Mỹ), thông qua luật Benford cho thấy xu hướng làm giả số liệu tài tăng liên tục suốt 50 qua Nhiều nghiên cứu khác cho thấy luật Benford công cụ hữu hiệu cho phép phát giả mạo tài Vào năm 1998, người ta thử lấy số liệu báo cáo thuế tổng thống Mỹ Bill Clinton để thử với luật Benford, thú vị số liệu ông tuân theo luật Đến đây, bạn đọc thử lấy bảng số liệu tài thử kiểm chứng xem
Để đến ứng dụng thứ hai, mạn phép giới thiệu với độc giả số kiến thức "lạc tông": ảnh số!
(66)Bạn có biết?
Vào năm 2000, Kodak thống kê kỷ lục giới: số lượng ảnh giới đạt đến số 80 triệu ảnh
Chỉ 14 năm sau, ngày có khoảng 1.8 tỷ ảnh đưa lên internet! Con số cịn nhiều tồn ảnh lịch sử lồi người cộng lại tính đến 2004, năm đời Flickr
Dự kiến vào cuối năm 2015, số lượng ảnh đạt ngưỡng trillion, tức 1000 tỉ ảnh! Gấp 12.500 lần so với 15 năm trước đó!
Nếu ảnh in với kích thước 4x6 (inches) dán lại với chiều dài tổng cộng 200 triệu dặm, dài đường từ trái đất đến mặt trời trở về! Trong số hàng ngàn tỉ ảnh này, 87% ảnh chụp từ thiết bị di động có 13% từ máy ảnh chuyên dụng Điều có nghĩa có hàng trăm tỉ ảnh lưu trữ dạng JPEG, chuẩn nén phổ biến nay!
Theo thống kê củaBusiness InsidervàThời báo New York.
Thông thường, điểm ảnh tổng hợp màu bản: đỏ, xanh dương, xanh thiết bị điện tử tổng hợp màu lại để có màu mà nhìn thấy Ứng với màu điểm ảnh, máy tính sử dụng giá trị nguyên dương từ đến 255 (là byte) để thể độ mức độ màu Để lưu trữ vậy, ảnh có kích thước cỡ trung bình, ví dụ như3:4562:304điểm ảnh, tốn tối thiểu3:4562:3043D23:887:872 byte, tức xấp xỉ 22.8 MB Bạn đọc thử xem lại ảnh số với độ phân giải tương tự, thấy số lớn so với số thực tế lưu máy tính hay điện thoại bạn lưu ảnh có phần mở rộng jpg Để có kích thước nhỏ vậy, thiết bị điện tử áp dụng kỹ thuật nén ảnh, mà phổ biến kỹ thuật nén với chuẩn JPEG!
Độc giả tìm hiểu chi tiết chuẩn nén thông qua nguồn khác, nhấn mạnh lại ưu điểm JPEG: tảng nén JPEG dựa biến đổi cosine rời rạc (DCT – discrete cosine transform), phép biến đổi trực giao nên ma trận nghịch đảo ma trận chuyển vị, điều cho phép tính tốn thực nhanh, phù hợp với thiết bị di động! Hơn nữa, biến đổi DCT cho phép dễ dàng giữ lại thành tố quan trọng lược bỏ thành tố không quan trọng, đặc biệt với thị giác người, nhờ mà ảnh nén với tỉ lệ cao!
(67)Tạp chí Epsilon, Số 05, 10/2015
Original Image Compressed Image (high quality) Compressed Image (low quality)
Hình 7.1: Ví dụ nén ảnh JPEG Ảnh bên trái: ảnh gốc, không nén Ảnh giữa, nén với tỉ lệ thấp (cho ảnh chất lượng cao hơn) ảnh bên phải: nén tỉ lệ cao
1
0 0.05 0.1 0.15 0.2 0.25 0.3 0.35
Original Image Benford Law
Compressed Image (high quality) Compressed Image (low quality)
Hình 7.2: Tần suất xuất chữ số sau biến đổi DCT từ ảnh Hình 7.1
một viết Epsilon Việc biết thông tin có nghĩa gì? Bạn đọc thử hình dung ta biết ảnh có nén từ phần mềm chỉnh sửa ảnh, ví dụ Photoshop hay Picasa mà khơng phải từ máy chụp ảnh ảnh bị thay đổi không đáng tin cậy nữa! Hoặc ảnh nén nhiều lần có nghĩa ảnh chép trực tiếp từ máy ảnh mà qua công đoạn trung gian Và vậy, luật Benford "vén màn" bí mật đằng sau ảnh số!
(68)Ghi chú
Mặc dù nhiều tập hợp số liệu tự nhiên, luật có hạn chế Các số khơng ngẫu nhiên, ví dụ kết xổ số hạn chế tập hợp xác suất hạn hẹp
(69)ĐIỀU KIỆN NGOẠI TIẾP CỦA MỘT TỨ GIÁC KHÔNG LỒI VÀ ỨNG DỤNG
Đỗ Thanh Sơn – THPT chuyên KHTN
TÓM TẮT
Bài báo đưa khái niệm tứ giác không lồi ngoại tiếp với số ứng dụng
1 Mở đầu
Chúng ta biết điều kiện ngoại tiếp tứ giác lồi Điều kiện đựơc phát biểu sau Điều kiện cần đủ.Một tứ giác lồi ngoại tiếp đường tròn tổng độ dài cặp cạnh đối
Tuy nhiên việc giải toán, nhiều ta gặp phải tốn khảo sát tính chất tiếp xúc cạnh đường thẳng chứa cạnh tứ giác khơng lồi với đường trịn Trong trường hợp ta khơng thể sử dụng điều kiện ngoại tiếp tứ giác lồi để khảo sát toán
Nội dung báo đề cập đến điều kiện "ngoại tiếp" tứ giác khơng lồi có hình dạng định nghĩa sau ứng dụng để giải toán
2 Các định nghĩa
Định nghĩa 1.Cho tứ giác lồiABCD có cặp cạnh đối không song song GọiM giao điểm đường thẳngAB vàCD,N giao điểm đường thẳngADvàBC (có thể coiB nằm giữaAvàM;Dnằm giữaAvàN) Hình tạo đoạn thẳng liên tiếp AM; M C; CN; NAlà hình tứ giác lõm Chúng ta khảo sát vấn đề liên quan đến hình tứ giác
B
C
D A
M
(70)Nếu cấu hình xem tập hợp gồm tứ giác lồiABCD, tam giácBCM tam giácCDN, ta cịn gọi tứ giác toàn phần
Để cho tiện, từ ta gọi tứ giác có hình dạng định nghĩa tứ giác toàn phần ký hiệu làABCDMN Trong ký hiệu chữA; B; C; D đỉnh tứ giác lồi, điểmM; N đỉnh sinh từ cặp đường thẳng chứa cạnh đối diện tứ giácABCD cắt Cũng thế, đơi ta cịn nói tứ giác tồn phầnABCDMN sinh tứ giác lồiABCD
Các điểmA; B; C; D; M; N đỉnh tứ giác đoạnAB; BC; CD; DA; AM; M C; CN; ND; AN cạnh tứ giác Các đoạn thẳngAC; BD; MN đường chéo tứ giác
Nếu khơng có thích gì, nói tới tứ giác toàn phầnABCDMN ta hiểu tứ giác có cấu định nghĩa
Định nghĩa 2.Ta nói tứ giác tồn phầnABCDMN ngoại tiếp đường tròn.O/, nếu.O/là đường tròn nội tiếp tứ giácABCD Để cho gọn, ta nói tứ giác tồn phầnABCDMN ngoại tiếp, ta hiều tứ giác ngoại tiếp đường tròn
I
A
B
C
D
M
N
3 Điều kiện ngoại tiếp
Định lý 1.Tứ giác toàn phầnABCDMN ngoại tiếp khiAM CCN DAN CCM
K
F A
E L B
C=C'
D
M
(71)Tạp chí Epsilon, Số 05, 10/2015
Chứng minh Giả sửABCDMN ngoại tiếp Theo định nghĩa, tồn đường tròn.O/nội tiếp tứ giác lồiABCD Ta ký hiệu điểmK; L; E; F tiếp điểm của.O/và cạnh tương ứngAB; BC; CD; DA Khi ta cóAM D AKCKM,CN D NL CL, AN DAF CF N,CM D ME CE VậyAM CCN DAN CCN suy raAKCKM C NL CL DAFCF NCME CE (1) VìAK DAF; MK DME; NF DNL; CE DCL, nên (1)
Giả sửAM CCN DAN CCM Ta cần chứng minh rằngABCDlà tứ giác ngoại tiếp Gọi O/là đường tròn nội tiếp trong4MAD Ta chứng minh (O) đường tròn nội tiếp trong4NAB Ta ký hiệuK; E; F tiếp điểm của.O/với cạnh tương ứngAM; MD; DA của4MAD TừN kẻ tới.O/tiếp tuyếnNLkhácNA(Llà tiếp điểm) Đường thẳngNLcắt MDtạiC0và ta coiC nằm giữaC0vàM Theo điều kiện cần ta cóAMCNC0DANCM C0 suy raAM AN D M C0 NC0(2) Từ giả thiết ta suy AM AN D CM CN (3) Từ (2) (3) ta cóCM CN DM C0 NC0suy raCM CN D CM CC C0 NC0hay NC0 D C0C CCN Đẳng thức chứng tỏ C vàC0trùng Tức cạnhBC tiếp xúc với.O/:
Định lý 2.Tứ giác toàn phầnABCDMN ngoại tiếp khiBM CBN DDM CDN:
K
F A
E L B=B'
C
D
M
N
Chứng minh Ta sử dụng ký hiệu chứng minh định lý để chứng minh định lý 2.Gỉa sửABCDMN ngoại tiếp, ta cóBM DMK BK,BN DBLCLN; DM D DECME,DN DNF DF Rõ ràngBM CBN DDM CDN suy raMK BKCBLC LN DDECMECNF DF (*) VìMK DME; NLDNF; BK DBL; DE DDF, nên (*)
(72)4 Ứng dụng
Bây ta ứng dụng hai định lý vào giải toán
Bài tốn Đường trịn tâmI nội tiếp trong4ABC tiếp xúc với cạnhBC; CA; AB lần lượt tạiM; N; E.
a) Chứng minh tứ giácIMBEvàIM CN ngoại tiếp.
b) Chứng minh tiếp tuyến chung hai đường tròn nội tiếp tứ giác và khác đường thẳngIM đi quaA.
A
B C
I
N
M E
O1 K O2
Lời giải a) Rõ ràng ta cóIM DIE vàBM DBE, nên tứ giác lồiIMBE thõa mãn điều kiện IM CBE DIECBM Từ suy tứ giácIMBE ngoại tiếp Ta ký hiệu.O1/là đường tròn nội tiếp tứ giác Tương tự, ta suy tứ giácIM CN ngoại tiếp và.O2/là đường tròn nội tiếp tứ giác
b) Ta thấy rằngIM tiếp tuyến chung của.O1/và.O2/ Kẻ quaA đường thẳng tiếp xúc với.O2/và khácAC GọiK giao điểm đường thẳng vớiIM Ta thấy tứ giác không lồi ACMK ngoại tiếp O2/ nên ta cóAC CMK D AK CM C Để chứng minhAK tiếp xúc với O1/ta cần nghiệm lại điều kiệnAK CMB D MK CAB hay AK MK D AB MB Thật vậy, điều kiệnAC CMK D AK CM C tương đương với AK MK D AC M C D AN CNC / M C D AN C.NC M C / DAN D AE D AB BE D AB BM Đây điều cần chứng minh Ta thấy rằng.O1/và.O2/khác phía tiếp tuyến chungAK, nênAK tiếp tuyến chung hai đường tròn Bài tốn Tứ giácABCD ngoại tiếp đường trịn Trên cạnhAB ta lấy điểmM Đường thẳng
(73)Tạp chí Epsilon, Số 05, 10/2015
B C
A
D
M N
K
J
I
T P
Lời giải Ta thấy đường thẳng IKchứa đường phân giác tam giác BM C AMN kẻ từM Đường thẳngKI chứa phân giác củaANM\ Ta gọi2˛ DANM ; 2ˇ\ D
\
AMN, ta cóIKJ[ D˛Cˇ(góc ngồi của4KMN / Mặt khác ta cóBAD[ D2˛C2ˇ (góc ngồi của4AMN / TừAta kẻ tiếp tuyếnAT tới đường tròn.J /(khácAB) GọiP giao điểm củaAT vàCM Vì.J /nội tiếp tứ khơng lồiBAP C, nên ta cóABCCP DBC C AP tương đươngAB BC DAP CP VìABCDngoại tiếp, nênAB BC DAD CD Từ kết suy raAP CP DAD CDhayAP CCD DADCCP Đẳng thức chứng tỏ tứ giác lồiADCP ngoại tiếp đường tròn Tức làAT tiếp xúc với.I / Do tiaAI phân giác củaDAP[ XétIAJd, ta cóIAJd DIAP[CPAI[ D˛Cˇ:Do đóIKJ[ DIAJd Hơn điểmK vàAcùng phía đối vớiIJ, nênA; J; I; K đồng viên Đó điều phải chứng minh
Bài toán Cho tam giácABC Giả sử đường elip.E/với tiêu điểmB; C cắt cạnh
AB; AC lần lượt tạiM; N Dựng tiếp tuyến với.E/tại điểmM vàN GọiI là giao điểm tiếp tuyến Chứng minh tiaAI là phân giác củaBAC[.
C B
A
M
N I
(74)Lời giải Gọi P giao điểm đoạn thẳng BN CM Trong tứ giác tồn phần AMPNBC ta cóMB CM C DNBCNC Do tứ giác ngoại tiếp Theo định nghĩa tồn đường tròn nội tiếp tứ giácAMPN Cần lưu ý rằngMI vàNI tia phân giác góc đỉnhM vàN tứ giácAMPN Vậy thìI tâm đường trịn nội tiếp tứ giácAMPN Tức làAI tia phân giác củaBAC[
Bài tốn Cho hai đường trịn.O1/và.O2/có bán kính khác nằm nhau.T1T2
vàT3T4là tiếp chung hai đường tròn (T1vàT3là tiếp điểm thuộc.O1/,T2vàT4
thuộc O2// GọiO là giao điểm cuat đường thẳngT1T2vàT2T4 Trên đoạnOT1 ta lấy điểmM vàOT4lấy điểmN GọiMM1 là tiếp tuyến với.O1/khácT1T2,MM2là tiếp tuyến với.O2/khácT1T2,N N1 là tiếp tuyến với.O1/khácT3T4,N N2là tiếp tuyến với O2/khác
T3T4 GọiA là giao điểm củaMM1 vàN N1,B là giao điểm của N N2 vàMM2,C là giao điểm củaMM2vàN N1,D là giao điểm củaMM1 vàN N2 Chứng minh đường thẳng
AB và tiếp tuyến chung ngồi hai đường trịn đồng quy.
O1
O2 T1
T3
T4
T2
O M
M1
M2
N N
N A
B
C D 1
2
Lời giải ví dụ dành cho bạn đọc
Tài liệu tham khảo
(75)KHOẢNG CÁCH GIỮA TÂM ĐƯỜNG TRÒN EULER VÀ TÂM ĐƯỜNG TRÒN APOLLONIUS
Trịnh Xuân Minh – Macau
TÓM TẮT
Như tiêu đề nêu, phần giới thiệu với bạn đọc hệ thức liên hệ tâm hai đường tròn tiếp xúc ngồi với ba đường trịn bàng tiếp tam giác Cách chứng minh tác giả lâu (2009) tương đối cồng kềnh nên hy vọng sau viết có lời giải hình học đơn giản dành cho Để cho ngắn gọn đỡ phức tạp, điều biết xin không chứng minh đây, thay vào người viết thích nguồn để bạn đọc tiện tham khảo
Cho4ABC ký hiệu tương ứng sau: S diện tích4ABC
plà nửa chu vi4ABC
Rlà bán kính đường trịn ngoại tiếp4ABC r bán kính đường trịn nội tiếp4ABC
M ˛M; ˇM; M/nếu˛MMA!CˇMMB!CMM C!D E0 Trước tiên nhắc lại số định lý hệ thức sau:
Định lý Euler.Trong tam giác, chân ba đường cao, ba trung điểm ba cạnh, ba trung điểm ba đoạn thẳng nối ba đỉnh với trực tâm, tất chín điểm nằm đường tròn gọi đường tròn điểm hay đường tròn Euler
(76)Đường trịn Euler có bán kính R
2 hệ thống tâm Kimberling, tâm làX5với ˛X5 Dacos.B C /:
Định lý Feuerbach.Trong tam giác, đường tròn Euler tiếp xúc đồng thời với đường tròn nội tiếp ba đường tròn bàng tiếp
Hình Định lý Feuerbach
Định lý cơng bố năm 1822 nhà hình học người Đức, Karl Wihelm Feuerbach (1800-1834)
(77)Tạp chí Epsilon, Số 05, 10/2015
Hình Đường trịn Apollonius Đường trịn Apollonius có bán kính p
2Cr2
4r có tâm Kimberling X970 với ˛.X970/ D R.p2 r2/acosA a2S:
Một số hệ thức 4:1/ S D abc
4R Dpr D.p a/ra 4:2/ a2Cb2Cc2 D2p2 2r2 8Rr 4:3/ acosACbcosBCccosC D 2S
R 4:4/ cos2ACcos2BCcos2C D3 a
2
Cb2Cc2 4R2 D3
p2 r2 4Rr 2R2 4:5/ abcosC CbccosACcacosB D a
2
Cb2Cc2/ Dp
2 r2 4Rr
4:6/ acosBcosC CbcosC cosACccosAcosB D S R 4:7/ MA2 D ˇMc/
2
C.Mb/2C2bcˇMMcosA ˛M CˇM CM/2
4:8/ ˛M CˇM CM/M S2D˛MAS2CˇMBS2CMCS2 ˛MˇMc2CˇMMa2CM˛Mb2
˛M CˇM CM
Đường tròn Euler đường tròn Apollonius gây ý đặc biệt với thân tơi tính chất tiếp xúc chúng với ba đường tròn bàng tiếp tam giác Cũng mà tơi nghĩ đến tồn hệ thức đẹp liên hệ chúng,
Định lý.Gọi.E; RE/và.E0; RE0/lần lượt đường tròn Euler đường trịn Apollonius 4ABC Khi đóEE02 D.RE CRE0/2
r
RE
Chứng minh.Ta có˛E0 DR p2 r2acosA a2S Áp dụng 4.2 4.3
X
cycli c
˛E0 DR p2 r2 X
cycli c
acosA S X cycli c
a2
DR p2 r2 2S
R S 2p
(78)Áp dụng hệ thức 4.7 vớiM E và˛E Dacos.B C /thu được4AE2 D R2C2bccosA Kết có cách gián tiếp thơng qua việc xét quan hệ vị trí giữaE với điểm đặc biệt khác đường thẳng Euler (trọng tâm, trực tâm, tâm đường đường tròn ngoại tiếp )
Như vậy,
4˛E0AE2 D R2C2bccosAR p2 r2acosA a2S D
R3 p2 r2 8RS2acosAC8R2S p2 r2cos2A R2Sa2 Áp dụng 4.2, 4.3 4.4 ta có
4 X cycli c
˛E0 AE2 DR3 p2 r2 8RS2 X
cycli c
acosA C8R2S p2 r2 X
cycli c
cos2A R2S X cycli c
a2
D
R3 p2 r2 8RS22S R C8R
2
S p2 r2
3 p
2 r2 4Rr 2R2
R2S 2p2 2r2 8Rr
D26R2S p2 r2 16S3 2S p2 r2 4Rr2 p2 r2CR2 D26R2S p2 r2
16S3 4S p2 r22
2S p2 r2
R2 8Rr
C8R3rS D 4S p2 r22C8RS 3RC2r/ p2 r2 16S3C8R3rS
Suy ra, X
cycli c
˛E0 AE2 D S p2 r2
C2RS 3RC2r/ p2 r2
4S3C2R3rS 2/
Lại có
˛E0ˇE0c2 Dc2R p2 r2acosA a2SR p2 r2bcosB b2S
D4R3S p2 r22ccosAcosB 4R2S2 p2 r2.bccosACcacosB/C16R2S4 Áp dụng 4.5 4.6
X
cycli c
˛E0 ˇE0 c
D4R3S.p2 r2/2 X cycli c
ccosAcosB 8R2S2.p2 r2/ X
cycli c
abcosC C48R2S4
D4R3S.p2 r2/2 S R 8R
2
S2.p2 r2/.p2 r2 4Rr/C48R2S4 D 4R2S2 p2 r22C32R3rS2 p2 r2C48R2S4 3/
Sau ta áp dụng (1), (2) (3) vào 4.8 vớiM E0vàS E ta thu 8RrS EE02D
S p2 r22C2RS 3RC2r/ p2 r2 4S3C2R3rSC 4R2S2 p2 r22
32R3rS2 p2 r2
48R2S4 8RrS
D S p2 r22
C2RS 3RC2r/ p2 r2
4S3C2R3rSC pRh p2 r22 8Rr p2 r2 12S2i
(79)Tạp chí Epsilon, Số 05, 10/2015
EE02 D R 2r/ p
2 r22 16Rr2 C
4RS RC2r/ p2 r2 16RrS C
4S r R3 3p2R 2p2r 16RrS
D R 2r/ p
2 r22
C4Rr.RC2r/ p2 r2 16Rr2 C
R3 3p2R 2p2r 4R D R Rr C Rr C
p2R 4r 4p2 C p2 8r2 16 C r2 16 C p4 16r2 p4 8Rr
p2r 4R r3 8R D R 2r R
CR p
Cr2 4r
p2Cr2 C
p2 Cr2 4r
2 p2Cr22 8Rr D
" R
2 C
p2Cr2 4r #2 2r R
D.RE CRE0/2
1 r RE
điều phải chứng minh:
Tài liệu tham khảo
(80)(81)TỔNG QT MỘT BÀI TỐN THI VƠ ĐỊCH NGA NĂM 2005
Trần Quang Hùng – Phan Anh Quân
TÓM TẮT
Bài viết tổng qt hóa tốn từ thi vô địch Nga năm 2005 công cụ điểm đẳng giác với lời giải túy hình học
Bài tốn sau đề nghị tác giả Andrey Badzyan kỳ thi vô địch tồn Nga năm 2005 vịng thi tỉnh dành cho lớp
Bài toán Cho tam giác ABC với đường tròn ngoại tiếp O/ và đường tròn nội tiếp là
.I /.M là trung điểm củaAC vàN là trung điểm cung ACd chứa B Chứng minh rằng ∠IMAD∠I NB.
A B
C O
I
M N
E
Lời giải Gọi BI cắt.O/tại E khácB Từ kết quen thuộcE tâm ngoại tiếp tam giác IBC Từ kết hợp hệ thức lượng tam giác vng thìEI2 DEA2 DEM:EN Từ hai tam giácEMI vàEI N đồng dạng Suy ra∠IMAD∠EI N IME 90ıD∠EI N 90ıD ∠I NB Ta có điều phải chứng minh
Nhận xét.Bài toán kết đẹp có nhiều ý nghĩa việc ứng dụng phát triển Chúng xin giới thiệt toán toán ứng dụng kết sau
Bài toán Cho dây BC cố định đường tròn.O/.A di chuyển trên O/,I là tâm nội tiếp tam giácABC.IA cắt O/tại D khácA.M là trung điểm BC.N thuộc O/sao cho
(82)D A
B C
O I
M K S
T P
N
Lời giải Gọi tiaIM cắt.O/tạiT, tiaMI cắt.O/tạiS,MD cắt.O/tạiK cố định Ta thấy ID2 DBA2 DDM:DKsuy tam giác4DMI 4DIKsuy ra∠MIDD∠IKM 1/ Ta lại có DN k S T suy DTd D NS Từd PN k AD suy dNA D PD Từ suy rad
d
PD DSAc CDTd suy ra∠PKD D∠DI T 2/
Vậy từ.1/; 2/suy raP; I; K thẳng hàng hayP I quaK cố định
Nhận xét.ĐiểmP tiếp điểm đường trịn mixtilinear nội với đường trịn.O/ Bài tốn lần mở rộng [2] tác giả Trần Quang Hùng sau
Bài toán Cho tam giác ABC nội tiếp đường tròn O/vàP; Q đẳng giác tam giác
ABC AP cắt.O/tạiDkhácA.DN là đường kính của.O/.ON cắtBC tạiM Chứng minh rằng∠PMB D∠ANQ.
Sau nhờ lời giải tác giả Phan Anh Quân, toán mở rộng thêm lần sau Bài toán Cho tam giác ABC nội tiếp đường tròn O/vàP; Q đẳng giác tam giác
ABC AP cắt O/ tại D khácA. M là điểm thuộc đoạn BC. DM cắt O/ tại N khác D. Chứng minh rằng∠PMB D∠ANQ.
Để giải toán sử dụng bổ đề sau, tham khảo [3] Lời giải bổ đề đề nghị nhiều tác giả [3] xong lời giải sau tác giả Phan Anh Quân coi ngắn gọn đẹp
Bổ đề Cho tam giácABC nội tiếp.O/.P; Qđẳng giác tam giácABC.AP cắt.O/
(83)Tạp chí Epsilon, Số 05, 10/2015
A
B C
O
P Q
M E
N
G H
Chứng minh bổ đề Gọi AQ cắt O/tại N khácA cắtBC H Do P; Q đẳng giác, ta dễ thấy tam giác đồng dạng 4CHN 4ACM 4CPM 4QCN (g.g) suy HN:AM DCM:CN DQN:PM suy MP
MA D NH NQ D
ME
MQ vậyPE kAQ Trở lại toán
A
B C
O P
Q
D
R M
N
T
Lời giải Phan Anh Quân Gọi DQ cắt BC R theo bổ đề biết thìPR k AQ Lấy T thuộc DN cho P T k AN Theo định lý Thales suy DR
DQ D DP DQ D
DT
(84)RT kQN Vậy hai tam giácAQN vàPRT đồng dạng có cạnh tương ứng song song Từ đó∠BMD D∠M CDC∠MDC D ∠BADC∠CAN D ∠QAC C∠CAN D ∠QAN D ∠RP T Từ tứ giácP TMRnội tiếp Suy ra∠PMB D∠P TRD∠ANQ
Bài tốn mở rộng có nhiều ứng dụng hay, tác giả Trần Quang Hùng đề nghị [2] Bài toán Cho tam giácABC vớiP; Qlà hai điểm đẳng giác phân giác góc∠BAC và
M là trung điểm củaBC Chứng minh tâm ngoại tiếp tam giác PQM luôn thuộc một đường thẳng cố định khiP thay đổi.
A
B C
O M N
P
Q Q Q Q Q Q Q Q Q Q Q Q Q Q Q Q Q Q Q Q Q Q Q Q Q Q Q Q Q Q Q Q Q Q Q Q Q Q Q Q QQQQQQQQQQQQQQQQQQQQQQQQQ
H
Lời giải LấyH trênBC choPH ?BC GọiN trung điểmBCd chứaA DoP; Qđẳng giác áp dụng tốn tổng qt ta có∠QNAD∠PMB suy ra∠AQN D∠HPM D∠PMN (chú ý∠NADD90ı), tứ giácQPMN nội tiếp Vì tâm ngoại tiếp tam giácPQM nằm trung trực củaMN cố định
Các bạn làm toán sau để luyện tập
Bài toán Cho tam giác ABC nội tiếp đường tròn O/vàP; Q đẳng giác tam giác
ABC AP cắt.O/tạiD khácA.DN là đường kính của.O/.ON cắtBC tạiM GọiPM cắt
AQtạiT.
a) Chứng minh rằngQ; M; N; T cùng thuộc đường tròn. b) Chứng minh rằngPM kAQkhi khiQthuộcOM.
Bài toán Cho tam giácABC nội tiếp đường tròn.O/ Đường thẳng`đi quaO cắtBC tại
M.Qlà điểm thuộc` vàP là đẳng giác củaQtrong tam giácABC Chứng minh rằng
AP và`cắt đường tròn.O/khi khiPM kAQ.
Tài liệu tham khảo
[1] Đề thi vô địch toàn Nga năm 2005 từ AoPS forum
http://www.artofproblemsolving.com/community/c6h32163
[2] Tran Quang Hung and Pham Huy Hoang, Generalization of a problem with isogonal conjugate points
http://www.jcgeometry.org/Articles/Volume2/JCG2013V2pp39-42.pdf
[3] Topic Isogonal points and parallelism
(85)ÁP DỤNG DÃY SỐ VÀO GIẢI CÁC PHƯƠNG TRÌNH VÀ BẤT PHƯƠNG TRÌNH HÀM
Đỗ Minh Khoa – Võ Quốc Bá Cẩn
1 Lời dẫn
Các toán phương trình hàm bất phương trình hàm ln thú vị lơi người làm tốn Từ phương trình đó, vài phép đơn giản, ta tìm tính chất đặc biệt hàm số cho chí cơng thức tổng qt hàm Tuy nhiên, sâu vào vấn đề việc dùng phép để giải chưa đủ, đặc biệt với tốn bất phương trình hàm Do vậy, cần có cơng cụ khác bỗ trợ để tăng cường thêm tính hiệu phương pháp Lúc này, việc sử dụng dãy số giới hạn cách linh hoạt giúp đường trở nên sáng sủa dễ dàng nhiều
Nhận thấy vai trò lớn dãy số giới hạn toán phương trình, bất phương trình hàm, chúng tơi định thực viết để chia sẻ kinh nghiệm cách sử dụng để học hỏi nhận ý kiến đóng góp từ quý đồng nghiệp gần xa
2 Sử dụng giới hạn tốn phương trình, bất phương trình hàm
Dưới số kỹ thuật sử dụng chặn kẹp dãy số thường sử dụng giải tốn: Trong nhiều trường hợp, ta cần tìm cơng thức tổng quát hàm số,
hướng mà ta nghĩ đến thiết lập bất đẳng thức dạng: anf x/bn;
ở đây.an/; bn/là hai dãy chọn cho bất đẳng thức với mọin(ứng với mỗixcố định) Lúc này, limanDlimbn DL.x/thì cách chuyển sang giới hạn, ta tìm cơng thức tổng qt củaf x/làL.x/:
Nếu cần suy xét tính chất củaf x/;ta thiết lập bất đẳng thức dạng: A.f /anB.f /;
(86)Bài tốn Tìm tất hàm sốf WR!Rthỏa mãn: Với mọix2 R;ta có
f 2x/D2f x/ (2.1)
và
ˇ
ˇf x/ x ˇ
ˇ1: (2.2)
Lời giải Từ (2.1), cách sử dụng quy nạp, ta chứng minh f 2nx/ D2nf x/
với mọix 2Rvà với mọin2N:Bây giờ, ta chứng minhf x/Dvới mọix 2R:Thật vậy,
giả sử cóx0 2Rsao chof x0/Ôx0:tf x0/Dx0C"vi"Ô0:Khi ú, ta cú
f 2nx0/D2nf x0/D2nx0C2n" với mọin2N:Suy
ˇ
ˇf 2nx0/ 2nx0ˇˇD2nj"j
với mọin2N:Do (2.2) nênˇˇf 2nx0/ 2nx0j 1:Kết hợp với đẳng thức trên, ta thu 2nj"j
với mọin2N:Tuy nhiên, kết thỏa mãn với mọin2N:Mâu thuẫn thu
được cho ta kết vừa khẳng định trên, tứcf x/Dx với mọix2 R:
Bài toán (VMO, 2013-A) GọiF là tập hợp tất hàm sốf WRC !RCthỏa mãn
f 3x/f f 2x/ Cx
với mọix 2RC:Tìm sốAlớn để với mọif 2F và với mọix > 0;ta có
f x/Ax: Lời giải Từ giả thiết, ta suy
f x/f f 2x C x (2.3)
với mọix 2RC:Dof
f 2x
> 0nên ta có
f x/ > x
với mọix 2RC:Đặta1 D
1
3:Từ hai bất đẳng thức trên, ta suy f x/ > a1f
2x
3
Cx > a
2 2x C x D
2a21C1 x:
với mọix2RC:Bằng cách đặta2 D 2a
2 1C1
3 áp dụng kết vừa thu vào (2.3), ta có f x/ > a2f
2x
3
C x > a
2 2x C x D
(87)Tạp chí Epsilon, Số 05, 10/2015
với mọix 2RC:Bằng cách lặp lại quy trình đánh giá giống vậy, ta thu
f x/ > anx (2.4) với mọix 2RCvà với mọin2N;trong đó.an/là dãy truy hồi xác định bởia1 D
1 anC1 D
2a2nC1 :
Ta chứng minh được.an/là dãy tăng bị chặn
2 nên có giới hạn hữu hạn Từ dễ dàng tìm limanD
2:Bây giờ, (2.4), ta chon! C1thì f x/.liman/x D
2x; 8x 2R
C:
Mặt khác, dễ thấy hàm sốf x/D
2xthỏa mãn điều kiện đề VậyAmaxD 2: Bài tốn (Bulgaria, 2006) Tìm tất hàm sốf WRC !RCthỏa mãn đẳng thức
f x Cy/ f x y/D4pf x/f y/ (2.5)
với số thựcx > y > 0:
Có thể đốn hàm số thỏa mãn phương trình có dạngf x/Dkx2:Một cách tiếp cận thường thấy chứng minhf 2x/D4f x/dựa việc tínhf 5x/bằng hai cách để từ quy nạp lênf nx/Dn2f x/;bước cuối sử dụng tính đơn điệu củaf để suy cơng thức tổng qt cho
Ở đây, giới thiệu cách tiếp cận khác để chứng minhf 2x/D 4f x/:Có thể thấy rằng, ta chứng minh lim
x!0Cf x/D0và limx!xC
f x/Df x0/thì từ phương trình đề bài, cách chox!yC;ta thu điều ta cần
Lời giải Trong (2.5), ta thayxbởixCythì
f xC2y/ f x/D4pf xCy/f y/ (2.6) với x; y > 0: Từ đây, dễ dàng suy f tăng ngặt RC: Từ kết dẫn đến f xCy/ > f y/;và ta suy
f xC2y/ > 4f y/
với mọix; y > 0:Tiếp tục, ta thayxbởixCyvào (2.6) sử dụng bất đẳng thức f xC3y/Df xCy/C4pf xC2y/f y/ > f y/C4p4f y/f y/D9f y/:
Cứ thế, thế, ta quy nạp lên rằng:
(88)với mọix; y > 0và với mọin2N:Thayy D
n vào bất phương trình trên, ta f
n
< f 1Cx/ n2
với mọix > 0và với mọin2N:Cố địnhxvà chon ! C1;ta thu lim
n!C1f
1 n
D0; từ suy
lim
x!0Cf x/D0: Bây giờ, (2.6), ta choy < x thu
0 < f xC2y/ f x/ < 4pf 2x/f y/:
Cố địnhxvà choy !0Cthì ta có lim y!0C
f x C2y/ f x/
D0;hay
lim x!x0C
f x/Df x0/:
Đến đây, cách cố địnhytrong (2.5) chox !yC;ta f 2y/D4f y/
với mọiy > 0:Dựa kết này, ta quy nạp f nx/Dn2f x/;
với mọix > 0vàn2N:(Chỉ việc choxD2y; 3y; : : : ; nyvào (2.5) được.) Từ suy
f n/Dn2f 1/
và
fm n
D f n m
n
n2 D f m/
n2 D m2
n2f 1/
với mọim; n2N:Nói riêng, ta cóf x/D kx2với mọix QC:Dof đơn điệu tậpRC
trù mật trongQCnên ta cóf x/Dkx2với mọix > 0:Hàm thỏa mãn yêu cầu đề Bài tốn (Bulgaria, 1998) Chứng minh khơng tồn hàm sốf WRC !RCthỏa mãn
f2.x/f xCy/
f x/Cy
(2.7)
với cặp số thực dươngx; y:
Lời giải Giả sử tồn hàm sốf Khi đó, ta viết lại giả thiết dạng f2.x/Cyf x/ yf x/f xCy/
f x/Cy ;
hay
f x/Cy
f x/ f xCy/
(89)Tạp chí Epsilon, Số 05, 10/2015 Từ đây, ta có
f x/ f xCy/ yf x/ f x/Cy
với mọix; y 2RC:Kết chứng tỏf hàm giảm thực trênRC:Bây giờ, ta cố
địnhx0 2RCvà chọn số tự nhiênnsao chonf xC1/1:Khi đó, ta có
f
xC k n
f
xC kC1 n
f xC k n
1n f xCkn
C 1n >
f xC1/ 1n f xC1/C 1n
n n n C n D 2n
với mọik 2N; k < n:(Chú ý hàm sốg.t /D t
t Cu vớiu > 0là hàm tăng trênR
C:)
Trong bất đẳng thức trên, choknhận giá trị từ0đếnn 1và cộng tất bất đẳng thức lại theo vế Khi đó, ta thu kết sau
f x/ f xC1/ >
2 (2.8)
với mọix > 0:Bây giờ, chọn số tự nhiênmsao chom2f x/;ta có f x/ f x Cm/D
f x/ f xC1/
C C
f xCm 1/ f m/ > m
2 f x/; từ suy raf xCm/ < 0;mâu thuẫn Vậy không tồn hàmf thỏa mãn (2.7)
Nhận xét Bài toán xuất đề thi IMC năm 1999 ời giải đáp án Để thiết lập tính chất (2.8) trên, ta sử dụng tổng sai phân Đây phép toán thú vị giúp ta thu nhiều chất quan trọng tốn có dạng “hiệu” Ngồi cách giải nêu trên, ta có cách khác sử dụng dãy truy hồi sau: Thayy Df x/vào (2.7), ta thu
f xCf x/
f x/
2 (2.9)
với mọix > 0:Trong ta thayxbởixCf x/thì có f xCf x/Cf xCf x/
f xCf x/
2 f x/
22
với mọix > 0:Mặt khác, theo (2.9), ta cóxCf x/Cf xCf x/
xCf x/Cf x/ : Từ đó, sử dụng tính nghịch biến hàmf;ta thu
f
xC
1C
f x/
f x/
22 (2.10)
với mọix > 0:Từ (2.9) (2.10) kết hợp vớif nghịch biến, cách sử dụng liên tục phép thayxbởixCf x/;ta dễ dàng chứng minh
f
xC
1C1
(90)với mọix > 0vàn2 N:Do1C
2 C C 2n D2
1
2n < 2vàf giảm nên từ trên, ta suy f xC2f x/ f x/
2nC1
với mọix > 0vàn2N:Trong bất đẳng thức trên, ta chon! C1thì thu
f xC2f x/ 0; mâu thuẫn vìf ln nhận giá trị dương
Bài tốn (Bulgaria, 2008) Tìm tất hàmf WR!Rthỏa mãn
f x Cy2/.yC1/f x/ (2.11)
với cặp số thựcx; y:
Lời giải Thayy D 1và thayx bởix 1vào (2.11), ta thu f x/0
với mọix 2R:Từ đây, kết hợp với (2.11), ta suy
f xCy/ pyC1f x/f x/
với mọix 2Rvà với mọiy 0:Kết chứng tỏf hàm không giảm trênR:
Bây giờ, ta viết lại (2.11) dạng
f x Cy2/ f x/yf x/ (2.12) với mọix; y 2R;và xét hai dãy.an/; bn/vớia0 Db0 D0;
an D1C
22 C C
n2; bnD1C
2 C C n:
Trong (2.12), ta thayxbởixCak thayybởi
kC1 có f x CakC1/ f xCak/
f xCak/ kC1
f x/ kC1
với mọix 2Rvàk2N:Trong bất đẳng thức trên, chok D0; 1; : : : ; n 1và cộng tất
cả bất đẳng thức lại theo vế, ta thu
f x Can/ f x/bnf x/
với mọix 2Rvàn2 N:Mặt khác, ta dễ dàng chứng minh đượcan< 2vàbn! C1(đây
hai kết quen thuộc, bạn đọc tự chứng minh) Do đó, từ bất đẳng thức trên, ta có f x C2/ f x/f xCan/ f x/bnf x/
với mọix 2Rvàn2N:Nếu tồn tạix0 2Rsao chof x0/ > 0thì từ bất đẳng thức trên,
cách thayx Dx0và chon! C1;ta thu điều mâu thuẫn (vế phải tiến đến dương vô vế trái số) Do vậy, ta phải cóf x/D0với mọix 2R:Dễ thấy hàm
(91)Tạp chí Epsilon, Số 05, 10/2015
Nhận xét Ngồi cách sử dụng tổng sai phân thơng qua hai dãy.an/; bn/như trên, ta tiếp cận toán theo cách khác sau: Ta xétx; y thayy py vào bất phương trình (2.11) thu
f xCy/ pyC1
f x/; 8x; y0: Đến đây, cách sử dụng liên tiếp bất đẳng thức trên, ta có
f xCy/Df
xC n 1/y n C y n ry n C1
f
xC n 1/y n
D r
y n C1
f
xC.n 2/y n C y n r y n C1
2 f
xC n 2/y n
r
y n C1
n f x/
với mọix; y 0vàn2N:Từ đó, ta suy
f x/D0 (2.13) với mọix 0:Thật vậy, giả sử tồn tạix0 0sao chof x0/ > 0:Thayx D x0vào bất đẳng thức cố địnhy Dy0> 0;sau chon! C1thì có
f x0Cy0/
f x0/ n!C1lim
1C r y n n
D C1:
Mâu thuẫn chứng tỏ khơng tồn sốx0 nói trên, hay nói cách khác, (2.13) Bây giờ, ta xétx < 0và thayy Dp xvào (2.11)
0Df 0/ p xC1f x/0
với mọix < 0:Từ suy raf x/D0với mọix < 0:Tóm lại, ta cóf x/D0với mọix2 R:
Đơi khi, ta sử dụng cơng thức nghiệm phương trình sai phân để tìm cơng thức tổng qt cho dãy lặp f x/;rồi từ dựa vào miền giá trị củaf x/ mà suy tính chất đặc biệt hàm số cho
Bài tốn Tìm tất hàm sốf WRC!RCthoả mãn
f f x/D6x f x/
với số thực dươngx:
Lời giải Cố định x > 0và đặtf0.x/ D x; fn.x/ D f fn 1.x/
:Từ giả thiết, cách sử dụng quy nạp, ta chứng minh
fnC2.x/CfnC1.x/ 6fn.x/ D0
với mọin2N:Đây phương trình sai phân tuyến tính cấp hai nên cách xét phương trình
đặc trưng nó, ta tìm cơng thức tổng quát củafn.x/là fn.x/D 2x f x/
5 3/ n
C3xCf x/
(92)với mọin2N:Dofn.x/ > 0nên kết trên, ta suy
2x f x/
3
n
C3xCf x/ >
với mọin2N:Nếu2x f x/ > 0thì cách xétnlẻ,nD2kC1và chok! C1;ta có
thể thấy bất đẳng thức ln Cịn nếu2x f x/ < 0thì cách xétn chẵn,nD2kvà chok! C1;ta thu kết luận tương tự Do vậy, ta phải có
f x/D2x: Hàm thoả mãn yêu cầu toán
Nhận xét Bài tốn giải phương pháp kẹp dãy số sau: Xét hai dãy số.an/và.bn/được xác định bởia0 D0; b0 D6và
anC1 D
1Cbn; bnC1 D 1Can: Dễ thấyan; bn> 0với mọin1:Ta có
anC2 D
1CbnC1 D 1C 1C6an
D 6.1Can/ anC7 ;
janC2 2j D
4jan 2j anC7
4
7jan 2j:
Từ đây, cách sử dụng nguyên lý ánh xạ co, ta chứng minh lima2nDlima2nC1 D2; hay liman D2:Tương tự, ta có limbn D2:
Bây giờ, ta chứng minh quy nạp
anx < f x/ < bnx (2.14) với mọin 2N:Dof f x/> 0nên6x f x/ > 0;từ ta dễ thấy khẳng định với
nD0:Giả sử khẳng định vớinDk;tức ta có akx < f x/ < bkx:
Khi đó, thayxbởif x/vào bất đẳng thức sử dụng giả thiết, ta thu akf x/ < f f x/
< bkf x/; hay
akf x/ < 6x f x/ < bkf x/: Từ đây, ta dễ dàng suy
akC1x D
1Cbkx < f x/ <
1Cakx DbkC1x:
Như vậy, khẳng định vớinDkC1:Bất đẳng thức (2.14) chứng minh
(93)Tạp chí Epsilon, Số 05, 10/2015
Bài tốn (APMO, 1989) Tìm tất song ánh tăng thực sựf WR!Rthoả mãn
f x/Cg.x/D2x
với mọix 2R;trong đóg.x/là hàm ngược củaf x/:
Lời giải Đặt f0.x/ D xvà fn.x/ D f fn 1.x/; tương tự chogn.x/:Thay x bởif x/vào phương trình đề bài, ta đượcf2.x/Cx D2f x/:Từ đó, quy nạp, ta chứng minh
fnC2.x/ 2fnC1.x/Cfn.x/D0:
Giải phương trình sai phân này, ta tìm
fn.x/DxCnf x/ x với mọin2N:Một cách tương tự, ta chứng minh
gn.x/DxCn
g.x/ x
Dx n
f x/ x :
Dof x/là song ánh tăng ngặt nênfn.x/vàgn.x/cũng hàm tăng thực Do đó, với x > y;ta cófn.x/ > fn.y/vàgn.x/ > gn.y/:Nói cách khác, ta có
xCnf x/ x> yCnf y/ y,nnf x/ x f y/ yo> y x (2.15)
x nf x/ x> y nf y/ y,nnf x/ x f y/ yo< x y: (2.16) Nếu f x/ x > f y/ y cách chon ! C1trong (2.16), ta thu điều mâu thuẫn Còn nếuf x/ x < f y/ ythì cách chon! C1trong (2.15), ta có kết tương tự Do vậy, ta phải có
f x/ xDf y/ y
với mọix > y:Nói cách khác,f x/DxCcvới mọix (ở đâyc số đó) Hàm thoả mãn yêu cầu toán
Nhận xét Bài toán đề dự tuyển IMO năm 1979
Bài toán (BMO, 2009) Tìm tất hàm sốf WN!Nthoả mãn
f f2.m/C2f2.n/
Dm2C2n2 (2.17)
với cặp số nguyên dươngm; n:
Lời giải Từ giả thiết, dễ thấyf đơn ánh Chú ý với mọin2N;ta có
.nC4/2C2.nC1/2 Dn2C2.nC3/2: Suy
f f2.nC4/C2f2.nC1/
(94)Dof đơn ánh nên từ đẳng thức này, ta suy
f2.nC4/C2f2.nC1/Df2.n/C2f2.nC3/:
Đây phương trình sai phân tuyến tính cấp4của dãyf2.n/:Giải phương trình này, ta tìm f2.n/ DAn2CBnCC CD 1/n (2.18) với mọin2 N:Bây giờ, ta tìm giá trị cụ thể củaA; B; C; D:Lấy bình phương hai vế
của (2.17) sử dụng (2.18), ta
AA.m2C2n2/CB.mC2n/C3C CD 1/mC2D 1/n2 CBA.m2C2n2/CB.mC2n/C3C CD 1/mC2D 1/n CC CD 1/A.m2C2n2/CB.mC2n/C3CCD 1/mC2D 1/n D.m2C2n2/2:
Cố địnhn:Chia hai vế đẳng thức chom4rồi chom ! C1;ta tìm đượcAD1:Do
.m2C2n2/CB.mC2n/C3C CD 1/mC2D 1/n2 CB
.m2C2n2/CB.mC2n/C3C CD 1/mC2D 1/n CC CD 1/.m2C2n2/CB.mC2n/C3CCD 1/mC2D 1/n D.m2C2n2/2; hay
2.m2C2n2/B.mC2n/C3C CD 1/mC2D 1/n C
B.mC2n/C3C CD 1/mC2D 1/n2 CB
.m2C2n2/CB.mC2n/C3C CD 1/mC2D 1/n CC CD 1/.m2C2n2/CB.mC2n/C3CCD 1/mC2D 1/n D0:
Cố địnhn:Chia hai vế đẳng thức chom3rồi chom! C1;ta tìm đượcB D0:Do đó, đẳng thức viết lại thành
2.m2C2n2/
3C CD 1/mC2D 1/nC
3C CD 1/mC2D 1/n2 CC CD 1/.m2C2n2/C3CCD 1/mC2D 1/n D0:
Trong đẳng thức trên, xétmchẵn,nchẵn chom; n! C1;ta dễ thấyC CD D0:Tương tự, xétmlẻ,nlẻ chom; n! C1;ta cóC D D0:Do đóC DD D0:Tóm lại, ta cóA D1vàB D C D D D 0:Suy f2.n/ Dn;hayf n/ Dnvới mọin2 N:Hàm
thoả mãn u cầu tốn
Đơi khi, ta phải xử lý toán với hàm liên tục Khi đó, tính chất thường sử dụng là: Nếu hàm sốf liên tục tạix0thì lim
x!x0
f x/Df x0/:
Nếu hàm sốf liên tục từ khoảngAvào khoảngB đơn ánh.A; B R/thì hàm
đơn điệu thực
(95)Tạp chí Epsilon, Số 05, 10/2015
Nếu hàm sốf liên tục từ khoảngARvào khoảngB R;vàf x1/Da; f x2/Db
(vớix1; x2 2Avàa; b 2B; a < b) thìf nhận giá trị trênŒa; b:
Nu hm sf liờn tc t AvoB vf x/ Ô M (A; B R; M số) hai điều sau thoả mãn:
ı f x/ > M với mọix 2AI ı f x/ < M với mọix 2A:
Bài tốn Tìm tất hàm liên tụcf WR!Rthoả mãn
f f x/Df x/C2x
với số thựcx:
Lời giải Từ giả thiết, dễ thấyf đơn ánh vàf 0/ D 0:Dof liên tục nên hàm đơn điệu thực Tiếp theo, ta chứng minhf toán ỏnh Tht vy, gi s tn ti sa2Rf x/Ôa
với số thựcx:Khi đó, ta cóf x/ > avới mọix 2Rhoặcf x/ < x với mọix2 R:
Giả sửf x/ > avới mọix 2R:Nếuf giảm ngặt ta có
2xCa < 2xCf x/Df f x/< f a/ với mọix;mâu thuẫn Do đóf tăng ngặt Suy ra, với mọix < 0;ta có
f x/ > f x/C2x Df f x/
> f a/:
Dof tăng ngặt nên từ đây, ta cóx > avới mọix < 0;mâu thuẫn
Tương tự, trường hợpf x/ < a với mọix R;ta thu điều mâu thuẫn Như
vậy,f tồn ánh song ánh Bây giờ, đặtf0.x/Dxvàfn.x/Df fn 1.x/
;ta chứng minh fnC2.x/ fnC1.x/ 2fn.x/D0
với mọin2N:Từ suy
fn.x/D 2x f x/
3 1/ n
CxCf x/
n:
Dof song ánh nên tồn hàm ngược Gọig.x/là hàm ngược củaf x/và đặtg0.x/Dx; gn.x/Dg gn 1.x/:Thayxbởig.x/vào phương trình ban đầu, ta
2g.x/Cx f x/D0 (2.19) với mọix 2R:Trong (2.19), ta tiếp tục thayxbởig.x/thì thu
2g2.x/Cg.x/ xD0 với mọix 2R:Từ đó, quy nạp, ta chứng minh
(96)với mọin2N:Suy
gn.x/D x 2g.x/
3 1/ n
C 2xC2g.x/
n :
Từ (2.19) suy ra2g.x/Df x/ x:Do gn.x/D 2x f x/
3 1/ n
C xCf x/
1
n
với mọin2N:Xét hai trường hợp sau:
Trường hợp 1:f tăng ngặt.Dễ thấyfn.x/vàgn.x/cũng tăng ngặt Dof 0/ D0nêng.0/D0 vàgn.0/ D0:Với số thựcx > 0;ta cógn.x/ > gn.0/D0nên
2x f x/
3 1/ n
C xCf x/
n >
với mọin 2N:Nếu2x f x/ > 0thì cách chonlẻ vàn ! C1;ta thu điều
mâu thuẫn Cịn nếu2x f x/ < 0thì cách chonchẵn vàn! C1;ta thu kết tương tự Do đó, ta cóf x/D2xvới mọix > 0:
Chứng minh tương tự, ta cóf x/D2xvới mọix < 0:Như vậy, trường hợp này, ta tìm đượcf x/D2x:Hàm thoả mãn yêu cầu toán
Trường hợp 2:f giảm ngặt.Dễ thấyf2n.x/tăng ngặt,f2nC1.x/giảm ngặt vàfn.0/D0:Với
mỗix > 0;ta cóf2n.x/ > f2n.0/D0Df2nC1.0/ > f2nC1.x/nên 2x f x/
3 C
xCf x/
n
> > f x/ 2x C
xCf x/
2nC1
với mọin2N:NếuxCf x/ < 0thì cách chon! C1;bất đẳng thức vế trái khơng
thể thoả mãn Cịn nếuxCf x/ > 0thì cách chon! C1;bất đẳng thức vế bên phải khơng thể thoả mãn Do vậy, ta phải cóf x/D x với mọix > 0:
Chứng minh tương tự, ta cóf x/D xvới mọix < 0:Tóm lại, trường hợp này, ta cóf x/D xvới mọix:Hàm thoả mãn yêu cầu toán
Bài tốn 10 (Chọn đội tuyển ĐH Vinh, 2012) Tìm tất hàm sốf liên tục từ tậpŒ0; C1/
vàoŒ0; C1/thoả mãn đẳng thức
2f x/Df
x x2CxC1
Cf
xC1
2
với số thực không âmx:
Lời giải Dof liên tục trênŒ0; C1/nên liên tục trênŒ0; 1:Từ suy tồn số a; b 2Œ0; 1sao cho
f a/D max
x2Œ0; 1f x/DM
f b/D
(97)Tạp chí Epsilon, Số 05, 10/2015
Do2M D2f a/Df a C1 Cf a
a2CaC1
và
f
aC1
M; f
a a2CaC1
M
(chú ý aC1
2 1và
a
a2CaC1 < 1) nên ta suy
f a C1 DM:
Từ đây, quy nạp, ta chứng minh f
aC2n 2n
DM
với mọin2N:Trong đẳng thức này, chon! C1;ta đượcf 1/DM:Chứng minh tương tự,
ta cóf 1/Dm:Do đóM Dm:Điều chứng tỏf hàm đoạnŒ0; 1: Giả sửf x/cvới mọix 2Œ0; 1:Khi đó, ta viết lại phương trình ban đầu dạng
f x/D 2f x C1 C 2c: Từ đẳng thức này, ta chứng minh quy nạp
f x/D 2nf
xC2n 2n C C
22 C C 2n
c
với mọin2N:Chon! C1;ta đượcf x/c:Hàm thoả mãn u cầu đề
Bài tốn 11 Tìm tất hàm sốf liên tục từRvàoRthỏa mãn điều kiện:
f x Cf yCz/Cf y Cf zCx/Cf zCf xCy/D0 (2.20)
với số thựcx; y; z:
Lời giải Dễ thấyf x/0là nghiệm phương trình nên ta cần xétf x/60: Thayx; y; zbởi x
2 vào (2.20), ta thu fx
2 Cf x/
D0
với mọix 2R:Tiếp tục, thayxbởi x
2 Cf x/vào đẳng thức trên, ta f
x 22 C
1 2f x/
D0
với mọix 2R:Từ đây, quy nạp, ta chứng minh
f
x 2nC1 C
1 2nf x/
(98)với mọix 2Rvàn2N:Trong đẳng thức trên, cố địnhxvà chon! C1;sau sử dụng tính
liên tục củaf;ta suy raf 0/D0:Từ đây, cách thayx Dy vàz D xvào (2.20), ta có f f 2x/ xD 2f x/
vi mix 2R:Dof x/60nờn tn tix0sao chof x0/Ô0:Xột dóy.an/c xỏc định
a0Dx0vàanC1 Df 2an/ an:Khi đó, dễ thấy
f an/D 2f an 1/D 2/2f an 2/D D 2/nf x0/ với mọin2N:Nếuf x0/ > 0thì ta có lim
n!C1f a2n/D C1và
lim
n!C1f a2n 1/D 1:
Cịn nếuf x0/ < 0thì ta có kết ngược lại Nhưng hai trường hợp này, ta thấy rằngf x/không bị chặn, màf liên tục nên tồn ánh
Đến đây, cách thayy Dz D0vào (2.20), ta có 2f f x/
Cf x/D0
với mọix 2R:Dof tồn ánh nên ta cóf x/D x
2; 8x 2R:Thử lại, ta thấy thỏa mãn Vậy phương trình cho có hai nghiệm làf x/D0vàf x/D x
2:
Bài toán 12 (IMO, 2013) Cho hàm sốf xác định tậpQCvà nhận giá trị tậpRthỏa
mãn đồng thời điều kiện sau:
(1) f x/f y/f xy/với mọix; y2 QCI (2.21) (2) f xCy/f x/Cf y/với mọix; y 2QCI (2.22)
(3) tồn số hữu tỉa > 1sao chof a/ Da: (2.23)
Chứng minh rằngf x/Dxvới mọix 2QC:
Lời giải Thayx D avày D 1vào (2.21), ta đượcaf 1/ a;tứcf 1/ 1:Từ đây, cách sử dụng quy nạp kết hợp với (2.22), ta chứng minh
f n/ n
với mọin2N:ThayxD p
q vớip; q 2N
; p; q/D1vày Dqvào (2.21), ta được
f
p q
f q/f p/p > 0:
Màf q/ q > 0nên từ ta có
f x/ >
với mọix QC:Kết hợp với (2.22), ta suy raf x/là hàm tăng thực trênQC:Bây giờ, từ
(99)Tạp chí Epsilon, Số 05, 10/2015 với mọin2N:Ta chứng minh
f x/x (2.24) với x 1; x QC: Thật vậy, giả sử có x0 cho f x0/ < x0: Khi đó, đặt
f x0/Dx0 "với0 < " < x0:Sử dụng quy nạp kết hợp với (2.21), ta có f x0n/fn.x0/D.x0 "/n
với mọin2N:Mặt khác, dof tăng nên ta có
f x0n/f bx n 0c
bx0nc x n 1: Do đó, kết hợp với bất đẳng thức trên, ta thu
xn0 1.x0 "/n
với mọin 2N:Nhưng, điều rõ ràng bất đẳng thức sai vớinđủ lớn Mâu thuẫn
này cho ta điều vừa khẳng định, tức (2.24) Tiếp theo, ta chứng minh
f x/Dx (2.25) với mọix 1; x 2QC:Chọnn2 Nsao choanxC1:Khi đó, ta có
anf an/Df an x/Cx f an x/Cf x/.an x/Cx Dan: Do vậy, dấu đẳng thức đánh giá phải xảy (2.25)
ThayxD p
q vớip; q 2N
; p; q/D1vày Dq vào (2.21), ta suy ra
f x/x
với mọix 2QC:Từ (2.22) sử dụng phương pháp quy nạp, ta có
f nx/nf x/
với mọix 2QCvàn2N:Từ đó, chox D
p
q vớip; q 2N
; p; q/D1vànDq;ta được
f x/x
với mọix 2QC:Vậyf x/Dxvới mọix 2QC:Ta thu kết cần chứng minh
Bài tốn 13 Tìm tất hàm sốf WR!Rthỏa mãn đồng thời điều kiện:
(1) f xC1/f x/C1với mọix 2RI (2.26) (2) f xy/f x/f y/với mọix; y2 R: (2.27)
Lời giải Thay x D y vào (2.27), ta đượcf x2/ f2.x/ 0: Từ suy raf x/ với x 0:Do đó, cách thayx D 0vào (2.26), ta có f 1/ f 0/C1 1:Tiếp tục, thayx Dy D1vào (2.26), ta cóf 1/ f2.1/;tức0f 1/ 1:Kết hợp với trên, ta f 1/D1và đóf 0/ D0:Từ (2.26) (2.27), quy nạp ta có
(100)với mọix 2R; n2N;và
f xn/fn.x/ (2.29)
với mọix 0vàn2N:Sử dụng (2.28), với mỗix > 0;ta có
f x/Df fxg C bxc
f fxg
C bxc bxc> x 1:
Từ đây, kết hợp với (2.29) (2.27), với mỗix > 1và với mỗin2N;ta thấy
.xn 1/fn 1
x
f xn/f 1
xn
f 1/D1:
Do vậy, ta có
f 1
x
pn xn 1
với mọix > vàn N:Trong bất đẳng thức trên, cố địnhx > 1và chon ! C1;ta thu
đượcf x
x;hay nói cách khác (chú ý rằngf 0/ D0vàf 1/D1),
f x/x (2.30) với mọix 2Œ0; 1:Mặt khác, (2.26) nên ta chứng minh quy nạp
f x n/f x/ n với mọin2N:Với mỗix < 0;ta cóbxc< 0nên
f x/Df fxg C bxc
f fxg
C bxc fxg C bxc Dx: (2.31) Trong (2.27), ta thayx Dy Dt vớit 2Œ 1; 0thì có
f2.t /f t2/t2
nên suy rat f t / t:Kết hợp với kết trên, ta thu đượcf t / Dt với mọit 2Œ 1; 0: Bây giờ, với mỗix < 1;sử dụng kết trên, ta có
1 x2f
2
.x/Df2.x/f2 1
x
f x2/f 1
x2
f 1/D1
nênf2.x/x2;suy rax f x/ x:Kết hợp kết với (2.31), ta đượcf x/Dx với mọix 0:Tiếp tục, thayy D 1vào (2.27) xétx > 0thì có
xDf x/f x/f 1/D f x/;
suy raf x/ x với x > 0:Kết hợp kết với (2.30), ta đượcf x/ D x với x 2.0; 1/:Cuối cùng, xétx > 1;ta có
1
xf x/Df x/f
1 x
f 1/D1
(101)Tạp chí Epsilon, Số 05, 10/2015
Bài tốn 14 (MOSP, 2007) Tìm tất hàm sốf WR!Rthỏa mãnf 1/D1và
f
yf x/C x y
Dxyf x2Cy2/ (2.32)
vi mi cp s thcx; y y Ô0/:
Lời giải Thay x D 0vào (2.32), ta đượcf yf 0/ D 0:Nu f 0/ Ô thỡ bng cách chọn y D
f 0/;ta đượcf 1/D0;mâu thuẫn với giả thiết Do đó, ta phải cóf 0/D0:
Bõy gi, gi s rng tn tia1 Ô0sao chof a1/ D0:Thayx D a1 vày D 1vào (2.32), ta đượcf a21C1/D0:Từ đó, ta xây dựng dãy.an/vớianC1 Dan2C1thỏa mãn
f an/D0
với mọin2N:ThayxD1vào (2.32), ta
f
yC y
Dyf y2C1/
vi miy Ô0:Tip tc, thayxbixC
x vào (2.32) sử dụng kết trên, ta có f
xyf x2C1/C x C1 xy D y.x C1/ x f
.x2C1/2Cx2y2 x2
vi mix; y Ô0:Thayybi y
x vo phương trình trên, ta f
yf x2C1/C x C1 y D y.x C1/ x2 f
.x2
C1/2Cy2 x2
với mix; y Ô0:Mt khỏc, s dung (2.32), ta cng cú
f
yf x2C1/C x 2C1
y
Dy.x2C1/f x2C1/2Cy2 :
Kết hợp với đẳng thức trên, ta y.x2C1/
x2 f
.x2C1/2Cy2 x2
Dy.x2C1/f x2C1/2Cy2 ;
hay
f
.x2C1/2Cy2 x2
Dx2f x2C1/2Cy2
(102)nên tồn tạim2Nsao choam > b:Bây giờ, với ý rằngb > > b
am C1
;ta chọn x2D b
am vày
Db b am C1
2 để có ˆ <
ˆ :
.x2C1/2Cy2 Db x2C1/2Cy2
x2 Dam Thay số vào (2.33), ta thu đượcf am/D b
amf b/;hayf b/D0;mâu thuẫn Và thế, ta chứng minh đượcf x/D0khi khix D0:
Bõy gi, vi mix Ô0;ta cú th chọny choyf x/C x y Dx
2
Cy2(đây phương trình bậc ba ẩny nên ln có nghiệm thực) Thay vào (2.32), ta
.1 xy/f x2Cy2/D0:
Tuy nhiên, theo chứng minh trờn thỡf x2Cy2/Ô0:Do ú, ta phi cúy D
x:Suy
xf x/Cx
Dx2C x2; từ ta dễ dàng tìm đượcf x/D
x vi mix Ô0:Ta i n kt lun:
f x/D ˆ <
ˆ :
0 nux D0
x nux Ô0 Hm ny tha mãn yêu cầu toán
3 Một số bất phương trình hàm xây dựng các tập rời rạc
Bài toán 15 Với hàmg W ZC ! ZC; g.1/ D 1cho trước, chứng minh tồn tại
hàm sốf WZC!ZCthoả mãn
f n/ > g.n/
với mọin2 ZC;và
f mn/Df m/f n/
với cặp số nguyên dương nguyên tố nhaum; n:
Lời giải Ta xây dựng hàmf thoả mãn điều kiện Xét dãy faigi2ZC D.2; 3; 4; 5; 7; 8; 9; 11; : : :/
là dãy tăng luỹ thừa số nguyên tố Các số dãy có dạngp˛j
i xếp theo thứ tự tăng dần Ta có nhận xét rằng, hàmf WZC !ZCcó tính chấtf mn/Df m/f n/
(103)Tạp chí Epsilon, Số 05, 10/2015 Chọnf a1/ > g.a1/:
Giả sử sốf a1/; f a2/; : : : ; f ak 1/đã xác lập Xét tất số dạngg.s/; vớis tích số khác từ tậpfa1; a2; : : : ; akg:Ta chọn số bất kỳf ak/lớn tất số dạngg.s/:
Ta chọn dãy giá trịf ai/và thêm điều kiện nhân tính.f mn/Df m/f n//sẽ xác lập nên hàmf WZC!ZC:Hàmf thiết lập thoả mãn yêu cầu đề
Thật vậy, với mọin2ZC;ta biểu diễnnDai1ai2 aip vớii1< i2< < ip:Do f n/Df ai1ai2 aip/Df ai1/f ai2/ f aip/f aip/ > g.ai1ai2 aip/Dg.n/: Bài toán chứng minh xong
Bài toán 16 (Trung Quốc, 1993) Cho hàm sốf WRC !RCthoả mãn
f xy/f x/f y/ (3.1)
với mọix; y > 0:Chứng minh với mọix > 0vàn2ZCthì
f xn/f x/pf x2/p3
f x3/ pn
f xn/: (3.2)
Lời giải Ta chứng minh quy nạp theon:Dễ thấy khẳng định vớinD1:Giả sử khẳng định (3.2) với số1; 2; : : : ; n:Khi đó, ta có
f xk/f x/pf x2/p3
f x3/ qk f xk/ vớikD1; 2; : : : ; n:Nhânnbất phương trình lại, ta thu
fn.x/f n21.x2/ f
n.xn/f x/f x2/ f xn/: Nhân hai vế bất phương trình vớif x/f x2/ f xn/;ta
fnC1.x/f nC21.x2/ f nC1
n xn/f2.x/f2.x2/ f2.xn/: Mặt khác, sử dụng (3.1), ta lại có
VPD n Y
iD1
f xi/f xnC1 i/
fn.xnC1/:
Kết hợp với đánh giá trên, ta suy
fnC1.x/f nC21.x2/ f nC1
n xn/fn.xnC1/:
Nhân hai vế bất phương trình cuối chof xnC1/và lấy bậcnC1hai vế, ta suy khẳng định (3.2) vớinC1:Theo nguyên lý quy nạp, ta có khẳng định với mọin: Nhận xét Bằng cách đặtanDlnf xn/;ta phát biểu toán lại dạng dãy số sau: Choa1; a2; : : :là dãy số thực thoả mãnaiCj Caj với mọii; j D1; 2; : : : Chứng minh với mọin2 ZCthì
a1Ca2
2 C C an
(104)Bài tốn 17 Tìm tất sốa > 0sao cho tồn sốK > 0và hàmf WR!Rthoả
mãn
f x/Cf y/ f
x Cy
2
CKjx yja
với cặp số thựcx; y: Lời giải Từ giả thiết, ta suy
f xC2y
Cf y/ f
xC3y
C K
2ajx yj a;
f x/Cf xC2y f
3x Cy
C K
2ajx yj a
; f 3x4CyCf xC43y
2 f x
Cy
C K
2ajx yj a
:
(3.3)
(3.4)
(3.5) Lấy (3.3)C(3.4)C2(3.5);ta
f x/Cf y/ f
x Cy
2
C K
2a 2jx yj a
:
Từ đây, cách lặp lại quy trình trên, ta chứng minh quy nạp f x/Cf y/
2 f
xCy
C K
2n.a 2/jx yj a
với 2N:Nua < 2thỡ bng cỏch choxÔy vn! C1;ta thu điều mâu thuẫn
Do đóa2:Ta chứng minh tập hợp tất số thựcathoả mãn yêu cầu đề Thật vậy, vớia2;xétf x/D jxjavàK D
2a;ta chứng minh bất đẳng thức đầu thoả mãn với cặp số thựcx; y;tức
jxja C jyja
ˇ ˇ ˇ ˇ
xCy ˇ ˇ ˇ ˇ a
Cˇˇ ˇ x y ˇ ˇ ˇ a : (3.6)
Đặtf x; y/DVT(3.6) VP(3.6);ta có
f x; y/Df x; y/Df x; y/Df x; y/:
Do đóf x; y/là hàm chẵn vớixvà vớiy:Vì vậy, khơng tính tổng qt, ta giả sử x 0vày 0:Khi đó, bất đẳng thức (3.6) viết lại dạng
xaCya
x Cy
2 a
Cˇˇ ˇ x y ˇ ˇ ˇ a :
VớixCy D0;ta cóxDy D0nên bất đẳng thức hiển nhiên
Xét trường hợpxCy > 0:Khi đó, bất đẳng thức có dạng đối vớix; y nên ta chuẩn hốxCy D2:Theo đó, ta phải chứng minh
xaCya
(105)Tạp chí Epsilon, Số 05, 10/2015
Do0ˇˇ ˇ
x y
ˇ ˇ ˇ
xCy
2 D1nên 1C
ˇ ˇ ˇ
x y
ˇ ˇ ˇ a
1C x y/ D
x2Cy2 : Mặt khác, theo bất đẳng thức trung bình luỹ thừa, ta lại có
xaCya
x2Cy2
a2 x
2Cy2 : Do (3.7) chứng minh Bài toán giải xong
4 Một số tập tự luyện
Để kết lại viết này, xin nêu thêm số toán để bạn đọc tự nghiên cứu thêm Bài tập 4.1 (Bulgaria, 2008) Chof WRC !RClà hàm số thoả mãn
2f x2/xf x/Cx với mọix > 0:Chứng minh rằngf x3/x2với mọix > 0:
Bài tập 4.2 (IMO Shortlist, 2005) Tìm tất hàm sốf WRC !RCthoả mãn
f x/f y/D2f xCyf x/ với cặp số dươngx; y:
Bài tập 4.3 (KHTN, 2010) Tìm tất hàm sốf WRC!RCthoả mãn
f xC2y/ f x y/D3f y/C2pf x/f y/ với cặp số dươngx > y:
Bài tập 4.4 Tìm tất hàmf WRC !RCthoả mãn
ff f x/
Cf f x/
D2xC5
với số thực dươngx:
Bài tập 4.5 (Brazil, 2012) Tìm tất tồn ánhf WRC!RCthoả mãn
2xf f x/
Df x/hxCf f x/i
với số thực dươngx:
Bài tập 4.6 (Việt Nam TST, 2007) Tìm tất hàm liên tụcf WR!Rthoả mãn
f x/Df
x2C x C
1
(106)
Bài tập 4.7 Tìm tất hàm liên tụcf WR!Rthoả mãn
f 2xC1/Df x/ với số thựcx:
Bài tập 4.8 Tìm tất hàm liên tụcf WR!Rthoả mãn
f xCy f y/
Df x/Cf y f y/ với cặp số thựcx; y:
Bài tập 4.9 (Hà Nội, 2013) Tìm tất hàm liên tụcf WRC !RCthoả mãn đồng thời
điều kiện sau
(1) f 2x/D2f x/với mọix > 0I
(2) ff3.x/ ef x/
Dx2.ex 1/f x/với mọix > 0I
(3) f e 1/D.e 1/f 1/I
(4) f k/2Nvới mọik2N:
Bài tập 4.10 (Turkey, 2013) Tìm tất hàmf W R ! RC thoả mãn đồng thời điều
kiện
(1) f x2/Df2.x/ 2xf x/với mọix2 RI (2) f x/Df x 1/với mọix 2RI (3) f tăng thực trên.1;C1/:
Bài tập 4.11 (THTT, 2002) Tìm tất hàmf WR!Rthoả mãn
f x f y/
Df xCy2002/Cf f y/Cy2002 C1 với cặp số thựcx; y:
Bài tập 4.12 Tìm tất hàmf WR!Rthoả mãn
f x2/C4y2f y/ D
f xCy/Cy2
f x y/Cf y/ với cặp số thựcx; y:
Bài tập 4.13 (Tổng quát IMO 1992) Với số tự nhiênn2cho trước, tìm tất hàm sốf WR!Rthoả mãn đẳng thức
f xnCf y/DyCfn.x/ với cặp số thựcx; y:
Bài tập 4.14 (Saudi Arabia, 2014) Tìm tất hàmf WN!Nthoả mãn
f nC1/ > f n/Cf f n/
(107)Tạp chí Epsilon, Số 05, 10/2015
Bài tập 4.15 (Romania, 2004) Tìm tất đơn ánhf WN !Nthoả mãn
f f n/
nCf n/ với số nguyên dươngn:
Bài tập 4.16 (IMAR, 2009) Chứng minh với hàm sốf W RC ! RC tồn
nhất cặp số dươngx; ysao cho
f x Cy/ < yf f x/ : Bài tập 4.17 (IMO, 2011) Cho hàm sốf WR!Rthoả mãn
f xCy/yf x/Cf f x/ với mọix; y 2R:Chứng minh rằngf x/D0với mọix 0:
Bài tập 4.18 (IMO Shortlist, 2009) Chứng minh không tồn hàmf WR!Rthoả mãn
f x f y/
yf x/Cx với cặp số thựcx; y:
Bài tập 4.19 Cho hàm sốf WŒ0; 1 !Œ0; C1/thoả mãn f x/Cf y/
2 f x
Cy
C1
với mọix; y 2Œ0; 1:Chứng minh với mọia; b; c Œ0; 1; a < b < cthì c b
c af a/C b a
c af c/f b/C2:
Bài tập 4.20 Cho hàm sốf; gWŒ0; 1!Œ0; 1;trong đóf tăng nghiêm ngặt, thoả mãn f g.x/Dx
với mọix 2Œ0; 1:Chứng minh với mọin2Nthì
n X
kD1
f k
n
Cg k
n
< n n:
Tài liệu tham khảo
[1] Nguyễn Văn Mậu (chủ biên),Một số chuyên đề toán chọn lọc bồi dưỡng học sinh giỏi, Khoá bồi dưỡng giáo viên chuyên toán THPT, 2004
[2] Nguyễn Văn Mậu,Phương trình hàm với đối số biến đổi, NXB Đại Học Quốc Gia Hà Nội, 2015
[3] Nguyễn Trọng Tuấn,Các toán hàm số qua kỳ thi Olympic, NXB Giáo Dục, 2004 [4] Christopher G Small,Functional Equations and How to Solve Them, Springer, 2007 [5] An Introduction to the Theory of Functional Equations and Inequalities, Birkhauser, 2009 [6] Titu Andreescu, Iurie Boreico, Oleg Mushkarov, Nicolai Nikolov,Topics in Functional
(108)(109)GIẢI TÍCH VÀ CÁC BÀI TỐN CỰC TRỊ
Trần Nam Dũng - Đại học Khoa học Tự nhiên, ĐHQG - TP.HCM
Since the building of the universe is perfect and is created by the wisdom creator, nothing arises in the universe in which one cannot see the sense of some maximum or minimum.-Leonard Euler
Trong tốn trường phổ thơng, toán cực trị thuộc vào dạng toán gần với ứng dụng thực tế Những yêu cầu đường ngắn nhất, đường nhanh nhất, góc nhìn lớn nhất, tổng thời gian chờ đợi nhất, tổng chi phí nhất, tổng lợi nhuận cao nhất, diện tích lớn yêu cầu tự nhiên xuất phát từ toán sản xuất, đời sống khoa học Chính tốn cực trị cần có chỗ đứng xứng đáng chương trình tốn phổ thơng, phương pháp giải toán cực trị cần phải trình bày cách
Trên phương diện phương pháp, có hai cách tiếp cận cho lời giải tốn cực trị, phương pháp sử dụng bất đẳng thức phương pháp hàm số Với phương pháp bất đẳng thức, sơ đồ là: Để chứng minhM giá trị lớn hàm sốf x/trên miềnD.x vector), ta chứng minh :
.i / f x/ 6M với mọixthuộcD:
.i i / Tồn tạix0thuộcDsao chof x0/DM:
Phương pháp hàm số khảo sát hàmf x/trênDvà dựa vào định lý giải tích để tìm điểm cực trị giá trịM:
Chú ý rằng, chương trình phổ thơng khái niệm hàm nhiều biến chưa đề cập, cho nên, bắt gặp toán nhiều biến công cụ chủ yếu công cụ đạo hàm hàm số biến
Trong viết này, chủ yếu đề cập đến phương pháp giải tích để giải tốn cực trị Chúng ta bắt đầu toán cực trị hàm biến giải nguyên lý Fermat định lý tồn Weierstrass Sau chuyển sang toán cực trị nhiều biến giải phương pháp khử dần biến để đưa trường hợp biến Tiếp đến tốn cực trị có điều kiện
Trong phần cuối cùng, đề cập đến cách tiếp cận toán cực trị nhiều biến sử dụng cơng cụ tốn cao cấp (đạo hàm riêng theo biến, phương pháp nhân tử Lagrange) Đây phần dành cho giáo viên học sinh lớp chun để có nhìn tổng
quan mạch lạc vấn đề cực trị hàm nhiều biến Góc nhìn giúp học sinh
sau lên bậc học cao không bị làm theo qn tính, giải tốn cực trị nhiều biến công cụ thô sơ (và địi hỏi nhiều sáng tạo)
(110)bài lấy từ toán định lý kinh điển, toán thi Olympic, đề thi đại học Ngồi ví dụ có lời giải bình luận chi tiết, chúng tơi đưa số tập tự giải dành cho bạn đọc
1 Nguyên lý Fermat
Phương pháp có tên nguyên lý Fermat phương pháp mà biết đến:Nếu hàm số
f là khả vi điểm cực tiểu (cực đại) địa phương điểm dừng, tức là nghiệm phương trìnhf0.x/D0:Về điều trước Fermat nhắc tới:
“Về hai phía điểm có giá trị lớn giảm ban đầu không đáng kể.” -(Johan Kepler)
Điều ngược lại khơng đúng: Điểm dừng khơng phải điểm cực trị hàm số, ví dụ đơn giản sau: Hàmx3 điểmx D0:Để tìm giá trị cực trị hàm sốf;ta giải phương trìnhf0.x/D0;tìm tất điểm dừng điểm “nghi can” cho giá trị cực trị Sau ta sử dụng định lý tồn tại: Một hàm số liên tục đoạnŒa; bsẽ đạt giá trị lớn giá trị nhỏ Ta coi định lý hiển nhiên mặt hình học bỏ qua phép chứng minh Như giá trị lớn giá trị nhỏ hàm liên tục, khả vi đoạnŒa; btồn ta cần tìm giá trị điểm dừng hai đầu mút
Bổ đề 1.1 Nếu hàm sốf WR !Rliên tục vàf x/! C1khijxj ! C1thì đạt được
giá trị nhỏ trongRn:
Bổ đề 1.2 Nếu hàm sốf W.a; b/!Rliên tục vàf x/ ! C1khix !avàx !bthì nó
đạt giá trị nhỏ trên.a; b/:
Điều kiện tồn giá trị lớn phát biểu tương tự Chúng ta bắt đầu ví dụ kinh điển
Ví dụ 1.1 (Định luật Snellius) Tia sáng cắt đường biên hai mơi trường, vào với góc˛và ra với gócˇ(góc tia sáng với đường vng góc với đường biên điểm cắt) Khi đó
sin˛ va D
sinˇ vb ;
trong đóva; vblà vận tốc ánh sáng mơi trường đó.
Lời giải Ta sử dụng nguyên lý Fermat quang học: Ánh sáng đường từ điểm đến điểm chọn đường ngắn mặt thời gian Nếu ta lấy tia sáng hai điểmA; B nằm hai phía đường biên, cịn đường biên ký hiệu làl ta thu tốn tìm cực tiểu:
8 <
:
f M /D AM va C
BM
vb !min M 2l
(111)Tạp chí Epsilon, Số 05, 10/2015
Gọi AH; BK đường vng góc hạ từA; B tương ứng xuống ĐặtAH D a; BK D b HK Dc; HM Dxthì ta có
AM Dpa2Cx2; BH D q
b2C.c x/2: Từ
f M /Dg.x/D p
a2Cx2 va C
q
b2C.c x/2 vb :
Theo bổ đề1thì f M /đạt giá trị nhỏ điểmM0 Và theo nguyên lý Fermat thìf0.M0/D0:Nhưng
f0.M /Dg0.x/D x vapa2Cx2
c x vb
q
b2C.c x/2 :
Từ đóf0.M0/D0tương đương với x0 vapa2Cx2
c x0 vb
q
b2C.c x/2 ;
hay
sin˛ va D
sinˇ vb : Ta có điều phải chứng minh
Bài tốn Có miếng thép kích thước1m1m:Người ta muốn làm từ thép hình hộp khơng đáy cách cắt ở4góc hình vng kích thướcxx;gấp lên hàn lại Hỏi phải chọnxbằng để thể tích hình hộp lớn ?
Lời giải Rõ ràng ta phải có0x
2:Thể tích hình hộp V x/Dx.1 2x/2:
Với này, cần chút khéo léo ta dùng bất đẳng thức AM-GM để tìm giá trị lớn Tuy nhiên, phương pháp hàm số cho lời giải tự nhiên mà khơng địi hỏi sáng tạo đặc biệt nào:
V0.x/D12x2 8xC1D.2x 1/.6x 1/:
Từ ta cóV x/chỉ đạt giá trị lớn giá trị nhỏ điểm0(biên), (điểm dừng),
2 (biên) Vì
V 0/DV
1
D0; V
1
D
27;
nên ta suy raVmaxD
2
(112)Bài tập 1.1 Hãy giải tốn với miếng thép có kích thướcab phương pháp sau :
.a/ Dùng bất đẳng thức AM-GM .b/ Dùng đạo hàm
Ví dụ 1.2 Qua điểm nằm góc cho trước, kẻ đoạn thẳng có độ dài ngắn nhất có đầu mút nằm cạnh góc.
Lời giải Bổ đề1đảm bảo tồn đoạn thẳng ngắn Giả sử đoạn thẳng ngắn AB điểm nằm góc làM: QuaM ta kẻ đường thẳng khác A0B0:Gọi góc có hướng giữaA0B0vàAB:Hàm sốf / D A0B0 đạt giá trị nhỏ điểm D 0do f0.0/D0:Đặt˛ D∠OAB; ˇD∠OBAtrong đóO đỉnh góc Sử dụng định lý hàm số sin cho tam giácMAA0 vàMBB0;ta có
MA0 DMA sin˛
sin.˛ /; MB
0DMB sinˇ
sin.ˇC /: Từ
f DA0B0 AB DMA0CMB0 MA MB DMA
sin˛
sin.˛ /
CMB
sinˇ
sin.ˇC /
DMA 2sin
2 cos ˛
sin.˛ / MB
2sin2 cos ˛C 2 sin.˛C / : Như
f D
2sin2
"
MA cos ˛
sin.˛ / MB
cos ˛C
sin.˛C / #
:
Cho !0;ta
f0.0/DMAcot˛ MBcotˇ:
Nhưng vìf0.0/D0nên ta cóMAcot˛ DMBcotˇ:Kết có ý nghĩa hình học nào? Hạ đường vng gócOH xuốngAB:Dễ dàng kiểm tra HB
HA D cotˇ
cot˛:Mặt khác MA
MB D cotˇ
cot˛;suy
MADHB; MB DHA:
Như đoạn thẳng ngắn nhấtAB đặc trưng tính chất sau:Hình chiếu củaO lênAB
đối xứng vớiM qua trung điểm củaAB:
Nhận xét Tại tìm đặc trưng hình học đoạn thẳngABmà khơng nêu cách dựng ? Vấn đề với vị trí tổng quát, lời giải dựng thước compa Trong thực tế, có nhiều tốn cực trị ta đưa tính chất đặc trưng lời giải khơng tìm lời giải mang tính xây dựng
Bài tập 1.2 Đường thẳng qua điểm nằm góc, cắt góc thành tam giác có diện tích nhỏ Hãy tìm lời giải hình học lời giải giải tích cho tốn
(113)Tạp chí Epsilon, Số 05, 10/2015
Bài tập 1.4 Qua điểm nằm góc vng kẻ đường thẳng choOACOB nhỏ nhất.O đỉnh góc vng vàA; Blà giao điểm đường thẳng với cạnh góc vng) Với tốn tìm giá trị lớn giá trị nhỏ hàm số đoạnŒa; bta có số trường hợp đặc biệt đơn giản hiệu sau :
Hàm đơn điệu:Nếu f0.x/ > 0với mọix thuộc.a; b/thì hàm sốf tăng trênŒa; bvà ta có f a/6f x/6f b/với mọixthuộcŒa; b:
Hàm lồi:Nếuf00.x/60thì hàm sốf x/sẽ có nhiều điểm cực đại (nếu có điểm mà hàm số đạt giá trị lớn nhất) giá trị nhỏ hàm số đạt hai điểm biên
Ví dụ 1.3 (Đề thi Đại học khối A – 2008) Tìm tất giá trị tham số thựcmsao cho
phương trình p
2xCp4 2xC2p6 xC2p4 xDm;
có đúng2nghiệm thực phân biệt.
Lời giải Hàm số
f x/Dp2xCp4 2xC2p6 xC2p4 x;
xác định trênŒ0; 6là tổng hàm lồi nên hàm lồi Vì thếf x/sẽ có nhiều điểm cực đại Tính đạo hàm bậc nhất, ta
f0.x/D p1 2x C
1 2p4 x3
1 p
6 x
1 2p4 x:
Dễ thấyf0.2/D0;suy ra2là điểm cực đại Hàm số có chiều biến thiên tăng 0; 2/và giảm trên.2; 6/:Từ dễ dàng suy phương trìnhf x/D mcó hai nghiệm phân biệt
maxff 0/; f 6/g m < f 2/: Tức là2 p6Cp4
6m < 2Cp2 :
Bài tập 1.5 (Olympic 30 - - 1996)Tìm giá trị lớn hàm số f x/Dx9p1Cx2C13p1 x2;
trên đoạnŒ0; 1:
Bài tập 1.6 (Bài tốn góc sút khung thành)Cho đường thẳnglvà hai điểmA; Bnằm phía đối vớil:Tìm vị trí điểmM trênlsao cho góc∠AMB lớn
Bài tập 1.7 Tìm giá trị lớn hàm số f x/Dp2sinxC
q
15 10p2cosx:
(114)Ví dụ 1.4 Nhà địa chất địa điểmAtrong sa mạc, cách đường đất10km.AB D10
km, vớiB là điểm đường đất gầnAnhất) Ông cần điểmC;nằm con đường đất cáchB 50km Biết nhà địa chất di chuyển sa mạc với vận tốc30
km/h đường đất với vận tốc50km/h Hãy tìm phương án để nhà địa chất đếnC
sau thời gian nhất.
A
B D C
Ta nhận xét rằng, di chuyển với vận tốc khơng đổi đường với thời gian đường với quãng đường ngắn Do đó, đường ngắn có dạngADDC vớiD điểm trênBC:
Đặtx DBD;ta dễ dàng tính thời gian quãng đườngADDC f x/D
p
100Cx2 30 C
50 x 50 : Từ ta có ba cách tiếp cận sau :
Cách (Dùng đạo hàm)Ta có f0.x/D x
30p100Cx2 50 D
1 150
5x 3p100Cx2:
Giải phương trìnhf x/D0ta dễ dàng tìm nghiệmx D 15
2 suy fmin Df
15
2
D 19 15:
Vậy thời gian để nhà địa chất vềC 19 15: Cách Ta láu cá chút Vì biếtfminD
19
15 nên ta chứng minh ln điều Ta có
bất đẳng thức p
100Cx2 30 C
50 x 50
19 15; tương đương với
5p100Cx2C150 3x 190; hay
25.100Cx2/9x2C240xC1600;
.2x 15/20: Do ta cóf x/> 19
15;dấu xảy khix D 15
(115)Tạp chí Epsilon, Số 05, 10/2015
Cách Ta lại dùng “điểm rơi” x D 15
2 để đánh giá hàm f x/ bất đẳng thức Cauchy-Schwarz
f x/D p
100Cx2 30 C
50 x 50
D p
100Cx2 q
100C 152
30 q
100C 152
C 50 x 50
100C 15
2 x 30 252
C50 x 50 D
19 15: Từ dẫn đến kết luận toán
Bài tập 1.9 Nếu trướcfminD
19
15 tìm cách tiếp cận để giải tốn theo cách2: Bài tập 1.10 Nếu ta điểm rơi x D 15
2 ; làm để áp dụng bất đẳng thức Cauchy-Schwarz theo cách3?
2 Cực trị hàm nhiều biến
Với công cụ cấp trung học phổ thông, phương pháp giải toán nhiều biến số làm giảm dần biến số cách tìm cực trị theo phương Ý tưởng phương pháp minh hoạ hình ảnh sau:Để tìm người cao nhóm người xếp thànhmhàng, ta tìm người cao hàng so sánh người cao để tìm người cao tuyệt đối.
Ta bắt đầu ví dụ kinh điển hình học
Ví dụ 2.1 Trong tam giác nội tiếp đường trịn cho trước, tìm tam giác có diện tích lớn nhất.
Lời giải Khơng tính tổng quát, ta xét tam giácABC nội tiếp đường tròn đơn vị với A.0I 1/cố định vàB.x1; y1/; C.x2; y2/với điều kiệnx12Cy
2
1 D1; x 2Cy
2
2 D1thì toán cực trị sau
8 ˆ ˆ ˆ <
ˆ ˆ ˆ : ˇ
ˇ.x1 1/y2 x2 1/y1ˇˇ
2 !max x12Cy12D1
x22Cy22D1
Bài toán cực trị có điều kiện4biến chuyển thành toán cực trị2biến cách tham số hoá đường tròn đơn vị, cụ thể đặtx1 Dcos˛; y1 D sin˛; x2 D cosˇ; y2 D sinˇta quy toán việc tìm giá trị lớn giá trị nhỏ hàm số
f ˛; ˇ/Dsin˛ sinˇCsin.ˇ ˛/: Giữ˛cố định, xétf ˛; ˇ/như hàm số theoˇthì
(116)Từ ta tìm điểm dừng làˇ D ˛
2 Ck:Từ đó, để tìmf ˛; ˇ/maxta cần tìm giá trị lớn hàm
sin˛ sin ˛
2 Ck
Csin ˛
2 Ck ˛
;
tức giá trị lớn
f1.˛/Dsin˛ 2sin˛
; f2.˛/Dsin˛C2sin˛
:
Giải toán biến này, ta tìm đáp sốf ˛; ˇ/maxbằng
3p3
2 ;chẳng hạn khi˛ D
3 ; ˇ D
3 (vàf ˛; ˇ/minbằng
3p3
2 !) Đây tình tam giác cho Lời giải Cũng phương pháp tương tự, trước hết ta cố định cạnhBC dây cung độ dài2a đường tròn bán kínhR tìm vị trí điểmAtrên đường trịn cho diện tích tam giácABC lớn Có thể chứng minh dễ dàng điểmAcần tìm trung điểm cung lớnBC (nơi mà tiếp tuyến song song vớiBC /:
Diện tích tam giác cực đại
f a/DaRCpR2 a2:
Bây ta cần tìm giá trị lớn củaf a/trênŒ0; R:Tính đạo hàmf0.a/;ta f0.a/DRCpR2 a2 a
2 p
R2 a2;
vàf0.a/D0khi khiaD R p
3
2 ;từ ta tìm đượcfmaxD
3p3R2 :
Ví dụ 2.2 Cho tam giác đều ABC: Với điểm M nằm mặt phẳng tam giác, gọi
D; E; F lần lượt hình chiếu củaM lên đường thẳng.BC /; CA/; AB/:Tìm giá trị lớn nhất giá trị nhỏ biểu thức
MACMB CM C MDCMECMF :
Ví dụ 2.3 (Chọn đội tuyển Việt Nam 2001) Chox; y; zlà số thực dương thoả mãn
2xC4yC7z D2xyz:
Tìm giá trị nhỏ biểu thứcxCyCz:
Lời giải Rútz D 2xC4y
2xy > 0ta đưa toán toán cực trị hai biến
<
:
f x; y/DxCy C2xC4y
(117)Tạp chí Epsilon, Số 05, 10/2015 Tính đạo hàm theoy;ta
f0y.x; y/ D1C4.2xy 7/ 2x.2xC4y/ 2xy 7/2 D1
4x2C28 2xy 7/2: Từ đó, ta tìm được, với mỗixcó định thìf x; y/đạt giá trị nhỏ điểm
y0 D 2x C
r 1C
x2: Khi
f x; y0/DxC 11 2x C2
r 1C
x2 Dg.x/: Tính đạo hàm
g0.x/D1 11 2x2
14 q
1C x72
x3:
Phương trìnhg0.x/D0tương đương với.2x2 11/2.x2C7/D784(với điều kiện2x2 > 11/ có nghiệmxD3:Đây điểm cực tiểu (dof ! C1khix!0vàx! C1/:Từ
f x; y/minDg.x/minDg.3/D 15 :
Vậy giá trị nhỏ cần tìm 15 :
Nhận xét Ngoài thủ thuật tham số hoá, thay khử dần biến số ví dụ nêu trên, cịn làm giảm số biến số hàm số cách sử dụng tính chất bất biến hàm, ví dụ tính (khơng đổi phép co dãn), tính đối xứng (khơng đổi với chuyển vị, hốn vị)
Ví dụ 2.4 (Đề thi Đại học khối B, 2008) Chox; ylà số thực thoả mãn điều kiệnx2Cy2 D1:
Tìm giá trị lớn giá trị nhỏ biểu thức
2.x2C6xy/ 1C2xyC2y2:
Lời giải Bài tốn quy toán cực trị hàm biến cách tham số hoá lượng giác quen thuộcx Dcost; y Dsint:Tuy nhiên, ta cịn có cách tiếp cận khác: Thay số1ở mẫu số bằngx2Cy2D1để thu biểu thức nhất, tức
2.x2C6xy/ x2Cy2C2xyC2y2
sau dựa vào tính để giảm số biến số hàm số Trước hết ta cần hiểu lại có đẳng thức
min x2Cy2D1
2.x2C6xy/
1C2xyC2y2 D.x;y/min
2R2n.0;0/
(118)Rõ ràng ta có
min x2Cy2D1
2.x2C6xy/
1C2xyC2y2 Dx2Cminy2D1
2.x2C6xy/ x2Cy2C2xyC2y2:
Nhưng hàm sốf x; y/D 2.x
2C6xy/
x2Cy2C2xyC2y2 hàm (bậc0;tức làf tx; ty/D f x; y//nên ta có
min x;y/2R2n.0;0/
2.x2C6xy/
x2Cy2C2xyC2y2 Dx2Cminy2D1
2.x2C6xy/ x2Cy2C2xyC2y2;
(từ điểm khác.0; 0/đều co dãn về1điểm nằm đường trịn đơn vị) Bây ta cần tìm giá trị lớn giá trị nhỏ hàm số
f x; y/D 2.x
C6xy/ x2Cy2C2xyC2y2;
vi.x; y/thucR2n.0; 0/:
Nuy D0thỡf x; y/D2:Viy Ô0;ta tt D x y
f x; y/D 2.t
2C6t /
t2C2t C3 Dg.t /: Ta có
g0.t /D 4t C12/.t
2C2tC3/ .2tC2/.2t2C12t / t2C2t C3/2 D
4.2t C3/.3 t / t2C2tC3/2 :
Từ tìm đượcgmin Dg
3
D 6; gmaxDg.3/D3:Chú ý giá trị vô bằng2:
Bài tập 2.1 (Việt Nam MO 2004) Cho x; y; z số thực dương thoả mãn điều kiện xCyCz/3 D32xyz:Tìm giá trị lớn giá trị nhỏ biểu thức
P D x
Cy4Cz4 xCyCz/4:
Bài tập 2.2 (Theo Việt Nam MO 2003)Chof t /D t
2C2t 1
t2C1 :Tìm giá trị lớn giá trị nhỏ biểu thứcf x/f y/vớix; ylà số thực thoả mãn điều kiệnxCy D1: Bài tập 2.3 (Đề thi cao đẳng khối A, B, D năm 2008)Chox; y số thực thoả mãn điều kiệnx2Cy2 D2:Tìm giá trị lớn giá trị nhỏ biểu thức
(119)Tạp chí Epsilon, Số 05, 10/2015
Bài tập 2.4 (Saudi Arabia 2015)Chox; y; zlà số thực dương thỏa mãn điều kiện x CyCz/
x C y C z D10:
Tìm giá trị lớn giá trị nhỏ biểu thức P D.x2Cy2Cz2/
1 x2 C
1 y2 C
1 z2
:
Ví dụ 2.5 Cho ba số thựca; b; c đơi khác nhau, tìm giá trị nhỏ biểu thức
a2Cb2Cc2
1
.a b/2 C b c/2 C
1 c a/2
:
Lời giải Hàm số
f a; b; c/D a2Cb2Cc2
1 a b/2 C
1 b c/2 C
1 c a/2
;
thuần bậc0còn hàm số
g.a; b; c/D a b/2 C
1 b c/2 C
1 c a/2;
bất biến phép tịnh tiến:g.a; b; c/Dg.aCt; bCt; cCt /:Sử dụng tính chất ta giảm số biến số tốn tìm giá trị nhỏ
Khơng tính tổng qt (do tính đối xứng !), giả sửa > b > c:Đặta b Dx; b c Dy thìc aD x Cy/vàaDcCxCy; b DcCy:Ta có
f a; b; c/Dg.x; y; c/D
3c2C2.xC2y/cC.x Cy/2Cy2
1 x2 C
1 y2 C
1 xCy/2
:
Cố địnhx; ykhi đóg.x; y; c/đạt giá trị nhỏ khic Dc0 D xC2y
3 :Từ g.x; y; c/minDg.x; y; c0/D
3.x
CxyCy2/
1 x2 C
1 y2 C
1 x Cy/2
Dh.x; y/:
Do tính bậc0của hàm sốh.x; y/;ta cần tìm giá trị nhỏ củah.x; y/ với xCy D1;sau đó, sử dụng tính đối xứng củah.x; y/;ta biểu diễnh.x; y/như hàm theo t Dxy với ý0 < t
4 W
h.x; y/D
3.1 t /
1 2t t2 C1
D 2.1 t /
3t2 Dk.t /: Ta có
k0.t /D
(120)trên
0;1
nênkmin.t /Dk
1
D
2: Vậy giá trị nhỏ cần tìm
2 đạt được, chẳng hạn x D y D 2; c D
1
2; tức c D
2; b D0; aD 2:
Bài tập 2.5 (British MO 1986)Chox; y; zlà số thực thoả mãn điều kiệnxCyCz D0 vàx2Cy2Cz2 D6:Tìm giá trị lớn biểu thức
F Dx2y Cy2z Cz2x:
Bài tập 2.6 (Đề thi đại học khối D, 2008)Chox; y số thực dương thay đổi Tìm giá trị lớn biểu thức
.x y/.1 xy/ 1Cx/2.1Cy/2:
Bài tập 2.7 (Chọn đội tuyển Việt Nam 1993)Chox1; x2; x3; x4là số thực thoả mãn điều kiện
2 x Cx
2 2Cx
2 Cx
2
4 1:Tìm giá trị lớn giá trị nhỏ biểu thức AD.x1 2x2Cx3/2C.x2 2x3Cx4/2C.x2 2x1/2C.x3 2x4/2:
Bài tập 2.8 (Việt Nam MO 2002)Chox; y; zlà số thực thoả mãn điều kiệnx2Cy2Cz2 D9: Chứng minh
2.xCyCz/ xyz 610:
Bài tập 2.9 (Việt Nam MO 2008) Chox; y; z số thực không âm, đơi khác Chứng minh ta có bất đẳng thức
.xyCyz Czx/
1
.x y/2 C y z/2 C
1 z x/2
4:
Hỏi dấu xảy ?
3 Cực trị hàm nhiều biến góc nhìn Tốn cao cấp
Với tốn phổ thơng, kể tốn thi học sinh giỏi cấp, kiến thức giải tích biến đủ để xử lý Tuy nhiên, để có góc nhìn tổng qt hơn, để giới thiệu nét đẹp tốn cao cấp, giới thiệu với em cách sơ lược giải tích nhiều biến, phương pháp nhân tử Lagrange Đây phương pháp để sáng tạo toán mới, kiểm tra kết sơ cấp khác
Lưu ý, việc giới thiệu kiến thức cho đối tượng nào, mức độ nhằm mục đích là điều phải cân nhắc.
Nguyên lý Fermat mở rộng sang trường hợp nhiều chiều mà khơng có thay đổi đáng kể Để tìm giá trị cực trị hàmf x/;trong đóx D.x1; x2; : : : ; xn/2Rn;ta tìm điểm
dừng từ hệ phương trình
(121)Tạp chí Epsilon, Số 05, 10/2015 có số phương trình số ẩn số
Sau ta sử dụng định lý Weierstrass (định lý tồn tại), thay đoạnŒa; bbằng tập compactA– tập đóng bị chặn - khơng gianRn:Đóng có nghĩa chứa tất điểm giới hạn
Tương tự trường hợp chiều, ta gặp vấn đề tìm giá trị cực trị miền mở Bổ đề giúp xử lý số trường hợp
Bổ đề 3.1 Nếu hàm sốf WRn !Rliên tục vàf x/! C1khi
jxj D q
x12Cx22C Cx2
n! C1;
thì đạt giá trị nhỏ trongRn:
Ví dụ 3.1 Trong tam giác nội tiếp đường trịn cho trước, tìm tam giác có diện tích lớn nhất.
Lời giải Sử dụng định lý Weierstrass tồn giá trị lớn giá trị nhỏ hàm liên tục xác định compact, ta suy tam giácA0B0C0với diện tích lớn tồn Sử dụng lý luận hình học quen thuộc, ta suy tam giácA0B0C0này phải
Ví dụ 3.2 Cho a; b; c là số thực thỏa mãn điều kiện a > b > c; a 5; a Cb 8; aCbCc D10:Tìm giá trị lớn củaP Da2Cb2Cc2:
Lời giải Kết hợp điều kiện ta suy bộ.a; b; c/thỏa mãn điều kiện đề nằm tập đóng, bị chặn (một hình phẳng thuộc mặt phẳngaCbCc D10/:Suy tồn điểm a; b; c/sao choP lớn nhất, doaCb 68nênc >2:
Nếua < 5vàc < b đặtm D min.5 a; b c/;ta thay.a; b; c/bằng bộ.a0; b0; c0/với a0 DaCm; b0 Db m; c0 Dcthì ta có
a02Cb02Cc02D.aCm/2C.b m/2Cc2
Da2Cb2Cc2C2m2C2.a b/m > a2Cb2Cc2:
Điều mâu thuẫn Vậy ta phải cóaD5hoặcb Dc: NếuaD5thìbCc D5và
2.b2Cc2/D.bCc/2C.b c/2D25C.b c/2 D25C.5 2c/2:
DoaCb 68nênc >2:Suy ra065 2c 61:Từ suy ra2.b2Cc2/625C1;suy rab2Cc2 613;suy
a2Cb2Cc2 638: Nếub Dcthì doa65nênc >
2:Ta có
a2Cb2Cc2 D.10 2c/2C2c2 D6c2 40cC100D6
c 10
2
C 100 :
Do
3 6c 10
3 ;nên ta có
c 10
3
C100 66
10
2
C 100 D
(122)Vậy giá trị lớn cần tìm là38:
Ví dụ 3.3 (Định lý đại số) Một đa thức không đồng số với hệ số phức bất kỳ luôn có nghiệm phức.
Lời giải Xét đa thức p.z/và xét đại lượng jp.z/j hàm hai biến số thựcx vày;trong đóz DxCiy:Theo bổ đề3;hàm số đạt giá trị nhỏ điểm z0 D x0; y0/:Khơng tính tổng qt, cách đổi biến số, giả sửz0 D0:Giả sử điểm hàm sốp.z/không bằng0:Gọiklà số nguyên dương nhỏ cho hệ số củazk đa thức khác0:Khi ú
p.z/Da0Cakzk C Canzn; k>1; ak Ô0:
Ngoi raa0 Ô 0vỡa0 D p.0/ D p.z0/ Ô0:Bõy gi ta lấy nghiệm phứcu phương trìnha0Cakzk D0;tức phức bậckcủa a0ak1:Ta có
jp.t u/j D ja0Caktkuk Co.tk/j D j.1 tk/a0Co.tk/j < a0Dp.0/;
vớit > 0đủ nhỏ Mâu thuẫn theo giả thiết,z0 D0là điểm hàm số đạt giá trị nhỏ Định lý chứng minh
Ví dụ 3.4 (Đề thi chọn đội tuyển trường PTNK năm 1999) Chox; ylà số thực thoả mãn điều kiện06x; y62và16xCy 63:Tìm giá trị lớn giá trị nhỏ của
ADx2Cy2Cxy 3x 3y:
Lời giải Hàm sốf x; y/D x2Cy2Cxy 3x 3y khả vi liên tục compactDxác định bất đẳng thức đề (Dlà hình lục giác !) vậyf x; y/đạt giá trị lớn giá trị nhỏ trênD:Các giá trị lớn giá trị nhỏ đạt điểm dừng biên củaD:
Hệ phương trình tìm điểm dừng có dạng (
2xCy 3D0 2yCx 3D0
có nghiệm nhất.x; y/ D.1; 1/:Giá trị điểm nàyf 1; 1/D 3:
Trên biên x D y 2; ta có f x; y/ D y2 3y có giá trị lớn 2đạt 0; 1/; 0; 2/;có giá trị nhỏ
4 đạt
0;
:Tương tự biên y D0; 16x62:
Trên biênxD2và06y 61;ta cóf x; y/Dy2 y 2;có giá trị lớn 2đạt tại.2; 0/; 2; 1/và giá trị nhỏ
4 đạt
2;
:Tương tự biêny D 2và 06x61:
Trên biênxCy D1; 6x 61ta cóf x; y/ Dx2 x 2có giá trị lớn đạt tại.0; 1/và.1; 0/và giá trị nhỏ
4 đạt
1 2;
1
(123)Tạp chí Epsilon, Số 05, 10/2015
Trên biênxCy D3và16 x 2;ta cóf x; y/ D x2 3xcó giá trị lớn đạt tại.1; 2/; 2; 1/và giá trị nhỏ
4 đạt 3
2;
:
Từ giá trị lớn củaf 2đạt đỉnh hình lục giác.0; 1/; 0; 2/; 1; 2/; 2; 1/; 2; 0/; 1; 0/và giá trị nhỏ 3đạt điểm.1; 1/:
Nhận xét Vì compact mơ tả bất đẳng thức đẳng thức (ví dụ hình cầu
x2Cy2Cz2D1/nên tốn cực trị compact quy tốn cực trị có điều kiện Với tốn này, sử dụng phương pháp nhân tử Lagrange để tìm lời giải
Ta giới thiệu nội dung phương pháp thơng qua trường hợp tốn cực trị nhiều biến với điều kiện ràng buộc:
Để tìm cực trị hàm sốf x1; x2; : : : ; xn/với điều kiện ràng buộcg.x1; x2; : : : ; xn/D0;
ta xét hàm
F ; x1; x2; : : : ; xn/Df x1; x2; : : : ; xn/Cg.x1; x2; : : : ; xn/:
Sau ta tìm cực trị củaF:Chú ý điểm cực trị thoả mãn điều kiện
g.x1; x2; : : : ; xn/DF0.; x1; x2; : : : ; xn/D0;
nên cực trị f với điều kiện ràng buộcg.x1; x2; : : : ; xn/D0:
Ví dụ 3.5 Tìm giá trị giá trị nhỏ hàm sốf x; y/D5x2C2xyC3y2với điều kiện
g.x; y/D7x2C2xyC4y2–3D0:
Lời giải XétLDf x; y/Cg.x; y/:Đạo hàm theo biếnxvày;ta (
10xC2yC.14xC2y/D0 2xC6yC.2xC8y/D0
Từ tínhtừ phương trình cho nhau, ta 10xC2y
14xC2y D
6yC2x 8yC2x:
Từ suy y
x D f 1; 2g:Thay vào phương trìnhg.x; y/D0;ta tìm số điểm “nghi
vấn” cực trị Tính giá trị hàm số f điểm này, ta thu giá trị nhỏ đạt điểm
.x; y/D
1
3;
;
1 3;
1
:
Từ dẫn đến kết luận toán
(124)Bài tập 3.1 (Bài toán Entropi cực đại)Vớinsố dươngx1; x2; : : : ; xnsao cho n X
kD1
xk;tìm
giá trị nhỏ tổng n X
kD1
xkln.xk/:(tổng với dấu trừ gọi entropi)
Bài tập 3.2 Tổng của5số thực bằng1;tổng bình phương chúng bằng13thì giá trị nhỏ tổng lập phương chúng ?
Bài tập 3.3 Tổng của5số thực bằng1;tổng bình phương chúng bằng11thì giá trị lớn tổng lập phương chúng ?
Ví dụ 3.6 (Chọn đội tuyển Việt Nam 2001) Chox; y; zlà số thực dương thoả mãn điều kiện2xC4yC7z D2xyz:Tìm giá trị nhỏ biểu thứcxCyCz:
Lời giải Xét
LDxCyCzC.2xC4yC7z 2xyz/; hệ phương trìnhLx DLy DLz D0có dạng
8 ˆ <
ˆ :
1C.2 2yz/D0 1C.4 2zx/D0 1C.7 2xy/D0 Từ ta tìm
8 ˆ ˆ ˆ ˆ ˆ <
ˆ ˆ ˆ ˆ ˆ :
2yz D2C 2zx D4C 2xy D7C
Mặt khác, điều kiệnxC4yC7z D2xyz;có thể viết lại thành
2yz C 2zx C
7
2xy D1: Thay biểu thức vừa tìm vào, ta tìm
2 2C1C
4 4C1C
7
7C1 D1: Biến đổi tương đương, ta phương trình
1123C502–1D0: Phương trình có nghiệmD
8; D
4˙p2
14 (loại dẫn đếnyz < 0/:Từ ta có
ˆ <
ˆ :
(125)Tạp chí Epsilon, Số 05, 10/2015
Từ tính điểm dừng là.x; y; z/D
3; 2;
:Sự tồn giá trị nhỏ đảm bảo bổ đề3;do
3;
2;
chính điểm mà hàm số đạt giá trị nhỏ
Bài tập 3.4 Cho a; b; c số thực dương cho trước vàx; y; z số thực dương thay đổi thoả mãn điều kiệnxyz DaxCbyCcz:Chứng minh
xCyCz > r
bCcC 2bc d C
r
cCaC 2ca d C
r
aCbC 2ab d ; đód số thực dương xác định phương trình
a aCd C
b bCd C
c
cCd D1:
Ví dụ 3.7 Tìm điểmP nằm tam giác cho tổng tỷ số độ dài cạnh khoảng cách từP đến cạnh đạt giá trị nhỏ nhất.
Lời giải Gọi a; b; c độ dài cạnh tam giác vàx; y; z khoảng cách từP đến cạnh tương ứng Ta cần tìm giá trị nhỏ hàm số
f x; y; z/D a x C
b y C
c z:
Trong đại lượngx; y; zliên quan với thơng qua diện tích tam giác NốiP với đỉnh tam giác, ta ba tam giác tổng diện tích diện tíchS tam giác Như
S D ax C
by C
cz : Ta có tốn:
8 <
:
f x; y; z/D a x C
b y C
c
z !min axCby Ccz D2S
Lập nhân tử Lagrange
L.; x; y; z/D a x C
b y C
c
z C.axCby Ccz 2S /: Hệ phương trình tìm điểm dừng có dạng
8 ˆ ˆ ˆ ˆ ˆ < ˆ ˆ ˆ ˆ ˆ : a
x2 CaD0 b
y2 Cb D0 c
z2 CcD0
(126)Ví dụ 3.8 (Bất đẳng thức Holder) Chox1; x2; : : : ; xnlà số thực dương vàp; qlà số dương thoả mãn điều kiện
p C
q D1:Chứng minh ta có bất đẳng thức
n X
kD1
xkyk n X
kD1 xkp
!p1 n X
kD1 xkq
!q1 :
Lời giải Do tính nhất, ta cần chứng minh n
X
kD1
xkp D1; n X
kD1
ykq D1: 1/
thì
n X
kD1
xkyk 1:
Ta cần tìm giá trị lớn n X
kD1
xkyk với điều kiện ràng buộc.1/:Xét nhân tử Lagrange
LD n X
kD1
xkyk C n X
kD1
xkp !
C n X
kD1
ykq !
:
Hệ phương trình tìm điểm dừng có dạng (
yi Cpxip D0; i D1; : : : ; n xi Cpyiq D0; i D1; : : : ; n
Viếtnphương trình dạngyi D p/xip 1;lấy luỹ thừaq cộng lại vế theo vế, với ý rằng.p 1/q Dp;ta suy p D 1;suy rayi D xip 1:Đây điểm dừng điểm hàm số
n X
kD1
xkyk đạt giá trị lớn Nhưng rõ ràng giá trị lớn bằng1nên ta có điều phải chứng minh
Cuối cùng, bình luận lời giải số tốn bất đẳng thức góc nhìn tốn cao cấp
Ví dụ 3.9 (British MO 1986) Chox; y; zlà số thực thoả mãn điều kiệnxCyCz D0và
x2Cy2Cz2 D6:Tìm giá trị lớn biểu thức
F Dx2y Cy2z Cz2x:
Lời giải Bài tốn có nhiều cách tiếp cận khác Nếu dùng phương pháp hàm số, ta rútz D x y thay vào đưa tốn hai biến:
Chox2CxyCy2 D3;tìm giá trị lớn của
(127)Tạp chí Epsilon, Số 05, 10/2015
Sau biến đổi điều kiện thành3.xCy/2C.x y/2D12để đặt xCy D2cos; x–y D2p3cos; đưa toán hàm biến
Phương pháp hàm số giải theo cách khác đẹp đẽ hướng dẫn tập25:Dưới ta trình bày cách giải tuyệt đẹp sử dụng bất đẳng thức Cauchy-Schwarz Ta viết
3F Dx.2xyCz2/Cy.2yzCx2/Cz.2zxCy2/: 2/ Từ đó, áp dụng bất đẳng thứcCauchy-Schwarz, ta có
9F26.x2Cy2Cz2/Œ.2xy Cz2/2C.2yzCx2/2C.2zxCy2/2 D6Œx4Cy4Cz4C4.x2y2Cy2z2Cz2x2/C4xyz.xCyCz/ D6Œ.x2Cy2Cz2/2C2.xyCyzCzx/2
D6.36C18/D369:
Suy raF 66:Phần dấu xảy xin dành cho bạn đọc
Ở lời giải từ khơng khí ! Sau phút trấn tĩnh, ta bật câu hỏi:Làm có thể biết mà tách3F như ở.2/để áp dụng bất đẳng thức Cauchy-Schwarz ?
Thực ra, điều không hồn tồn mị mẫm may mắn Ta thử lý giải lời giải góc nhìn tốn cao cấp, áp dụng phương pháp nhân tử Lagrange cho tốn tìm giá trị lớn hàmF với điều kiện ràng buộc
xCy Cz D0; x2Cy2Cz2 D6: 3/; Ta xét hàm
F x; y; z; ; /Dx2yCy2zCz2xC.xCyCz/C.x2Cy2Cz2 6/: Ta có hệ tìm điểm dừng, ngồi hai phương trình.3/sẽ cịn phương trình
8 ˆ <
ˆ :
2xyCz2CC2x D0 2yzCx2CC2y D0 2zxCy2CC2zD0
Tt nhiờn ta ch xộtxyz Ô0;cng ba phng trỡnh ny lại với ýxCyCz D 0ta D0;và
2xyCz2 x D
2yzCx2 y D
2zxCy2
z D 2: 4/ Tuy giải cụ thể nghiệmx; y; znhưng để ý thấy.4/ điều kiện xảy đẳng thức bất đẳng thức Cauchy-Schwarz điều lý giải ta lại thành công tách3F trong.2/và áp dụng bất đẳng thức Cauchy-Schwarz Ví dụ 3.10 Giải ví dụ18theo sơ đồ sau:
(128).b/ Đặtt Dxyz và tínht theox;từ tìm miền giá trị củat:
.c/ ĐặtG Dxy2Cyz2Czx2:Hãy tính tổngF CG và tíchF Gtheot:
.d / Từ suy cơng thức tínhF theot:Sẽ có hai giá trị choF như nghiệm phương trình bậc2:
.e/ Tìm giá trị lớn nhấtF (khi tìm giá trị lớn nhất, ta lấy nghiệm với dấuC/:
Ví dụ 3.11 Vớin>2vàx1; x2; : : : ; xnvày1; y2; : : : ; ynlà2nsố thực thoả mãn điều kiện
n X
iD1
a2i D1; n X
iD1
bi2D1; n X
iD1
aibi D0:
Chứng minh rằng
n X
iD1
!2 C
n X
iD1 bi
!2 n:
Lời giải Bất đẳng thức khó chịu có lời giải ngắn gọn ấn tượng Đặt AD
n X
iD1
ai; B D n X
iD1
bi: Khi khai triển bất đẳng thức hiển nhiên
1 Aai Bbi2 >0; ta
1 2Aai 2Bbi C2ABaibi CA2a2i CB2bi2>0: Choi chạy từ1đếnnrồi cộng bất đẳng thức lại vế theo vế, ý
n X
iD1
a2i D1; n X
iD1
bi2D1; n X
iD1
aibi D0; ;
ta
A2CB26n; điều phải chứng minh
Dưới góc nhìn tốn sơ cấp khó giải thích lại nghĩ lời giải này, lại biết để bình phương đại lượng1 Aai Bbi cộng lại ? Chúng ta lại phải đổ cho “kinh nghiệm, óc phán đốn” hay “nhạy cảm tốn học” Tốn cao cấp giúp giải thích lời giải độc đáo :
Xét tốn tìm giá trị lớn hàm số
f a; b/D n X
iD1
!2 C
n X
iD1 bi
!2 ;
với điều kiện ràng buộc
n X
iD1
ai2D1; n X
iD1
bi2 D1; n X
iD1
(129)Tạp chí Epsilon, Số 05, 10/2015 Lập nhân tử Langrange
LD n X
iD1
!2 C
n X
iD1 bi
!2 C
n X
iD1
a2i !
C n X
iD1
b2i !
C n X
iD1 aibi:
Các phương trình tìm điểm dừng có dạng (
2AC2ai Cbi D0; i D1; 2; : : : ; n 2BC2ai Cbi D0; i D1; 2; : : : ; n
Từ hệ ny, nu4 Ô0;thỡ tt c cỏcai bng v tất cácbi Điều xảy Do ta phải có4 D0:Lúc này, để hệ có nghiệm, ta lại phải có
A B D D 2: Ngồi ra, ta có
0D n X
iD1
aibi D n X
iD1
ai 2A 2ai/D 2A2 2:
Suy ra2D 2A2vàD 2AB:Từ ta có điều kiện1 Aai Bbi D0:Như điểm dừng ta có hệ thức1 Aai Bbi D0và sở để ta “mạnh dạn” thực phép bình phương nói
Ví dụ 3.12 (IMO Shortlist 2007) Choa1; a2; : : : ; a100 là số thực không âm thoả mãn điều kiệna21Ca22C Ca2100 D1:Chứng minh rằng
a12a2Ca22Ca3C Ca1002a1 < 12 25:
Lời giải ĐặtS D 100 X
kD1
a2kakC1(như thường lệ, ta xét số theo modulo100;tức ta đặt a101Da1; a102 Da2/:
Áp dụng bất đẳng thức Cauchy-Schwarz cho dãy.akC1/và.ak2C2akC1akC2/và sau bất đẳng thức Cauchy cho sốak2C1vàa2kC2:
Áp dụng đánh giá hiển nhiên sau 100
X
kD1
.a4k C2a ka
2
kC1C2a ka
2 kC2/
100 X
kD1 ak2
!2 ;
và
100 X
kD1
ak2ak2C1 50 X
iD1 a22i 1
! 50 X
iD1 a22i
! ;
ta thu
.3S /2 100 X
iD1 a2k
!2 C4
50 X
iD1 a22i
! 50 X
iD1 a2i2
!
1C 50 X
iD1
a2i 12 C 50 X
iD1 a22i
(130)Từ
S p
2
3 0:4714 < 12
25 D0:48: Ta có điều phải chứng minh
Ở đây, bước biến đổi đánh giá dễ hiểu, phức tạp ví dụ trước Tuy nhiên, lời giải ví dụ16;việc tách
3S D 100 X
kD1
akC1.ak2C2akC1akC2/
khá khó hiểu thiếu tự nhiên Điều trở nên dễ hiểu ta áp dụng phương pháp nhân tử Lagrange thấy giá trị lớn biểu thức đề đạt giá trị thoả mãn hệ phương trìnha2k 1C2akakC1D2ak với mọik D1; 2; : : : ; 100:Chính tỷ lệ điểm cực trị giúp mạnh dạn sử dụng bất đẳng thức Cauchy-Schwarz để đánh giá Bài tập 3.5 (IMO Shortlist 1995) Vớia; b; c số thực dương cho trước vàx; y; zlà số thực dương thay đổi thoả mãn điều kiệnxCyCz D aCbCc:Tìm giá trị lớn biểu thức
4xyz a2xCb2y Cc2z/:
Bài tập 3.6 (IMO 1984) Chox; y; zlà số thực không âm có tổng bằng1:Chứng minh bất đẳng thức
0xyCyzCzx 2xyz 27:
Tài liệu tham khảo
[1] J.Brinkhouse & V.Iu.Protasov,Lý thuyết cực trị qua ví dụ đơn giản, Tạp chí “Truyền bá tốn học” số9năm2005;trang32 55:
[2] Nguyễn Văn Mậu,Bất đẳng thức - Định lý áp dụng, Nhà xuất Giáo Dục,2006: [3] Đoàn Quỳnh chủ biên,Tài liệu giáo khoa chuyên toán11; 12;Đại số Giải tích, Nhà xuất
bản Giáo dục2011; 2012:
[4] V.Tikhomirov,Các câu chuyện maximum minimum, Nhà xuất MCCME,2006: [5] The Vietnamese Mathematical Olympiad 1990 2006/ Selected Problems, Education
Publishing House,2007:
(131)THẶNG DƯ BẬC HAI MODULO M
Nguyễn Hồng Lữ - Trường THPT Chuyên Lương Thế Vinh - Đồng Nai
LỜI GIỚI THIỆU
Các sốk-phương mod p/trong đóp số ngun tố đóng vai trị quan trọng trọng lí thuyết số Các sốk phương giới toán học quan tâm nghiên cứu từ xa xưa, đặc biệt từ kỷ17cho đến có nhiều cơng trình lí thuyết số nghiên cứu tính chất ứng dụng sốk-phương
Định nghĩa
Số k-phương mod m/: Cho số nguyên dương m; m và số nguyên a sao cho
.a; m/ D 1 Nếu tồn số tự nhiên x sao cho: xk a mod m/ thì ta nói a là số
k-phương modulemhay nói:alà số lũy thừa bậcktheo modulem, có người nói:a
là thặng dư bậckcủam.
Số phương modm: Cho số nguyên dươngm2và số nguyênasao cho.a; m/D1. Nếu tồn số tự nhiênxsao chox2a mod p/thì ta nóialà số phương module
m(cũng nóialà thặng dư bình phương củam)
Sốk–phương module nguyên tố đơn giản hay gặp số2-phương module ngun tố mà ngơn ngữ lí thuyết số ta gọi thặng dư bậc hai theo module nguyên tố hay số phương modpnguyên tố
1 Thặng dư bậc hai modulo p
1.1 Khái niệm
Cho số nguyênm, cho số nguyên tố lẻp:
Nếu phương trìnhx2 a mod p/có nghiệm ngun ta nói alà số phương modulem(cũng nóialà thặng dư bình phương củam)
Nếu phương trìnhx2a mod p/khơng có nghiệm ngun ta nóialà số phi phương modulem(cũng nóiakhơng phải thặng dư bình phương củam)
Nếua0 mod m/thì ta nói:akhơng phải số phương modulem, đồng thờia khơng phải số phi phương modulem
Kí hiệu:
+) aQRp:alà số phương modulep (viết tắt chữquadratic residue)
(132)Vì52 mod 1/1nên:3là số phương module11hay3QR11
Vìa2 mod 3/với số nguyênanên: 2là số phi phương module3 hay
2NR3.
Vìb2 mod7/ với số nguyênb nên:3 là số phi phương module7 hay
3NR7.
Định lí sau cho ta mối quan hệ phép nhân thặng dư bậc hai mod p/ Định lý 1.1 Choplà số nguyên tố lẻ Ta có:
Nếu:aQRpvàbQRpthìabQRp Nếu:aQRpvàbNRpthìabNRp Nếu:aNRpvàbNRpthìabQRp
Một cách tổng quát:Tích hai số phương phi phương modulep cho ta số phương modulep Tích số phương số phi phương modulepsẽ cho ta số phi phương modulep
Chú ý:Bạn thấy định lí giống với phép nhân dấu âm (-) với dấu dương(+) đại số: hai số dấu tích số dương; hai số trái dấu tích số âm!
Nhận xét:Ta cóaQRm,.aCm/QRm: Điều cho ta thấy: Nếu phần tử lớp thặng dư modulomlà số chình phương modulomthì phần tử lớp thặng dư modulo m
Ví dụ 1.2 Với modulo bằng7: số nguyên tố lẻ Ta có:
+) Tập hợp sốf1I2I4glà3số phương module7
+) Tập hợp sốf3I5I6glà3số phi phương module7.
Nhận xét:có
2 D3cho loại
Ví dụ 1.3 Với modulo bằng13: số nguyên tố lẻ Ta có:
+) Tập hợp số1I3I4I9I10I12là6số phương module13
+) Tập hợp số2I5I6I7I8I11là6số phi phương module13.
Nhận xét:có 13
2 D6số cho loại Từ ví dụ 2, ví dụ ta đến định lí sau:
Định lý 1.2 Nếuplà số nguyên tố lẻ tập1; 2; :::; p 1số thặng dư bình phương củap bằng số số khơng phải thặng dư bình phương củapvà bằng p
(133)Tạp chí Epsilon, Số 05, 10/2015
Chứng minh Để ý rằng:k2 p k/2 modp/, nên tập hợp˚12I22I:::I.p 1/2 có p
2 cặp đồng dư (6 ) với theo modp Cho k chạy từ1đến p
2 ta đặt k
ak mod p/vàak thuộc tập hợpf1I2I: : :Ip 1g, có p
2 số phương modp Ta chứng minhai ¤aj với1i ¤j p
2 Thật (phn chng): Gi s cú1 i Ôj p
2 màai D aj; đói
j2 mod p/).i j /.i Cj /chia hết chop, màiCj < p nên suy rai j phải chia hết chop )i Dj: iu ny mõu thun vi1i Ôj p
2
Vậy tậpf1I2I: : :Ip 1gcó p
2 số phương modp p
2 số phi phương modp
Định lý 1.3 Nếu p là số nguyên tố lẻ và p khơng ước số củaa thì phương trìnhx2 a mod p/hoặc vơ nghiệm có hai nghiệm khơng đồng dư theo modp.
Chứng minh Nhận thấy rằng: Nếu x b mod p/ nghiệm phương trình x2 a mod p/thìx b mod p/là nghiệm phương trìnhx2 a modp/ Nếu lớp thặng dư Œb modp/trùng với lớp thặng dưŒ b mod p/, suy rab b/phải chia hết chop, hay2b chia hết cho số nguyên tố lẻp, đób chia hết cho số nguyên tố lẻp (1)
Mặt khácb2a mod p/(2)
Từ (1)và(2) ta suy raachia hết chop: điều trái giả thiết!
2 Kí hiệu Legendre
2.1 Kí hiệu Legendre
Cho số nguyên tố lẻp vàalà số nguyên:
Nếu a số phương module p ta kí hiệu:
a p
D1
Nếu a khơng phải số phương module p ta kí hiệu: a
p
D
Nếu số nguyên tố p ước số a kí hiệu a
p
D0
2.2 Một số tính chất liên quan kí hiệu Legendre
Tính chất 2.1 Với số nguyên tốp (lẻ hay chẵn) ta ln có:
p
(134)Tính chất 2.2 Cho số nguyên tố lẻp Nếu:.a; p/ D1;.b; p/D1vàab mod p/thì: a p D b p :
Ví dụ 2.1 Xét xem số2014có phải số phương modulo7hay khơng?
Chứng minh Ta có:20101 mod7/nên theo tính chất2ta có:
2010 D 1
; mà theo tính chất ta có:
1
D1 Vậy
2010
D1hay2014là số phương modulo7
Tính chất 2.3 Choplà số nguyên tố lẻ: Với số ngun a,b ta ln có:
ab p D a p : b p
(Điều nói lên: Kí hiệu Lagrange có tính chất nhân)
Hệ 2.1 Vớia2Z;p là số nguyên tố:akhông chia hết chop ta ln có:
a2 p
D1.
Ví dụ 2.2 Xét xem số125có phải số phương modulo41hay khơng?
Chứng minh Ta có:1254 mod41/nên theo tính chất2ta có: 125 41 D 41
; mà theo tính chất ta có:
4 41 D 22 41
D1 Vậy suy ra: 125
41
D1hay125là số phương modulo41
Hệ 2.2 Nếu:.a; p/D1thì
an p D a p n
(vớinlà số nguyên dương tùy ý )
Ví dụ 2.3 Xét xem số75có phải số phương modulo97hay khơng?
Chứng minh Ta có:75 D 3:52 nên theo tính chất ta có 75 97 D 3:52 97 D 97 52 97
(1), để ý theo tính chất ta có 52
97
D1(2) Ta có102 mod 97/nên
3 97
D1(3) Từ (1),(2),(3) ta suy
75 97
D1
Tính chất 2.4 (Euler’s Criterion)Với số nguyênakhông chia hết cho số nguyên tố lẻp ta ln có:
a p
ap21 modp/.
(135)Tạp chí Epsilon, Số 05, 10/2015
Tính chất 2.5 (Tính chất gọi luật tương hỗ Gauss hay luật thuận nghịch bình phương):Với hai số nguyên tố lẻp,q phân biệt ta có:
p q : q p
D 1/p21: q
2 :
Luật viết:
p q
D 1/p21: q
2 : q
p
Hệ 2.3 Nếu hai số ngun tố lẻp,qcó dạng4kC1thì:
p q D q p
Nếu hai số ngun tố lẻp,q có dạng4kC3thì:
p q D q p .
Ví dụ 2.4 Xét xem13có phải số phương modulo17hay khơng?
Chứng minh Theo luật tương hỗ ta có: 13
17
D 1/1321: 17 : 17 13 hay: 13 17 D 17 13 ;
mặt khác ta có:
2217 mod 13/) 17
13
D1:
Từ đó, ta có:
13 17
D1
Tính chất 2.6 Choplà số nguyên tố lẻ; ta có:
2 p
D 1/p28 1:
Tính chất 2.7 Choplà số nguyên tố lẻ; ta có:
2 p
D 1/ŒpC41
Từ tính chất ta chứng minh mệnh đề quan trọng sau đây: T1:1là số phương modulop (với số nguyên tố p dù chẵn hay lẻ)
Chú ý: Từ tính chất T2 đến T12 thìp số nguyên tố lẻ T2: 1là số phương modulop ,p mod4/
T3: 1khơng số phương modulop ,p mod 4/ T4:2là số phương modulop ,p ˙1 mod8/
(136)T7:3là số phương modulop ,p ˙1 mod1/2 T8:3khơng số phương modulop ,p ˙5 mod 1/2 T9: 3là số phương modulop ,p mod6/
T10: 3khơng số phương modulop ,p mod 3/ T11:5là số phương modulop ,p ˙1 mod5/
T12:5là số phi phương modulop ,p ˙2 mod 5/ Bổ đề 2.1 Bổ đề Gauss:Cho số nguyêna, số nguyên tố lẻp.
Ta đặtm D p ;S D
m X
kD1 2ka p ta có: a p
D 1/S.
Có mối liên hệ khơng số phương modulo p nguyên thủy modp? Câu trả lời cho định lí sau:
Định lý 2.1 Cho số nguyên tố lẻp Choglà nguyên thủy modp/ Choalà số ngun Ta có:alà số phương modp ,ag2k mod p/.
Định lí suy rằng: Mỗi lũy thừa bậc chẵn nguyên thủy modp nguyên tố thặng dư bậc hai modp nguyên tố!
Ví dụ 2.5 2là nguyên thủy mod11 Theo định lí ta có: Các số ngun:4; 16;64;
256; 1024là số phương mod11, vì: D 22; 16 D 24;64 D 26 ;256 D 28;
1024D210.
3 Các ví dụ minh họa
Ví dụ 3.1 Xét xem số6có phải số phương modulo73hay khơng?
Chứng minh Ta có: 6 73 D 2 73 3 73 (1) Theo tính chất ta có:
2 73
D 1/7328 D1(2).
Vì73là số nguyên tố dạng4kC1nên theo luật tương hỗ Gauss ta có: 71 D 71
(3); để ý rằng:22 73(mod 3) suy
71
3
D1(4) Từ (1),(2),(3),(4) ta có
6 73
D1hay6là số phương modulo73
Ví dụ 3.2 Tính:
26 73
Chứng minh Theo tính chất nhân kí hiệu Lagrange có
26 73 D 1/:2:13 73 D 73 73 13 73
Vì 73 số nguyên tố nên theo tiêu chuẩn Euler có:
1 73
D 1/7321 D1I
2 73
(137)
Tạp chí Epsilon, Số 05, 10/2015
Theo luật thuận nghịch bình phương Gauss(Gauss’s Quadratic Reciprocity Law)ta có: 13
72
D 1/1321: 73 : 73 13 D 73 13 :
Mặt khác dựa vào tính chất: “Nếu: a; p/ D 1;.a; p/ D vàa b mod p/thì: a p D b p
” suy ra73 mod1/3nên 73 13 = 8 13 = 23 13 = 2 13
(hệ t/c nhân) ; mà
2 13
D 1/1328 D 1) 73 13 D Vậy có: 26 73
D Hay –26khơng thặng dư bình phương modulo73
Ví dụ 3.3 Tính
12 23
Chứng minh Cách 1: Ta có 12
23
D 22:3
23 D 22 23 3 23 D 2 23 3
23 Mà 23
D 1/2328 D1.
Theo luật thuận nghịch bình phương Gauss có:
3 23
D 1/321: 23 : 23
D 1/
2
D 1/ 1/3281 D1
Suy 12
23
D1
Cách 2: Vì12 11 mod 23/ Suy 12 23 D 11 23 D 1 23 11 23 mà 1 23
D 1/2321 D 1 Theo luật thuận nghịch bình phương Gauss có:
11 23
D 1/1121: 23 : 23 11
D 1/
1 11
D 1/:1D
Suy 11 23 D
(138)Chứng minh Phương trình (*),x2 41 mod 1/03: Điều cho ta thấy: Để trả lời câu hỏi phương tình (*) có nghiệm hay khơng ta phải trả lời câu hỏi số nguyên tố41có phải số phương modulo103hay khơng?
Để ý rằng41là số ngun tố dạng4kC1còn103là số nguyên tố dạng4kC3nên theo luật tương hỗ Gauss ta có:
41 103 D 103 41 :
Theo tính chất ta có 103 41 = 21 41
(vì10321 mod 41/) Theo tính chất ta có
21 41 D 3:7 41 D 3 41 : 7 41 :
Theo luật tương hỗ Gauss ta có: 41 D 41
mà41 mod 3/nên theo tính chất ta có: 41 D 1 D
(theo hệ tính chất 6) Tương tự: 7 41 D 41 D 1 D 1:
Như có: 41 : 41
D1suy có
41 103
D1 Suy ra41số phương modulo103nên (*) có nghiệm
Ví dụ 3.5 Chứng tỏ phương trình:x2 59yD30khơng có nghiệm ngun.xIy/.
Chứng minh Ta có:
30 59 D 2 59 3 59 5 59 mà 2 59
D 1/5928 D 1I 3
59
D 1/321: 59
2 : 59
3
D 1/ 2
3
:
( vì592 mod 3/) Mà
2
D 1/3281 D 1suy ra: 59 D1 Ta có: 59
D 1/521: 59 : 59 D 59 D
( vì59 mod 5/), mà
1
D 1/521 D1suy ra:
3 59
D1 Vậy 30
59
D
(139)Tạp chí Epsilon, Số 05, 10/2015
4 Thặng dư bậc hai modulo hợp số
4.1 Kí hiệu Jacobi
Đặt vấn đề: Như ta trình bày trên: Kí hiệu Lagrange vơ tiện lợi dùng cho moduloplà số nguyên tố, Lẽ tự nhiên bạn đặt câu hỏi: Nếu modulomkhông phải số ngun tố sao? Để có câu trả lời vào tìm hiểu kí hiệu Jacobi dành cho thặng dư bình phương modulo hợp số:
Định nghĩa Choalà số nguyên,mlà số nguyên dương lẻ. Giả sửmcó phân tích tiêu chuẩn làm Dps1
1 :p s2 :p s3 :::p sk
k Kí hiệu a
m
J sẽ gọi kí
hệu Jacobi tính sau:
a m J D a p1 s1 : a p2 s2 ::: a pk sk : /
Cần lưu ý bạn đọc bạn đọc: Vìmlẻ suy phân tích tiêu chuẩnmDps1 :p s2 :p s3 :::p sk k thì
các sồ nguyên tốpi (i D1; 2; :::; k) lẻ nên kí hiệu vế phải (*) hiểu kí hiệu Lagrange) Ví dụ: 1 15 J D 1 1
D 1/: 1/D1:
Nhận xét:
Khi m số ngun tố kí hiệu Jacobi trùng với kí hiệu Lagrange ! Từ (*) ta suy ra: Nếu a
m
J D a khơng thặng dư bậc hai pi (i D1; 2; :::; k)
a p1
si
D1thìa m
D1
Từ (*), khia m
J D1thì khơng thể suy a thặng dư bậc hai mọipi(i D1; 2; :::; k) Bởi ký hiệu Jacobi tích ký hiệu Legendre, nên có hai ký hiệu Legendre 1và ký hiệu Jacobi
Kí hiệu Jacobi : Cho alà số nguyên, mlà số nguyên dương lẻ không thiết mlà số nguyên tố
Nếualà số phương modulemthì ta kí hiệu:a m
J D1 Nếualà số phi phương modulemthì ta kí hiệu:a
m
J D Nếu số ngun dương lẻmlà ước số củaathì kí hiệua
m
J D0
4.2 Các tính chất sau thường dùng để tính nhanh ký hiệu Jacobi
TC1:Nếumlà số ngun tố lẻ kí hiệua m
J
trùng với kí hiệua m
(140)TC2:Vớialà số nguyên;mlà số tự nhiên lẻa m
J f 1I0I1g TC3:Vớialà số nguyên;mlà số tự nhiên lẻ
a m
J D0,gcd.aIn/Ô1 TC4:Via,bl hai s nguyờn;ml s tự nhiên lẻ
ab m
J
Da m J: b m J Hệ quả: ak m J
Da m
k
J
TC5:Vớialà số nguyên;m,nlà2số tự nhiên lẻ a mn J D a m J: a n J Hệ quả: a
m2 J D a m
J f0I1g:
TC6:Vớialà số nguyên;mlà số tự nhiên lẻ Nếu:ab modm/thì:a m J D b m J :
TC7:Vớialà số nguyên;mlà số tự nhiên lẻ 1
m
J D1:
TC8:Vớialà số nguyên;mlà số tự nhiên lẻ
1 m
J
D 1/m21 D
1 if m1 mod4 if m3 mod4: TC9:Vớialà số nguyên ;mlà số tự nhiên lẻ
1 m
J
D 1/m28 D
1 if m1I7 mod8 if m3I5 mod8: TC10(Luật tương hỗ Gauss): Vớim,nlà hai số tự nhiên lẻ,.m; n/D1thì
m n J n m
J D 1/
.m 1/.n 1/
4 :
Tính chất (giống ký hiệu Legendre) gọi:luật thuận nghịch bình phương.
TC11:Vớimlà số tự nhiên lẻ
2 m
J
D 1/m28 1:
TC12:Vớimlà số tự nhiên lẻ
2 m
J
D 1/ŒmC41:
TC12:2là số phương modulop ,p ˙1 mod 8/: TC13: 2là số phương modulop ,p 1; mod 8/: TC14:3là số phương modulop ,p ˙1 mod 12/:
TC14: 3là số phương modulop ,p mod 6/:
Chú ý:Ta mở rộng đồng dư thức Euler
a p
ap21 mod p với số nguyên tố p số nguyên a từ ký hiệu Legendre sang ký hiệu Jacobi là:a
m
L a m
(141)Tạp chí Epsilon, Số 05, 10/2015
4.3 Một ví dụ so sánh kí hiệu Lagrange kí hiệu Jacobi
Sử dụng kí hiệu Lagrane:Tính: 1001 9907 D‹ Ta có: 1001 9907 D 7 9907 11 9907 13 9907 1/ Mà: 9907 D 9907 D
D 2/
và 11 9907 D 9907 11 D 7 11 D 11 D 4
D1 3/
13 9907 D 9907 13 D 13
D1: 4/
Từ (1)(2)(3)(4) có:
1001 9907
D
Sử dụng kí hiệu Jacobi:Tính:
1001 9907 J D‹ Ta có: 1001 9907 J D 9907 1001 J D 898 1001 J D 2 1001 J 449 1001 J D 449 1001 J D 1001 449 J D 103 449 J D 449 103 J D 37 103 J 103 37 J D 29 37 J D 37 29 J D 29 J D J D 1:
So sánh: Sự khác biệt hai hai cách tính tốn là: tính bắng kí hiệu Legendre "tử số" phải phân tích thành lũy thừa số ngun tố Điều làm cho việc tính tốn cách sử dụng kí hiệu Legendre chậm so với cách tính bắng sử dụng kí hiệu Jacobi đáng kể Ví dụ 4.1 Tìm nghiệm ngun phương trình:x2Dy3 5:
Chứng minh Ta có: Nếuychẵn suy ray3 mod 8/)y3–5 mod8/)y3–53 mod 8/ ) x2 mod 8/ Điều khơng xảy vì: số phương 0I1I4 mod 8/
Nếuy mod 4/)y3 mod 4/ Điều khơng xảy vì: số phương chỉ0I1 mod 4/
Nếuy mod 4/)y D4zC1 Thay vào phương trình cho cóx2C4D4z 16z2C12zC3 )x2C40 mod 16z2C12zC3/)x2 mod 16z2C12zC3/) 4là số phương mod 16z2C12zC3/nên sử dụng kí hiệu Jacobi ta có
4
16z2C 12z C 3
J
(142)Ta lại có:
4
16z2C 12z C 3
J D
1
16z2C 12z C 3
J
4
16z2C 12z C 3
J D
1
16z2C 12z C 3
J
4
16z2C 12z C 3
J D
1
16z2C 12z C 3
J
2
16z2C 12z C 3
J D
1
16z2C 12z C 3
J
D 1/16z2C12z2 C3 1 1/8z2C6zC1 D 1: ./ Ta thấy (*) mâu thuẫn với (**) suy phương trình vơ nghiệm
Ví dụ 4.2 Giả sửnDa2Cb2;ulà ước củanvàu3 mod4/.Chứng minh rằngulà ước củaavàb.
Chứng minh (Phản chứng) Giả sửukhông ước củaahoặcukhông ước củab; chẳng hạn: ukhơng ước củaa, suy ra.aIu/D1 Do đó, tồn số nguyêny,zsao choayCuzD1hay ay modu/(1)
Vìulà ước củanDa2Cb2nêna2Cb2 mod u/
Suy ra1C.by/2 ay/2C.by/2Dy2.a2Cb2/0 mod u/
Từ suy phương trình:1Cx2 mod u/có nghiệm (nghiệmx Dby)
Suy rax2 mod u/(vớiu mod 4/): vơ lí ! (Vì: 1là số phương modulou khiu mod 4/)
Ví dụ 4.3 Cho 5số nguyên dương:z,y,k,m,n Chứng minh rằng:xmCynkhông chia hết cho4kxy–1.
Chứng minh Giả sửuD4kxy–1là ước số củaxmCyn
Nếum,ncùng chẵn:m D 2m1In D2n1, suy raxmCyn D.xm1/2C.yn1/2 chia hết chounên.xm1/2
yn1/2 .mod u/.
Suy y n1/2 u
!
J
D )
1 u
J
D , u 1.mod4/: Điều trái giả thiết u mod 4/
Nếum,nkhơng tính chẵn lẻ: Chẳng hạn:mD2m1;nD2n1C1, suy raxmCynD xm1/2Cy.yn1/2 chia hết chou.
Do đó.xm1/2 y.yn1/2 .mod u/: Suy
y.yn1/2 u
!
J
D1) y u
J
.yn1/2 u
!
(143)Tạp chí Epsilon, Số 05, 10/2015
) y u
J
yn1 u
2 J
D1) y u
J D1 /
+) Nếu y lẻ ta có y u J D 1 u J y u J (1)
Dou mod 4/nên 1
u
J
D 1(2)
Vì.yIu/D1vày,uđều lẻ nên theo luật thặng dư bình phương có: y u J: u y J
D 1/y21: u
2 D 1/ y
2 :.2kxy 1/D 1/ y
2 1/ .3/
Vìu mod y/, suy u y J D 1 y J ; mà 1 y J
D 1/y21 nên u
y
J D 1/y21 (4).
Từ (3), (4) suy ray u
J D1) y
u
J D 1: mâu thuẫn (*)
+) Nếuychẵn:y D2h:z đóhlà số nguyên dương,zlà số nguyên lẻ Khi y
u
J D
2hz u J D 2h u J z u J D 2 u h J z u J Mà 2 u J
D 1/u281 D1; z
u
J D
suy y u
J D 1: mâu thuẫn (*) Nếum,ncùng lẻ:mD2m1C1;nD2n1C1 Suy raxmCynDx.xm1/2
Cy.yn1/2chia hết chou nênx.xm1/2 y.yn1/2 .mod u/)x2.xm1/2 xy.yn1/2 .modu/ ) xy
u
J D1:
Ta có xy u J D x u J y u
J D 1: mâu thuẫn!
5 Bài tập thực hành
Bài tập 5.1 Sử dụng kí hiệu Legendre tính chất liên quan đến kí hiệu để tính: a/ 31 641 b/ 111 991 c/ 105 1009 Bài tập 5.2 Chứng minh
(144)Bài tập 5.3 Tìm tất số nguyên tố lẻpsao cho10là số phương modp Bài tập 5.4 Chop số nguyên tố ;p mod 8/và p
2 số nguyên tố.Chứng tỏ rắng: p
2 số phương modp Bài tập 5.5 Chứng minh
1
2 61
D
2
29 541
D
3
113 137
D
4
5 1583
D 1:
Bài tập 5.6 Choplà số nguyên tố Mersenne;p Chứng tỏ rắng:3là số phi phương modp
Bài tập 5.7 Cho số nguyên dươngbvà số nguyên tố lẻp không ước củab Chứng minh rằng: b
p
C 2b
p
C 3b
p
C:::
.p 1/b p
D0:
Bài tập 5.8 Cho số nguyên tố lẻp D8:kC1 GọihDordp.2/ Chứng minh rằng:hlà ước p
2
Bài tập 5.9 Cho số nguyên tố lẻp D4kC1vàplà số phương modqvớiqlà số nguyên tố lẻ Chứng minh rằng:qlà số phương modp
Bài tập 5.10 Chop,qlà hai số nguyên tố sinh đôi vớiq DpC2 Chứng minh rằng: Tồn số nguyênasao choplà ước củaa2 q ,tồn số nguyênbsao choq ước củaa2 p Bài tập 5.11 Cho số ngun tốp Khi ln tồn số tự nhiênmsao cho:m < 1Cpp m số phi phương modp
Bài tập 5.12 Chứng minh rằng: phương trìnhx2–149y D107có nghiệm ngun Bài tập 5.13 Giải phương trình:2x2Cx 13 mod 37/
Bài tập 5.14 Cho số ngun tốp Phương trìnhx2C2y2 Dpcó nghiệm ngun.x; y/khi khipD2hoặcp 1; mod 8/
Bài tập 5.15 Chứng minh phương trình:x2 4x–80 mod23/có nghiệm ngun Bài tập 5.16 Chứng minh phương trình:x2 31 mod 71/khơng có nghiệm nguyên Bài tập 5.17 Tìm tất số nguyên tốpsao cho
(145)Tạp chí Epsilon, Số 05, 10/2015
4 Phương trìnhx2C9xC190 mod p/có nghiệm
Bài tập 5.18 Giả sửplà số nguyên tố lẻ,mlà số nguyên cho.m; p/D Tìm điều kiện cần đủ để phương trình:x2CpyCmD0có nghiệm ngun.xIy/
Bài tập 5.19 Nếu p số nguyên tố lẻ biểu diễn thành tổng hai số phương ,p 1.mod4/
Bài tập 5.20 Chứng tỏ rắng: Nếu phương trình:x2C101y Dzcó nghiệm ngun phương trình:x50–101y–1D0có nghiệm ngun
Bài tập 5.21 Choa; b; c số nguyên, choplà số nguyên tố lẻ không ước củaa; b; c Chứng minh phương trình:.x2 ab/.x2 bc/.x2 ca/Dpy ln có nghiệm nguyên xIy/
Bài tập 5.22 Choplà số ngun tố Chứng minh
1 Phương trìnhx2–2y2Dp có nghiệm khip D2hoặcp ˙1 mod 8/ Phương trìnhx2C2y2Dp có nghiệm khip D2hoặcp 1; mod 8/ Phương trìnhx2–3y2Dp có nghiệm nếup mod 12/
4 Phương trình3x2–y2Dp có nghiệm nếup D2; 3hoặcp 11 mod 12/ Bài tập 5.23 Cho số nguyên tố p dạng4k C1 Chứng minh rằng: x D
p 1
Š nghiệm phương trình:x2C10 mod p/
Bài tập 5.24 Cho p số nguyên tố ;p mod 4/; biết rằngq D2pC1cũng số nguyên tố Chứng tỏ rằng:2p–1chia hết choq
Bài tập 5.25 Chứng minh tồn vô hạn số nguyên tố dạng:3kC1,4kC1,10kC9 Bài tập 5.26 Đặt:
S D
21008 2017
C
21009 2017
C
21010 2017
C:::C
22015 2017
1 2017
C
2017
C
22 2017
C:::C
21007 2017
Chứng minh rằng:2017:S số phương
Bài tập 5.27 Chứng minh rằng: Vớix,y hai số nguyên số x 2
2y2C3 khơng số nguyên
Bài tập 5.28 Kí hiệuEnD11:::1là số tự nhiên mà cách viết hệ đếm số10thì Encónchữ số1 Hãy xét xemE33có chia hết cho67hay khơng?
Bài tập 5.29 Chứng minh rằng: Mn D 2n C1 (n 2) số nguyên tố 3Mn 12 1.modMn/
(146)1 Fnlà số nguyên tố,3Fn 12 1.modp/, đóplà số nguyên tố.
2 Nếuplà ước ngun tố củaFnthìpcó dạng:p D1C2nC2:k (trong đóklà số ngun dương đó)
Bài tập 5.31 Tìm tất cặp số nguyên dương.x; n/thoả mãnx3C2xC1D2n
(Đề thi Olympic Toán Serbia năm 2007) Bài tập 5.32 Chop số nguyên tố; chonlà số nguyên Nếu tồn số nguyênx, y chop Dx2Cny2 n/là số phương modulep
Bài tập 5.33 Cho số nguyên dươnga3 mod 4/ Khi tồn vô hạn số nguyên tốp mod 4/sao cho: phương trìnhx2 a mod p/khơng có nghiệm ngunx
Bài tập 5.34 Cho p số nguyên tố lẻ;m số nguyên dương;a số nguyên cho a; p/D1
1 Chứng tỏ rằng: Phương trìnhx2 a mod pmC1/ có nghiệm,Phương trìnhx2 a modpm/có nghiệm
2 Chứng tỏ rằng: Phương trìnhx2 a mod pm/có hai nghiệm,
a p
D1
Bài tập 5.35 Tìm tất số ngun tốpsao chopcó thể biểu diễn dạng:pDˇˇx2 3y2ˇˇ đóx,y số nguyên khác0
Bài tập 5.36 (USAMO 2008) Chứng minh rằng: Với số nguyên dương n tồn n số nguyên dương k1I k2I :::I kn nguyên tố đôi ki với i D1; 2; :::; nsao chok1:k2:::kn 1là tích hai số nguyên dương liên tiếp
Bài tập 5.37 (Serbia 2008)Tìm tất nghiệm ngun khơng âm phương trình:12xC y4 D2008z
Bài tập 5.38 (Ba Lan 2007)Chứng minh rằng: phương trình:x2C5Dy3 khơng có nghiệm ngun
Bài tập 5.39 Cho số nguyên dươngn Chứng minh số thặng dư bình phương mod2n là:
2n 1
C2
Bài tập 5.40 Cho số nguyên dươngnvà số nguyên tố lẻp => Số thặng dư bình phương modpnlà:
pnC1 1 2.pC1/
C1
6 Các ý lịch sử
(147)Tạp chí Epsilon, Số 05, 10/2015
Chú ý 2: Định lí tương hỗ thặng dư bậc hai tiên đoán Adrien Marie Legendre (vào cuối năm 1700) Leonhard Euler thời gian khoảng 40 năm cố gắng chứng minh không thành, cuối ơng nêu định lí giả thuyết (vào khoảng năm 1744) Nhưng nhà toán học Đức vĩ đại Johann Carl Friedrich Gauss người chứng minh định lí vào năm 1797, ông tuổi 19 (!) Gauss gọi ’định lý vàng’ tự hào đến mức ông tiếp tục tìm cách chứng minh khác cho định lí cuối đời
Cuốn Reciprocity Laws: From Euler to Eisenstein (Luật tương hỗ bậc hai: Từ Euler đến Eisentein) Franz Lemmermeyer, xuất năm 2000, thu thập trích dẫn cho 196 chứng minh khác định lý
Chú ý 3: Các tính chất tác giả trích dẫn từ tài liệu tham khảo (có danh mục cuối đề tài này) Việc khơng chứng minh tính chất nhằm giảm tính nặng nề hàn lâm cho dề tài mà quan tâm đến áp dụng chúng!
Adrien-Marie Legendre [1752 – 1833]:sinh ngày 18 tháng năm 1752 Paris, Pháp ngày 09 tháng năm 1833 Paris
Legendre sinh gia đình giàu có, học Học viện Paris Ma Zhalin Ông đào tạo lĩnh vực: khoa học giáo dục, giáo dục đặc biệt toán học Thầy giáo dạy toán ông J F M Abbe (Abbé) nhà tốn học kính trọng lúc Vào năm 1770 tuổi 18, Legendre bảo vệ luận án tiến sĩ tốn học vật lý, thơng qua bảo trợ Abbe
Điều kiện kinh tế giả, đủ để giúp Legendre tham gia vào nghiên cứu khoa học Tuy nhiên, vào 1775-1780 ông bắt đầu tham gia giảng dạy toán học trường quân Paris Công việc nghiên cứu ông ý cộng đồng khoa học, vào năm 1782 gia nhậpViện Cơ học, năm 1785 bổ nhiệm làm Viện trưởng, thay ch P S Laplace (Laplace) Năm 1787, ông bổ nhiệm làm Viện trưởng Viện Hàn lâm Khoa học Paris Đài thiên văn Greenwich
Năm 1794, ông bắt đầu với tư cách giáo sư Đại học toán học túy Ông năm 1813 Trong số học, ơng đốn luật bình phương nghịch đảo (quadratic reciprocity law), sau chứng minh Gauss Ơng có số cơng trình tiên phong phân bố số nguyên tố, ứng dụng giải tích vào số học Phỏng đốn ơng vào năm 1796 Định lý số nguyên tố chứng minh chặt chẽ Hadamard de la Vallée-Poussin vào năm 1898 Carl Gustav Jacob Jacobi(10/12/1804 – 18/02/1851): Là nhà toán học người Đức gốc
Do Thái, người có nhiều đóng góp cho lĩnh vực: hàm elliptic, động lực học, phương trình vi phân, lý thuyết số Tên ông viết Carolus Gustavus Iacobus Iacobi sách giáo hoa Latin ông viết tên ông Karl dùng
(148)Biểu tượng Jacobi tổng quát biểu tượng Legendre Được giới thiệu Jacobi vào năm 1837, quan tâm mặt lý thuyết số học mô-đun ngành khác lý thuyết số, việc sử dụng lý thuyết số tính tốn, test số ngun tố phân tích thành nhân tử nguyên tố; ứng dụng quan trọng lí thuyết mật mã Jacobi người áp dụng hàm elliptic vào lý thuyết số, ví dụ chứng minh định lý Fermat phân tích số thành tổng số phương định lý Lagrange phân tích số thành tổng số phương, kết tương tự cho phân tích số thành tổng số phương Trong lý thuyết số ông tiếp tục công việc C F Gauss: chứng minh luật tương hỗ bậc hai giới thiệu kí hiệu Jacobi; góp phần lớn phát minh cho luật tương hỗ, tìm kiếm tính chất liên phân số, phát minh tổng
7 Thay cho lời kết
Sự ghi nhận tơn kính:Lịch sử ngành Lí thuyết số (Number Theory) nói chung Lí thuyết số k–phương nói riêng ghi nhận cơng lao to lớn lỗi lạc thiên tài Toán học như: Carl Friedrich Gauss(1777-1855):
Nhà toán học kiệt xuất nước Đức, từ tuổi tỏ rõ tài tốn học phi thường
30 tuổi giáo sư toán học trường đại học Găottingen Nm 1804 ụng tr thnh thnh viờn Vin Hn Lâm khoa học Anh Những cống hiến to lớn Gauss bao trùm lên tồn lĩnh vực tốn học Chính Gauss làm thay đổi mặt tốn học nên giới cơng nhận ơng nhà toán học vĩ đại lịch sử lồi người, gọi "Ơng Hồng tốn học"
Leonhard Euler (1707 – 1783):
Sinh năm 1707 Basel, thành phố nhỏ tuyệt đẹp ven bờ sông Ranh (Rhin) Thụy Sĩ
Khả toán học Euler bộc lộ sớm Năm 13 tuổi, cậu bé sinh viên trường đại học Tổng hợp Basel (Thụy Sĩ) Năm 1731, chàng niên Euler 24 tuổi trở thành viện sĩ Viện hàn lâm Petersburg (Pê-tec-bua)
(149)Tạp chí Epsilon, Số 05, 10/2015
sau mổ vài ngày, ơng lại lao vào làm việc, tính tốn khơng nghỉ nên mắt trái hỏng lại từ đó, ơng bị mù hẳn Năm đó, ơng phải chịu nhiều bất hạnh: nhà cháy, cải hết Rồi hai năm sau, bà Euler qua đời Người ta tưởng từ ơng phải giã từ khoa học Nhưng tình u ơng Tốn học khơng giảm sút sức mạnh sáng tạo óc thiên tài nơi ơng thật vĩ đại Với trí nhớ kì diệu, mù, ơng đọc cho thư kí viết phát minh Trong Tốn học, khơng có nhà Toán học nhắc đến nhiều Euler Những ngày cịn học phần logarit lượng giác chương trình phổ thơng hồn tồn dựa theo cách trình bày Euler Ơng cịn người đề nhiều kí hiệu Tốn học, chẳng hạn quen thuộc với kí hiệu số (pi) Nhà toán học Pháp La-pla-xơ (Laplace) gọi ông "người thầy chung tất chúng ta"
Cuộc đời Euler gương sáng chói lịng say mê lao động sáng tạo khơng mệt mỏi, vượt lên ngăn trở bất hạnh ngẫu nhiên; Khi hỏng mắt, 17 năm cuối đời, ông hồn tất 416 cơng trình khoa học, tức trung bình năm nghiên cứu thành cơng 25 cơng trình có giá trị xuất sắc!
Ơng từ trần vào mùa hè năm 1783 Ơng để lại 865 cơng trình khoa học, in thành 72 tập lớn, tập ngót 600 trang Sau ơng mất, Viện hàn lâm Petersburg, công bố thảo ông khoảng 47 năm hết
Pierre de Fermat (sinh ngày 20 tháng 8, 1601 Pháp – 1665):
Là học giả nghiệp dư vĩ đại, nhà toán học tiếng cha đẻ lý thuyết số đại
Xuất thân từ gia đình giả, ơng học Toulouse lấy cử nhân luật dân làm chánh án Chỉ trừ gia đình bạn bè tâm giao, chẳng biết ơng vơ say mê tốn Mãi sau Pierre de Fermat mất, người trai in dần cơng trình cha kể từ năm 1670 Năm 1896, hầu hết tác phẩm Fermat ấn hành thành tập dày Qua đó, người đời vô ngạc nhiên khâm phục trước sức đóng góp dồi ơng Chính ơng người sáng lập lý thuyết số đại, có định lý bật: định lý nhỏ Fermat định lý lớn Fermat (định lý cuối Fermat)
Tài liệu tham khảo
[1] "250 Problems in Elementary Number Theory," Waclaw Sierpinski, 1994 [2] "Elementary Number Theory," David M Burton, 1980
[3] "Elementary Number Theory," Giusefe Melfi, 1998 [4] "Elementary Number Theory," A.J Hildebrand, 2011
[5] "Number Theory Structures, Examples, and Problems," Titu Andreescu Dorin Andrica [6] "Quadratic Congruences Olympiad Training," Dusan Djukic
[7] "Elementary Number Theory", Beuker, 2012 [8] "Số học," Hà Huy Khoái, NXB Giáo Dục, 2006 [9] Một số viết đồng nghiệp mạng1[sic]
[10] Các giảng Số học Nguyễn Hồng Lữ (tài liệu lưu hành nội bộ)
1
(150)(151)VỀ MỘT PHÂN HOẠCH TẬP CÁC SỐ TỰ NHIÊN THÀNH HAI TẬP HỢP CÓ TỔNG CÁC PHẦN TỬ
BẰNG NHAU
Nguyễn Văn Lợi - Nguyễn Hải Đăng - Nguyễn Thành Khang (Đại học Tổng hợp Budapest, Hungary)
LỜI GIỚI THIỆU
Bài viết cần thỏa mãn số điều kiện biểu diễn đơn giản, xây dựng cầu trung chuyển thuật tốn xếp ba lơ thuật tốn ăn tham Nhờ đó, số kết phân hoạch tập số nguyên dương thành hai tập hợp có số lượng phần tử chứng minh
1 Mở đầu
Từ năm 1990, nghiên cứu toán tối ưu toán học rời rạc bắt đầu phát triển Việc phân bổ tập hợp theo điều kiện cho trước, nhiều lần thuật toán xếp ba lơ thuật tốn tham ăn sử dụng Bài tốn xếp ba lơ phát biểu sau: tìm cách chọn đồ vật để xếp vào hai ba lô ba lô chứa nhiều đồ
Thuật tốn xếp ba lơ dùng để giải tốn diễn giải sau: trước tiên, ta xếp đồ vật theo thứ tự giảm dần khối lượng Tiếp đó, ta ta xếp vào ba lơ vật Sau lần xếp, người ta lại kiểm tra xem ba lơ cịn nhiều chỗ hơn, ưu tiên xếp trước Tiếp tục trình ta nhận cách xếp tối ưu Về thuật toán tham ăn, nội dung là: ta xếp vào ba lơ khơng cịn bỏ thêm nữa, sau thay đổi vị trí đồ vật từ ba lơ sang ba lơ kia, để hợp lý hóa cơng việc xếp (xem [1, 2, 3, 4, 5] tài liệu tham khảo đó)
Trong này, sử dụng phương pháp trung gian lưu chuyển hai thuật toán Trước xếp chọn đồ vật nhỏ nhất, gọi tập hợpK;với mục đích: tham ăn tương đối đầy ba lơ, ta dùng đồ vật nhỏ từK;để tiếp tục chèn vào lỗ hổng, đầy ba lô Một tập hợp vật nhỏ gọi biểu diễn đếnk;nếu vật nặng từ1(nhỏ) đếnk(đủ nặng) biểu diễn tổng đồ vật lấy từ tậpK:
2 Phát biểu toán
(152)Trong phạm vi này, tập hợp xét đa tập hợp làm việc với đa tập hợp hữu hạn số nguyên dương
Ngoài ra, tập hợpS gọi phân hoạch thành tập hợpA1; A2; : : : ; Ak (
A1[A2[ [Ak DS Ai \Aj D; 1i < j k
Bổ đề kết tiếng sử dụng nhiều trình chứng minh Bổ đề Từnsố nguyên cho trước, chọn vài số để tổng chúng chia hết chon.
Chứng minh Ký hiệu tập hợp n số nguyên số AD fa1; a2; : : : ; ang: Xét tổng sau
s1Da1; s2Da1Ca2;
: : :
snDa1Ca2C Can Chia số
fs1; s2; : : : ; sng
cho sốnta đượcnsố dư thuộc tập hợpf0; 1; 2; : : : ; n 1gNếu có số dư nói bằng0ta suy điều phải chứng minh Trái lại, giả sử số dư thuộc tập hợpf1; 2; : : : ; n 1g
Áp dụng nguyên lý Dirichlet, ta thấy tồn hai số dư Giả sửsk sj modn/với k > j:Ta suy
sk sj DajC1CajC2C: : :Cak mod n/: Bổ đề chứng minh
Trong bổ đề trên, ta thấy khơng có ràng buộc số lượng phần tử số chọn Dưới định lý tiếng liên quan đến vấn đề này, yêu cầu phải chọn có số lượng phần tử cụ thể
Định lý (P Erdos, A Ginzburg, A Ziv) Từ2n 1số nguyên cho trước, chọn đượcnsố sao cho tổng chúng chia hết chon:
Định lý chứng minh năm 1961 Bạn đọc tham khảo [3,4,5] Tiếp theo, ta xét bổ đề phụ nữa:
Bổ đề Nếu trongn 1số ngun dương cho trước khơng tồn nhóm số có tổng chia hết chonthìn 1số có số dư chia chon:
Chứng minh Giả sửn 1số cho làa1; a2; : : : ; an Ta chứng minh phương pháp phản chứng Giả sử ngược lại, tồn hai số khơng có số dư Khơng tính tổng qt, giả sử số làa1; a2 Đặtsi Da1Ca2C Cai 1Cai; 2i n Xét dãy số
a1; a2; s2; s3; : : : ; sn
„ ƒ‚ …
n 2số
(153)Tạp chí Epsilon, Số 05, 10/2015
trong cónsố khơng có số chia hết chon:Theo nguyên lý Dirichlet, tồn hai số có số dư chia chon:
Xột hiu ca hai s ny Vỡa1 a2 Ô0 modn/nên hiệu củasi số tronga1; a2hoặcsj Trong trường hợp, hiệu tổng vài số trongn số ban đầu Do đó, tất cản 1có số dư chia chon:
Bổ đề chứng minh
Trong phần tiếp theo, cho tập hợpAcókphần tử số nguyên dương không lớn hơnN tổng số bằng2N:
Bài toán Tồn hay không giá trịK nhỏ để với số ngun dươngk K;tập hợp
Aln phân hoạch thành hai tập có tổng phần từ tập bằngN ‹ Từ sau, ta ký hiệu
AD (
a1< a2 < : : : < ak ˇ ˇ ˇ ˇ ˇ
ai ZC; n; i D1; k;
k X
iD1
ai D2N )
:
Tập hợpAthỏa mãn điều kiện viết làA.2N; k/
Ta ký hiệu dxe D minfn2Zjnxg số nguyên dương bé khơng nhỏ x Ví dụ,
d3e D3;d3; 5e D4;d 2; 1e D 2; : : :
Định lý sau cho chặn sốK Hơn nữa, nếuN lẻ giá trịK DN C1là giá trị nhỏ cần tìm
Định lý Mọi tập hợp gồmN C1số nguyên dương không lớn hơn N và có tổng bằng2N;
ln phân hoạch thành hai tập con, tập có tổng phần tử bằngN:
Chứng minh TừN C1số cho, ta lấyN số Theo bổ đề 1, ta chọn từN số vài số có tổng chia hết choN:Tổng nhỏ hơn2N dương bằngN: Phần bù tập nêu có tổng phần tử bằngN:
Định lý chứng minh
Từ định lý nêu trên, ta thấy nếuN lẻ, số phần tử làk DN, ta xét tập hợp sau đây:
AD <
:
2I2I2I: : :I2 „ ƒ‚ …
Nsố
9 =
; :
Tổng phần tử củaAbằng2N;nhưng tất phần tử củaAlà chẵn, nênAkhông thể phân hoạch thành hai tập có tổng bằngN (là số lẻ) Điều chứng tỏK DN C1là giá trị cần tìm khiN số lẻ
Trường hợpN số chẵn, tốn khó chứng minh Ta trường hợp N nhỏ
(154)2 Trường hợpN D4:Ta chứng minhK D4:Thực vậy, xét tập hợpAD f3I3I2g:Nhận thấy không tồn tập A có tổng phần tử bằng4:Từ ta suy raK 4: Xét bốn số1a1 a2 a3 a4 4, vớia1Ca2Ca3Ca4D8 Ta chứng minh tồn nhóm số có tổng bằng4:
Trường hợpa4 D4 Điều chứng minh hiển nhiên
Trường hợp a4 D 3, suy a1 Ca2 Ca3 D nên a1 Do đó, a1Ca4 D4
Trường hợpa4 D2, suy raa1 Da2Da3 D2 Ta có điều cần chứng minh Vậy, vớiN D4thìK D4:
3 Trường hợpN chẵn vàN 6là trường hợp phức tạp, trước bắt tay vào giải quyết, ta nêu số bổ đề nhỏ làm cầu nối
Ta xem xét bổ đề sau:
Bổ đề VớiN D2nvàAlà tập hợp2n 1số nguyên dương không lớn hơnN và có tổng bằng2N:Khi đó,Acó thể phân hoạch thành hai tập con, tập có tổng phần tử bằngN:
Chứng minh Theo định lý 1, tồn tạinsố thuộcAvà có tổng chia hết chon:Tổng số đạt giá trị bằngnhoặc2nhoặc3n:Nếu tổng số bằng2nDN ta phân hoạch đượcAthành hai tập có tổng phần từ bằngN:Ta cần xét trường hợp cịn lại
Gọi n số có tổng chia hết cho n a1 a2 : : : an n số lại A b1 b2 : : :bn Ta đặt
aDa1Ca2C Canvàb Db1Cb2C Cbn 1: Ta xét2trường hợp sau
1 Trường hợpa D nvàb D 3n:Khi đó, ta cóa1 D a2 D : : :D ak D1 Vì b D 3nnên bn
Nếubn n, ta chọnbn 1và số số1để số có tổng bằngN: Nếu3bn Dm < nthìB D fbn 1; an; an 1; : : : ; an mC1gsẽ có tổng phần tử bằngn:Xét nsốa1; a2; b1; b2; : : : ; bn Theo Bổ đề 1, ta chọn tậpC gồm số số có tổng chia hết chon:Nhận thấy, tổng phần tử củaC bằngn hoặc2n:Ta lại tiếp tục xét2trường hợp:
– Nếu tổng phần tử củaC bằng2n;ta phân hoạchAthànhC vàAnC;mỗi tập có tổng phần tử bằngN:
– Nếu tổng phần tử củaC bằngn; ta phân hoạchAthànhD D B [C AnD;mỗi tập có tổng phần tử bằngN:
(155)Tạp chí Epsilon, Số 05, 10/2015
Nếu chọn số số trongn 1đó có tổng chia hết chon;thì tổng số chọn bằngnhoặc bằng2n:Ta lại có trường hợp:
– Nếu tổng số chọn bằng2n;thì tập số tạo thành tập Acó tổng phần tử bằng2nDN:
– Nếu tổng số chọn bằngn;thì tập số với tập hợpfb1; b2; : : : ; bn 1g hợp thành tập củaAcó tổng phần tử bằng2nDN:
Nếu trongn 1số bất kỳ, khơng chọn số số có tổng chia hết chonthì theo Bổ đề 2,n 1số có số dư chia chon:Chọn bộn 1số khác, cách lập luận tương tự, ta nhận đượcai aj modn/với mọi1i < j n Ta xét trường hợp sau:
– Nếuai modn/với mọi1i nthì
a1 Da2 D: : :Dan D1vàan Dan DnC1: Khi tập hợp
fb2; b3; : : : ; bn; ang có tổng phần tử bằng2nDN:
– Nếuai modn/với mọi1i nthì
a1 Da2D: : :Dan D2vàan DnC2: Khi tập hợp
fb1; b2; b3; : : : ; bn 1; ang có tổng phần tử bằng2nDN:
– Nếuai modn/với mọi1i nthì
a1Da2 D: : :Dan D3:
Khi dễ dàng chọn tập A có tổng phần tử bằng2nDN: Mệnh đề 2.1 chứng minh hoàn toàn
Chú ý ta sử dụng thuật toán ăn tham chứng minh bổ đề
Từ bổ đề trên, ta suy giá trịK cần tìm thỏa mãnK N 1trong trường hợpN chẵn, nhiên việc tìm giá trị K cịn nhiều khó khăn
Dưới ta số chặn củaK:Ta xét ví dụ sau:
1 VớiN D6mC2thìK 4mC3 Để có phản ví dụ vớikD4mC2;ta chọn tập hợp
AD <
:
1; 3; 3; : : : ; „ ƒ‚ … 4mC1số
9 =
; :
(156)2 VớiN D6mC4thìK 4mC4:Để có phản ví dụ vớik D4mC3;ta chọn tập hợp
AD <
:
2; 3; 3; : : : ; „ ƒ‚ …
4mC2 =
;
Khi đó, phân hoạch tậpAsẽ cho ta tập có tổng phần tử chia hết cho3 tập lại có tổng phần tử chia cho3dư2:Điều chứng tỏK 4mC4: VớiN D6mthìK 3mC2:Để có phản ví dụ vớik D3mC1;ta chọn tập hợp
AD <
:
2; 2; 2; : : : ; „ ƒ‚ …
3m 1số
; 6m =
;
Khi khơng tồn tập củaAcó tổng phần tử N D 6m:Điều chứng tỏK 3mC2:Để tiếp tục nghiên cứu khả phân hoạch của2N;chúng ta phải phân tích sâu có mặt phần tử tạo thành tậpAtrong mục
3. Cấu trúc tập hợp số khả chia đôi
Trong phần xây dựng lý thuyết nhỏ để làm cầu nối hai thuật tốn: thuật tốn xếp ba lơ thuật tốn ăn tham Ý tưởng thuật toán xem xét giá trị phần tử nhỏ củaA;qua kết hợp hai thuật tốn để giải toán
Tập hợpAcác số nguyên dương gọi biểu diễn đếns với số nguyên dươngt khơng vượt qs tồn tập củaAcó tổng phần tử bằngt:Tập hợpAđược gọi hoàn chỉnh nếuabằng tổng phần tử củaA;thìAbiểu diễn đếna:
Bổ đề ChoAlà tậpksố nguyên dương không vượt quáN vàAbiểu diễn đếnN:Khi đóAlà tập hồn chỉnh.
Chứng minh Giả sửAD fa1; a2; a3; : : : ; akgvàaD a1Ca2C Cak Ta chứng minh quy nạp theoN:
XétN D1;suy raai D1vớii D1; 2; : : : ; k Khi đóAlà tập hồn chỉnh Giả sử bổ đề vớiN:Ta chứng minh bổ đề vớiN C1:Thực vậy, xét
AD fa1; a2; a3; : : : ; akg
là tập gồmksố nguyên dương không vượt qNC1:Nếu trongAkhơng có số bằngNC1; theo giả thiết quy nạp,Asẽ tập hoàn chỉnh
Ngược lại, giả thiết trongAcó số số bằngN C1:Giả sử
N C1Da1 Da2 D: : :Dai > aiC1aiC2: : :an:
Đặtb DaiC1CaiC2C Can Nhận thấy biểu diễn số nguyên dươngs N thành tổng số số trongAthì khơng có số bằngN C1xuất tổng đó, nên tập
B D faiC1; aiC2; : : : ; akg
(157)Tạp chí Epsilon, Số 05, 10/2015
1 Vớis b, theo giả thiết quy nạp,s biểu diễn thành tổng số số trongB: Xétb < s a:Khi đó, tồn tạiq; r 2N chos bDq.N C1/ r;với0r N:
Dễ thấy0qi;vì ngược lạiqiC1thì
s i C1/.N C1/Cb r DaC.N C1/ r > a:
Nhận thấy rằngb r biểu diễn thành tổng số số trongB:Khi đó, ta cần bổ sung thêm vào đóq sốN C1sẽ số số trongAcó tổng bằngs:
Do đó, bổ đề vớiN C1nên theo nguyên lý quy nạp, bổ đề chứng minh VậyAlà tập hoàn chỉnh với mọiN ZC:
Từ Bổ đề trên, ta dễ dàng chứng minh bổ đề sau
Bổ đề NếuAvàB là hai tập hồn chỉnh thìC DA[B là tập hồn chỉnh.
Nhận thấy rằng12 Akhi vả tồn tập hoàn chỉnh củaA:GọiH hợp tất tập hoàn chỉnh củaA:Theo Bổ đề 4,H tập hoàn chỉnh A:
Hơn nữa,H tập hồn chỉnh có số phần tử lớn củaA:Vậy là, ta thu kết sau
Bổ đề ChoAlà tập số nguyên dương và12 A Khi đó, tồn tập hồn chỉnh củaAcó số phần tử lớn Gọi tập hoàn chỉnh làH vàhlà tổng phần tử của
H:Nếu tất phần tử củaAđều khơng vượt q h thìH DA:Nếua Avàa … H thì
ahC2.
Chứng minh Chứng minh Thực vậy, nếuaDhC1thìH [ faglà hồn chỉnh Điều trái với tính lớn H
Bổ đề Nếu tậpA.2N; k/vớik N
2
C2thìAcó ba phần tử có giá trị nhỏ hơn4:
Chứng minh Giả sử phản chứng có nhấtk 2phần tử A khơng bé hơn4:Ta có 2N 2C4.k 2/2C4
N
> 2N Điều vơ lý Do đó, có ba phần tử củaA có giá trị nhỏ hơn4:Vìa1 a2 a3là ba phần tử bé củaAnên ta có
.a1; a2; a3/2 f.1I1I1/; 1I1I2/; 1I1I3/; 1I2I2/; 1I2I3/; 1I3I3/; 2I2I2/; 2I2I3/; 2I3I3/; 3I3I3/g
Bổ đề chứng minh
Định lý Cho A.2N; k/vớiN 2vàk N
2
(158)Chứng minh Xét trường hợpN Theo Bổ đề 4, ta cóH DA Bởi vậy, phân hoạch tậpAthành hai tập có tổng phần tử bằngN:GọiS tổng phần tử củaH
AnH D˚b1; b2; : : : ; bp :
Áp dụng Bổ đề 5, ta được2N DS Cb1Cb2C Cbp vàbi S C2 Gọiq số phần tử củaH:Khi đók DpCq;vàS q:Ta có
2N DS Cb1Cb2C Cbp S Cp.S C2/DS C.k q/.S C2/ SC.k S /.SC2/D S2C.k 1/S C2k
S2C N
2
C1
SC2 N
2
C4
Ta suy
f S /D S2C N
2
C1
S C2 N
2
C4 2N 0:
Mặt khác,
f 3/D 9C3 N C1 C2 N
C4 2N D5 N
2
2N >
vớiN > 4và f N D N 2 C N C1 N C2 N
C4 2N D4
N
C2 2N >
Kết hợp điều vừa thu vớiS 3ta suy raS
N
:Khi
2N S Cp.S C2/ N Cp N C2 :
Do đóp 2;ta xét trường hợp:
1 Nếup D1thìb1D2N S 2N a1 2/:Ta suy rab1 N C1;điều vơ lý Nếup D2thì
S q Dk N
2
vàb1; b2 SC2 N
2
C2:
Ta có
N b1 N N 2 < N S
(159)Tạp chí Epsilon, Số 05, 10/2015 Phép chứng minh hoàn tất
Xéta1 D1 Kết hợp với Bổ đề 5, ta có
.a1; a2; a3/2 f.1I1I1/; 1I1I2/; 1I1I3/; 1I2I2/; 1I2I3/g Ta có định lý sau đây:
Định lý ChoA.2N; k/vớiN 2; k
N
C2và
.a1; a2; a3/2 f.1I1I1/; 1I1I2/; 1I1I3/; 1I2I2/; 1I2I3/g:
Khi đó, tập hợpAcó thể phân hoạch thành hai tập có tổng phần tử bằngN:
Trong định lý trên, với tậpA.2N; k/màN 2; k N
2
C2; a1 D1vàAbiểu diễn đến3;thìAcó thể phân hoạch thành hai tập có tổng phần tử bằngN:Ta xem xét khả phân hoạch tậpAthành hai tập có tổng phần tử N khia1 2:
Bổ đề Cho tậpA.2N; k/vớiA.2N; k/màk N
2
C2; a12Khi đó, hai số của
Acó tổng không vượt quáN:
Chứng minh Ta cần chứng minhak 1Cak N:Thật vậy, ta có a1Ca2C: : :Cak 2.k 2/2
N
2
N
Từ ta suy raak 1Cak N:Mệnh đề chứng minh Định lý Cho tậpA.2N; k/vớik
N
C2,N chẵn vàa1 D a2 D 2 Khi ta có thể phân hoạch tập A thành hai tập có tổng bằngN:
Chứng minh Thực phân hoạchADC [L, C D fc1; c2; : : : ; cug tập tất số chẵn A
LD fl1; l2; : : : ; lvg
là tập tất số lẻ củaA:Ta cóu 2; uCv Dkvà v số chẵn Đặtv D2t:Xét tập hợp B D fb1; b2; : : : ; buCtg
được xác định sau:
bi D Ci
2 với1i u
buCj D
lj Clv j
(160)Nếua4 4thì2N 6C4.k 3/6C4 N D4 N
C2 > 2N Điều vô lý Do đó, ta cóa43 Từ suy raa33 Suy ba phần tử bé củaB là.1I1I1/; 1I1I2/ hoặc.1I1I3/ Nhận thấy tổng phần tử củaB bằngN dễ dàng thấy phần tử củaB khơng vượt q N
2 Để áp dụng Định lý cho tậpB;ta cần chứng minh uCt
N
C2
Thực vậy, ta có
uCt D2Cu 2C v
2 2C
uCv 2 C
u 2 2C
2
N
C u
2 D2C N
4 C u
2 Ta có trường hợp:
1 NếuN chia hết4thì
N
D N
4 vàuCt 2C
N
C u
2 2C
N
2 Nếunkhông chia hết4thì
N C2 D
N
Khi
uCt 2C N
4 D2C
N C2
1
2 D2C N
DouCt số nguyên, nênuCt 2C
N
Áp dụng Định lý cho tậpB.N; uCt /với uCt 2C
N
thìB phân hoạch thành hai tập có tổng phần tử N Từ ta suy ra,Acũng phân hoạch thành hai tập có tổng phần tử bằngN: Định lý chứng minh
Kết hợp kết Định lý 2, 3, ta có định lý sau Định lý Cho tậpA.2N; k/vớik
N
2
C2;kchẵn vàa2 2 Khi ta phân hoạch tậpAthành hai tập có tổng bằngN:
4 Áp dụng
(161)Tạp chí Epsilon, Số 05, 10/2015
Chứng minh Ta cókD4mC33mC3D N
2
C2 Ta có2trường hợp:
1 Nếua3 3thìa3Ca4C: : :Cak 3.k 2/D12mC3 Ta suy raa1Ca21:Điều vô lý
2 Nếua3 Ta suy raa2 Bây giờ, ta áp dụng Định lý thìAcó thể phân hoạch thành hai tập có tổng phần tử bằngN:
Hệ chứng minh
Hệ 4.2 Cho tậpA.2NIk/vớiN D6mC4vàk D 4mC4:TậpA.2N; k/có thể phân hoạch thành hai tập có tổng phần tử nhau.
Chứng minh Ta cókD4mC43mC4D
N
C2 Ta xét2trường hợp:
1 Nếua3 3thìa3Ca4C Cak 3.k 3/D12mC6 Ta suy raa1Ca2 2:Do đóa1 Da2D1; a3 Da4 D: : :Dak D3 Bởi vậy, ta dễ dàng phân hoạch tậpA thành hai tập có tổng phần tử bằngN:
2 Ta xét trường hợpa32, đóa2 Áp dụng Định lý thìAcó thể phân hoạch thành hai tập có tổng phần tử bằngN:
Hệ chứng minh
Hệ 4.3 Cho tậpA.2NIk/vớiN D6mvàkD3mC2:TậpA.2N; k/có thể phân hoạch được thành hai tập có tổng phần tử nhau.
Chứng minh Trước hết, trường hợpa22là hệ trực tiếp Định lý 6, màAcó thể phân hoạch thành hai tập có tổng phần tử bằngN:Xét trường hợpa2 3:Ta chứng minha3 Da4 D3:Nếua4 4thì
a2Ca3C Cak 6C4.k 3/12mC2 Điều vơ lý Từ ta suy
a2 Da3Da4 D3:
Ta phân hoạchADC [K, đóC D fc1; c2; : : : ; cuglà tập số chia hết cho3củaA vàK D fk1; k2; : : : ; kvglà tập số khơng chia hết cho3củaA:Khi đóu 3IuCv Dk: Ta phân hoạch
K DK1[K2[ [Kt
sao cho tổng phần tử củaKi chia hết cho3với mọiivàt lớn Theo Bổ đề ba số nguyên tồn số số có tổng chia hết cho3:Bởi vậy,jKij 3;8i Từ ta suy rat v
3
Giả sử tổng phần tử củaKi bằngdi , với1i t:Xét tập hợp B D fb1; b2; : : : ; buCtg
trong đóbi D Ci
3 với1i uvàbuCj D dj
(162)Vìu3nên ba phần tử bé củaBlà.1I1I1/:Nhận thấy tổng phần tử củaB bằng4m: Ta chứng minhbi 2mvới mọii:Thật vậy, ta cần chứng minh
ak 2Cak 1Cak 6mC2:
Khi đó, từci; di 6mta suy rabi 2m:Ta có
a1Ca2C Cak 1C3.k 4/D1C3.3m 2/D9m 5:
Suy
ak 2Cak 1Cak 3mC56mC2:
Để áp dụng Định lý cho tập B, ta cần chứng minh:uCt mC2:Thực vậy, ta có uCt D3Cu 3C v
3 3C
uCv
3 3C
3m
3 mC2:
Áp dụng Định lý cho tậpB.4m; uCt /vớiuCt mC2thìB phân hoạch thành hai tập có tổng phần tử bằng2mD N
3 Từ đó,Acũng phân hoạch thành2tập có tổng phần tử bằngN:
Hệ chứng minh
Từ kết chặn tìm Mục 1, toán đặt giải trọn vẹn Ta có định lý sau
Định lý Cho tập hợpAcókphần tử số nguyên dương không lớn hơnN và tổng của số bằng2N:Khi đó, tồn giá trịK nhỏ để với mọik K;tậpAln có thể phân hoạch thành hai tập có tổng phần từ tập N, đó
K DN C1;khiN lẻ.
K D2;khiN D2:
K D4mC3;khiN D6mC2: K D4mC4;khiN D6mC4: K D3mC2;khiN D6m:
Cuối ta giải tập điểm xuất phát có tác dụng thúc đẩy tồn nghiên cứu Bài tốn Chứng minh trong35số ngun dương khơng vượt q50và có tổng bằng
100thì chọn nhóm số có tổng bằng50:
Chúng ta dễ dàng suy kết nhờ việc áp dụng kết toán lớn trường hợpN D50:Dưới cách chứng minh độc lập kết Áp dụng Bổ đề dễ dàng chứng minh hệ sau
(163)Tạp chí Epsilon, Số 05, 10/2015
Chứng minh Bằng cách phân hoạch tập số cho thànhA1IA2I: : :IAk cho tổng phần tử tậpAi chia hết cho5vàklớn
Áp dụng hệ ta thấy, tậpAi có khơng q5phần tử Suy rak7:
Giả sử tổng phần tử tậpAi bằng5ai Khơng tính tổng qt, giả sửa1 a2 a3 : : :ak Khi đóa1Ca2C Cak D20
1 Trường hợpkD7thì tậpAi có đúng5phần tử Áp dụng bổ đề 2, với tậpAi phần tử có số dư chia cho5:
Vìk lớn nên tất cả35số cho có số dư chia cho5;vì ngược lại ta chọn tập có hơn5phần tử có tổng phần tử chia hết cho 5:Mặt khác, 35 số cho có số1hoặc số2;nên tất số cho có dạng 5mC1hoặc tất số cho có dạng5mC2:
Nếu tất số cho có dạng5mC1, ta suy trong35số cho có nhất22số bằng1:Từ dẫn tới
a1 Da2 Da3 Da4 D1: Ta có
a5Ca6Ca7 D17 suy raa7 Ta có2trường hợp:
– Nếu6a7 10thì ta bổ sung vào tậpA7 số1để tập Acó tổng phần tử bằng50:
– Nếua7 11thì tồn tập B tậpA7sao cho tổng phần tử B khơng nhỏ hơn28:Khi ta bổ sung vào tậpB số1để tập củaAcó tổng phần tử bằng50:
Nếu tất số cho có dạng5mC2;thì ta suy trong35số cho có nhất29số bằng2:Từ dễ dàng chọn tập củaAgồm25số2và có tổng phần tử bằng50:
2 Trường hợpk8;ta cóak 13
Nếuak D13thì tậpAnAk gồm nhất30số có tổng bằng35:Khi tập AnAk tồn nhất25số1:TậpAk có tổng phần tử bằng65nên tồn tập tập conB tậpAk cho tổng phần tử củaB không nhỏ 33:
Khi đó, ta bổ sung vào tậpB nhiều nhất17số1để tập củaAcó tổng phần tử bằng50:
Nếuak D12thì tậpAnAk gồm nhất30số có tổng bằng40:Khi tập AnAk tồn nhất20số1:TậpAk có tổng phần tử bằng60nên tồn tập tập conB tậpAk cho tổng phần tử củaB không nhỏ 30:
Khi đó, ta bổ sung vào tậpB nhiều nhất20số1để tập A có tổng phần tử bằng50:
(164)– NếujAkj 3thì tồn tập tập B tập Ak cho tổng phần tử củaB không nhỏ hơn35:Khi ta bổ sung vào tậpB nhiều 15số1để tập củaAcó tổng phần tử bằng50:
– NếujAkj D2thì tậpAnAk gồm nhất33số có tổng bằng45:Khi tậpAnAk tồn nhất21số1:TậpAk có tổng phần tử bằng55nên tồn tập tập conB tậpAk cho tổng phần tử B không nhỏ hơn28:
Nếu tậpAnAkcó nhất22số1:Khi ta bổ sung vào tập B nhiều 22số1để tập A có tổng phần tử bằng50:
Nếu tậpAnAk có đúng21số1thì12số cịn lại có giá trị bằng2:Khi ta bổ sung vào tậpB thêm1số2và nhiều nhất20số1để tập củaAcó tổng phần tử bằng50:
Nếuak 10thìai 10;8i k Ta cần chứng minh chọn số số từ tậpT D fa1; a2; : : : ; akgcó tổng bằng10:Đây trường hợp riêng tốn lớn với trường hợpN D10:Nhận thấy ta cần chứng minh cho trường hợp kD8:
Giả sử phản chứng không tồn tập tậpT có tổng phần tử 10:Để ngắn gọn, ta gọi giả thiết (Q-N)
Theo bổ đề trên, tồn tậpB T cho tổng phần tử củaB chia hết cho5: Suy tổng phần tử củaB bằng5:
– NếujBj D5thìa1 D a2 D a3 D a4 D a5 D1:Để giả thiết (Q-N) xảy ta phải cóa6; a7; a8 4, từ
a1Ca2C Ca8 17; điều vơ lý
– NếujBj D4, đóB có chứa3số1và1số2thìa1 Da2 Da3D1 Để giả thiết (Q-N) xảy ta phải cóa5; a6; a7; a8 Nhưng ta lại có
a5Ca6Ca7Ca8D15)a5D3; a6 Da7 Da8 D4: Khi đóa6Ca7Ca1Ca2D10, trái với giả thiết (Q-N)
– NếujBj D3thìBsẽ chứa nhất1số1vàT nB gồm5số có tổng bằng15: Nếu tồn tập conC củaT nB có tổng phần tử chia hết cho5thìC hoặcC [B có tổng phần tử bằng10;trái với giả thiết (Q-N)
Nếu không tồn tập củaT nB có tổng phần tử chia hết cho5thì tất phần tử củaT nB có số dư chia cho Ta thấy có1trong 3trường hợp sau đây:
2
6
T nB D f1; 1; 1; 6; 6g T nB D f2; 2; 2; 2; 7g T nB D f3; 3; 3; 3; 3g
(165)Tạp chí Epsilon, Số 05, 10/2015
– NếujBj 2thìT nB gồm6số có tổng bằng15:Chọn5số trongT nB áp dụng bổ đề ta thấy tồn tập conC củaT nBcó tổng phần tử chia hết cho5:Khi đóC hoặcC [Bsẽ có tổng phần tử bằng10;trái với giả thiết (Q-N) Vậy giả thiết (Q-N) không xảy
Bài tốn giải hồn tồn
Cuối cùng, tác giả xin trân cảm ơn bạn bè đồng nghiêp hội toán Internet: BÀI TOÁN HAY - LỜI GIẢI ĐẸP - SAY MÊ TOÁN HỌCvề ý kiến sâu sắc giá trị
Tài liệu tham khảo
[1] J Bang-Jensen, G Gutin, and A Yeo,When the greedy algorithm fails, Discrete Optimiza-tion,1 (2004), 121-127
[2] Cormen, Leiserson, and Rivest,Introduction to Algorithms,1990
[3] P Erdos, G Abraham, and Z Abraham,Theorem in additive number Theory,Bull Research Council, Israel, 10F; 41-43; 1961 20
[4] G Gutin, A Yeo, and A Zverovich,Traveling salesman should not be greedy: domination analysis of greedy-type heuristics for the TSP, Discrete Applied Mathematics,117 (2002), 81-86
[5] L Lovász, J Pelikán, and K Vesztergombi,Discrete matematics Elementary and beyond,
(166)(167)TỐI ƯU TỔ HỢP I: CÁC BÀI TOÁN TỐI ƯU VỀ CÁC HỆ TẬP HỢP
Gil Kalai - Hebrew University of Jerusalem and Yale University
LỜI GIỚI THIỆU
Bài viết giới thiệu toán tối ưu liên quan đến hệ tập hợp Đây giảng Gil Kalai seminar"Các khái niệm bản"tại ĐH TH Hungari David Kazhdan xng.1
Paul Erdăos
1 Ba bi toỏn mở đầu
Chúng ta bắt đầu ba toán giống nhau, từ dễ đến khó
Bài tốn Cho N D f1; 2; 3; : : : ; ng Hỏi kích thước lớn họF các tập củaN
thoả mãn điều kiện hai tập hợp thuộcF có phần giao khác rỗng? (Một họ được gọi họ giao nhau).
Trả lời:Kích thước lớn là2n Chúng ta đạt điều chọn tất tập chứa phần tử ‘1’ Chúng ta đạt số lớn hơn, từ cặp gồm tập hợp phần bù nó, ta chọn tập hợp vào họ tập hợp
1
(168)Bài tốn Tìm kích thước lớn họF các tập củaN sao cho hai tập hợp bất kỳ thuộcF có hợp khácN?
Câu trả lời hiển nhiên toán chẳng qua toán Chỉ cần chuyển qua phần bù Và ta có đáp số giống toán I Ta phát biểu toán khác bên
Bài tốn mới.Tìm kích thước lớn họF tập N cho với hai tập hợp bất kỳS; RthuộcF, ta có giao chúng khác rỗng hợp chúng khácN
Một ví dụ họ tập hợp tất tập hợp chứa phần tử1nhưng khơng chứa phần tử2 Họ có2n 2tập hp Phi mt vi nm sau Erdăos xut tốn này, Kleitman chứng minh khơng có họ lớn với tính chất
Bi toỏn (Gi thuyt Erdăos-Sos) ChoF l h cỏc đồ thị vớiN là tập đỉnh Giả sửa rằng hai đồ thị họ có chung tam giác HỏiF có kích thước lớn bao nhiêu?
Tổng số đồ thị với n đỉnh là2.n2/ (Chú ý: ta đếm đồ thị mà không tính đến đẳng cấu chúng.)
Một ví dụ đơn giản họ tập hợp với tính chất cho tất tập hợp chứa tam giác cố định Ví dụ tất đồ thị chứa cạnh
f1; 2g;f1; 3g;f2; 3g:
Họ chứa
8 tất đồ thị Tồn hay không họ lớn đồ thị với tính chất cho?
Erdăos v Sos a gi thuyt l cõu trả lời khơng – khơng thể tìm họ lớn Giả thuyết vấn đề mở
(169)Tạp chí Epsilon, Số 05, 10/2015
2 Hai định lý họ tập với điều kin giao cho trc
nh lý Erdăos-Ko-Rado:Mt h giao cáck-tập củaN, đó2k nchứa tối đa n
k !
tập hợp
Định lý Fisher - deBruijn-Erdăos:H cỏc caN cho hai tập hợp khác họ có phần tử chung có nhiều nhấtnphần tử
Erdăos v deBruijn kt lun rngn im khụng thng hng mặt phẳng xác định n đường thẳng Hãy thử suy điều từ định lý trên!
Tất cáck-tập củaN chứa phần tử1cho ví dụ dấu cho định lý Erdos-Ko-Rado Với định lý Erdăos deBruijn ta ly h
ff1g;f1; 2g;f1; 3g; : : : ;f1; ngg
hoặc thay tập hợp
f2; 3; : : : ; ng; lấy mặt phẳng xạ ảnh hữu hạn
Mặt phẳng Fano điểm xạ ảnh hữu hạn bậc 2
3 Phép chứng minh định lý deBruijn Erdăos bng i s
tuyn tớnh.
Phộp chng minh nh lý Fisher - de Bruijn-Erdăos cú th trỡnh bày sau: Giả sử cómtập hợpA1; A2; : : : ; Amtrong họ Xét ma trận liên kết họ: Phần tử (i,j) ma trận ‘1’ nếui 2Aj Sự kiện cốt yếu cộtc1; c2; : : : ; cmcủa ma trận liên kết độc lập tuyến tính, từ suy ramn
(170)hs; si DX X˛i˛j˝ci; cj˛
Ta lưu ý nếui vàj khác thì˝ci; cj˛
D1vàhci; cii D jAij: Như
hs; si D m X
iD1
˛2i.jAij 1/C m X
iD1 ˛i
!2 :
Như tích bằng0khi tất hệ số˛i bằng0: Chứng minh ví dụ "các lý luận chiều tổ hợp"
4 Định lý Sperner
Định lý Sperner (1927) khẳng định kích thước lớn họF tập N đối xích quan hệ bao hàm hệ số nhị thức n
n=2 !
Lubell tìm phép chứng minh đơn giản đẹp cho định lý Sperner:
GọiF đối xích giả sử cósk tập hợpkphần tử Ta tính cặp.; S / D 1/; 2/; : : : ; n//là hoán vị củaf1; 2; : : : ; ngvàS tập hợp thuộc họ có khởi đầu là, cụ thể làS D f.1/; 2/; : : : ; k/gvớiknào Với hốn vị, ta tìm nhiều khởi đầuS họF (do điều kiện đối xích)
NếuS tập hợp k phần tử, ta tìm đúngkŠ.n k/Šhốn vị vớiS khởi đầu Kết hợp hai kiện này, ta có
n X
kD1
skkŠ.n k/ŠnŠhay nói cách khác n X
kD0 sk
n k
Bất đẳng thức (được gọi bất đẳng thức LYM) suy kết cần chứng minh
(171)Tạp chí Epsilon, Số 05, 10/2015
nh lý Erdăos-Ko-Rado cú mt phộp chng minh tương tự với chứng minh Ý tưởng tính cách cặp.; S /trong đóSlà tập hợp họ,là hốn vị vịng quanh 1/; 2/; : : : ; n// vàS "khoảng" liên tục
Một mặt ta có n 1/Š hốn vị vịng quanh dễ dàng thấy với hoán vị vậy, ta chọn nhiều k "khoảng" đôi giao Mặt khác, với tập hợp S có kŠ.n k/Š hốn vị vịng quanh mà S khoảng liên tục Như jFjkŠ.n k/Š.n 1/Škvà điều cho định lý Erdăos Ko Rado
Quay tr li mt chỳt v câu "dễ dàng nhận thấy" Phần sử dụng điều kiện2kn Một cách lý luận cho phần sau: xét khoảngJ mà phần tử tận bên trái nằm bên trái ý có k khoảng giao vớiJ mà phần tử tận bên trái nằm bên phảiz Một cách khác xét khoảngJ bào có độ dàikvà ý rằng2k 2khoảng có giao với khoảng chia thànhk 1cặp mà cặp chứa hai khoảng không giao
5 Định lý Turan toán Turan
Trường hợp đặc biệt định lý Turan cho đồ thị không chứa tam giác chứng minh Mantel vào năm 1907
Số lớn cạnh (ký hiệu làt2.n/) đồ thị n đỉnh không tam giác đạt đồ thị hai phe đầy đủ n đỉnh với kích thước hai phe gần tốt (cụ thể làhn
2 i
và n
C1
)
Định lý Turan dạng tổng quát chứng minh vào năm 40 kỷ trước Số lớn cạnh (ký hiệu làtr.n/) đồ thị n đỉnh không chứa đồ thị đầy đủr C1đỉnh đạt đồ thịr phe đầy đủ vớinđỉnh, kích thước phần gần tốt
Paul Turan
(172)vớiv,B– đỉnh lại Bây ý đỉnh thuộcAlập thành tập hợp độc lập (tức khơng có cạnh nối đỉnh củaA) Với đỉnh thuộcB ta xoá tất cạnh chứa đỉnh thay vào đó, nối đỉnh với tất đỉnh thuộcA Để ý đồ thị mới, bậc đỉnh không nhỏ bậc đồ thị ban đầu Và, nữa, đồ thị đồ thị hai phe (trong đóAlà phe) Cuối ta cần chứng minh với đồ thị hai phe số cạnh lớn hai phần có số đỉnh gần tốt
Sau chứng minh khác Xoá đỉnh đồ thịGvớinC1đỉnh không chưáKr Số cạnh đồ thị cịn lại khơng vượt qtr.n/ Thực điều tất đỉnh ý một cạnh tínhn 1lần Ta thu số cạnh trongG (và nghĩa tr.nC1/) không vượt phần nguyên tr.n/.nC1/
n Đánh giá cho kết xác toán
Chúng ta kết thúc chuyến tham quan thú vị toán mà Turan đưa vào năm 1940 Chúng ta muốn tìm số phần tử lớn tập hợp ba lập từ1; 2; : : : ; nkhông chứa "tứ diện", tức khơng chứa bốn ban có dạngfa; b; cg;fa; b; dg;fa; c; dg;fb; c; dg
Nếu độc giả chưa biết đáp số, thử đưa dự đoán Turan đưa giả thuyết giả thuyết cịn vấn đề mở
Một số ghi thêm dịch giả:
Gil Kalailà Giáo sư Toán Đại học Hebrew Jerusalem, Israel giáo sư thỉnh giảng Tốn Khoa học máy tính Đại học Yale, Mỹ Ông sinh năm 1955, bảo vệ luận án tiến sĩ năm 1983 hướng dẫn khoa học Micha Perles Ông giải thưởng Polya năm 1992 giải thưởng Erdos năm 1993 Ông tìm phương án thuật tốn simplex quy hoạch tuyến tính với thời gian mũ để chứng minh tính chất đơn điệu đồ thị có chuyển pha chặt, giải giả thuyết Borsuk số phần cần thiết để phân hoạch hình lồi thành tập có đường kính nhỏ
Từ năm 2008, ơng lập blog Combinatorics and more địa chỉgilkalai.wordpress comđể trình bày thảo luận vấn đề mà ông cộng quan tâm Blog có chất lượng chuyên môn cao, nhiều đề tài, báo khởi đầu từ thảo luận, gợi ý từ blog
Đối xích:Trong tập thứ tự (partial order set), đối xích họ phần tử đôi không so sánh với
(173)BÀI TOÁN HAY - LỜI GIẢI ĐẸP
Ban Biêp tập Epsilon
LỜI GIỚI THIỆU
Chuyên mục lấy cảm hứng từ viết thầy Nguyễn Duy Liên Epsilon số3về toán số6trong kỳ thi IMO2001với5cách giải khác Mục để dành viết toán hay, lời giải đẹp câu chuyện thú vị xung quan tốn lời giải
Tên chuyên mục mượn từ tên nhóm người yêu toán Facebook anh Nguyễn Văn Lợi sáng lập “Bài toán hay – Lời giải đẹp – Đam mê toán học” Chuyên mục ghi nhận đề cử bạn đọc chọn đăng kỳ1; toán
Kỳ chuyên mục giới thiệu với bạn đọc toán số6trong kỳ thi IMO 1983với lời giải tuyệt vời Bernhard Leeb
Năm1983khi tơi thi tốn quốc tế Paris, tơi quen hai người bạn Andrei Ratiu (người Romania) Bernhard Leeb (người Đức) Chúng tơi nói với tiếng Pháp tiếng tay Chúng sở thích đánh bóng bàn thường rủ chơi bóng phịng thể thao Lycee Luis Le Grande Chúng tơi có cổ động viên nhiệt tình bạn Nadia người Algeria, thi IMO chơi thân với Bernhard Chính bạn Nadia trả lời vấn Đài truyền hình Pháp “Ấn tượng em đoàn CHLB Đức đoàn Việt Nam” Đức năm có3bạn 42=42;trong Bernhard Bạn đạt giải thưởng đặc biệt với lời giải2dòng cho toán bất đẳng thức :
Choa; b; c là độ dài ba cạnh tam giác Chứng minh rằng
a2b.a b/Cb2c.b c/Cc2a.c a/>0: 1/ Thầy Lê Hải Châu kể lại phát short list cho trưởng đồn khơng giải (và đồn Việt Nam không làm được) Cho nên BGK thấy có lời giải chỉ2dịng q ấn tượng họp để trao giải đặc biệt cho Bernhard
Tìm hiểu thêm trước Bernhard đã2lần thi IMO năm1984còn thi thêm lần Các năm1981; 1982; 1984Bernhard huy chương bạc, dù điểm cao (thấp 35/:Nếu Bernhard đạt4huy chương vàng, thành tích đáng nể
Bernhard giỏi ngoại ngữ Cậu biết tiếng Pháp, tiếng Anh Sau học Moscow trao đổi thư từ (và ảnh cô bạn gái người Nga)
Hiện Bernhard Leeb giáo sư đầu ngành đại học Munchen, có nhiều cơng trình tốn học giá trị
(174)Lời giải (Của BGK) Vìa; b; c độ dài3cạnh tam giác nên đặt x D bCc a
2 ; y D
cCa b ; z D
aCb c ;
thìx; y; zlà số dương vàaD yCz; b D zCx; c D xCy:Thay vào.1/và thực biến đổi, ta có bất đẳng thức cho tương đương với
xy3Cyz3Czx3 >xyz.xCyCz/: 2/ Áp dụng bất đẳng thức Cauchy Schwarz ta có
.xy3Cyz3Czx3/.zCxCy/>xyz.y CzCx/2; với dấu xảy
xy3 z D
yz3 x D
zx3 y :
Tức dấu xảy khix Dy Dz:Vậy dấu xảy bất đẳng thức ban đầu tam giác cho tam giác đều
Trong lời giải này, mấu chốt sử dụng phép thếaDyCz; b DzCx; c DxCy (được gọi phép Ravi Khi thi vào năm1983;chúng chưa biết đến phép này) để đưa bất đẳng thức.1/chỉ vớia; b; c ba cạnh tam giác bất đẳng thức.2/đúng với số thực dươngx; y; z;tức giải khó khăn điều kiện ràng buộc
Lời giải (Của Bernhard Leeb) Không tính tổng qt ta giả sửaDmaxfa; b; cg: Khi ta có
a2b.a b/Cb2c.b c/Cc2a.c a/Da.bCc a/.b c/2Cb.aCb c/.a b/.a c/>0: Từ dẫn đến kết luận tốn
Lời giải tay Bernhard Leeb viết cho bưu thiếp Tiếc qua nhiều năm với nhiều lần di chuyển, tơi khơng cịn giữ bưu thiếp q mà nhớ lời giải kỷ niệm người bạn ký ức
Bài tốn thật toán hay lời giải Leeb q tuyệt vời Tơi nhiều lần chia sẻ với bạn bè toán lời giải nhiều người đồng tình với nhận định tiếng trầm trồ Trên diễn đàn Bài toán hay – Lời giải đẹp – Đam mê toán học, giáo sư Phùng Hồ Hải (huy chương đồng tốn quốc tế năm1986/đã có chia sẻ toán này, vấn đề rộng mối liên hệ toán sơ cấp nghiên cứu tốn học Chúng tơi xin trích đăng ý kiến giáo sư Hải
(175)Tạp chí Epsilon, Số 05, 10/2015
giác (dữ kiện), kỹ thuật thứ tự (có thể hiểu xét trường hợp để đơn giản hóa bài tốn), hệ số cho bình phương (yếu tố quan trọng để đảm bảo dấu bằng) Ngoài ra là tài Leeb, mà thầy giáo chịu không dạy được.
Bất đẳng thức3số đẹp không tốn hình học phẳng Tơi nhớ năm1995tới khoa toán trường York, thấy họ treo bảng tin khoa thách đố toán vậy, đề thi IMO
1995nếu không nhầm Các giáo sư hì hục giải Tuy nhiên việc lạm dụng khơng tốt. Ngay IMO1995cũng thua xa IMO1983;khơng nói đến nhiều ngày Tiêu chí đầu tiên cho toán sơ cấp dành cho học sinh giỏi phải đẹp, lời giải nó cũng phải sơ cấp đơn giản (nếu đề đẹp lời giải khắc phải hay có ý mới) Khi sáng tạo đề cần nghĩ làm việc cho em học sinh không phải cho - để khơng tạo “bài tốn độc hại” Tình giống nơng dân trồng dưa, suất mà phun đủ thứ thuốc tăng trọng, thuốc trừ sâu Đối với bất đẳng thức3số tôi cho sau32năm giới có nhiều (riêng Việt Nam chí ngộ độc) Có rất nhiều tốn đẹp đó, chắn cịn có tốn đẹp (nhưng ít) Tơi thiết nghĩ, dạy cho học sinh, nghĩ cho em, nên chắt lọc gì tinh túy để truyền đạt (trong có4ý tưởng thể IMO1986/:Nhưng không nên sa đà, sa đà làm lãng phí tuổi trẻ em.
(176)(177)ĐƠI ĐIỀU VỀ HÌNH HỌC PHI EUCLID
Trần Nam Dũng - Trích từ “Các câu chuyện tốn học", NXB Giáo dục
Chuyện kể rằng, vào năm1823;Farkas Bolyai.1975 1858/đã viết thư cho người trai Janos Bolyai.1802 1860/người Hungary : “Con đừng đi vào đường mà bố đi, đừng nhảy vào “hang không đáy” nuốt hết trí tuệ, tinh lực tâm huyết bố”
Đây lời khuyên từ đáy lòng, từ trách nhiệm người bố suốt đời nghiên cứu Định đề5của Euclid mà không thành công Khi biết thích nghiên cứu “lý thuyết đường song song”, F Bolyai sợ hãi viết cho (trong thư khác) sau: “Con chiến thắng lí thuyết các đường song song đường Bố đến cuối đường lạc vào đêm đen dày đặc, tia sáng nến khơng có chơn vùi bao niềm hạnh phúc đời Khi lao vào học thuyết cô quạnh về các đường song song, chẳng cịn Con lẩn tránh lẩn tránh những dục vọng thấp hèn, làm hao mịn sức lực con, cướp an nhàn, quấy đảo yên tĩnh giết chết niềm vui sống Bóng tối mịt mùng nuốt chửng chịi tháp khổng lồ chẳng có lóe sáng Trái Đất tối tăm Chẳng người đạt tới thực hồn mĩ chính trong hình học Chúa trời cứu với khỏi ham mê ôm ấp ”
Nhưng F Bolyai khơng ngờ câu nói ơng trước làm J Bolyai bị thu hút vào vấn đề (câu nói có nội dung sau: “Ai chứng minh được tiên đề đường thẳng song song, người sáng ngời viên kim cương to Trái Đất”) Và chàng J Bolyai trẻ tuổi khơng lời cảnh báo bố mà lùi bước Tránh thất bại người trước, J Bolyai theo đường riêng Ơng khơng tìm cách chứng minh Định đề5của Euclid, mà xét tiên đề độc lập Và phủ định Định đề5của Euclid, J Bolyai xây dựng hệ thống hình học (mà sau cịn gọi Hình học phi Euclid) Các kết hình học ông phong phú chứng minh ông hoàn thiện
J Bolyai nhà toán học thiên tài, bị đố kỵ, chê bai bị điều đơm đặt ông Cuộc sống J Bolyai bị bọn quý tộc chèn ép, bao vây tinh thần lẫn vật chất Người bố nhà toán học đầy tâm huyết thương con, từ học sai lầm rút đời nghiên cứu tốn học mình, F Bolyai vơ tình trở thành vật cản đường tìm tịi, sáng tạo
(178)ơng khơng thể ngợi khen cơng trình đó, tức tự khen Ơng nói tư tưởng J Bolyai phụ lục tư tưởng ông nhiều năm trước Sau đó, Gauss viết thư cho Goling với ý cho nhà toán học trẻ tuổi J Bolyai thiên tài
Phải nói đánh giá nhà tốn học lỗi lạc Gauss hồn toàn chân thực Thật vậy, từ năm 1824, thư gửi cho người bạn Tolinos, Gauss viết: “Tổng ba góc tam giác phẳng phải nhỏ hơn180ı;giả định dẫn đến những đặc thù khác hoàn toàn với hình học Tơi phát triển thu được kết hoàn toàn khiến ta hài lòng” Và thư tiếng mà Gauss gửi Frants Adonf Taurinus chứng tỏ rằng, Gauss nắm ý niệm quan trọng Hình học phi Euclid Nhưng đoạn rời rạc, phát kiến sâu sắc.Tuy nhiên, lúc Gauss không công bố kết nghiên cứu
Thư trả lời Gauss làm cho J Bolyai có hiểu lầm lớn J Bolyai nghĩ Gauss dùng uy danh để cướp quyền phát minh hệ thống hình học ơng Vì thế, J Bolyai đau lòng vầ thề vứt bỏ nghiên cứu toán học Tháng10 1848;J Bolyai bố gửi cho luận văn: “Nghiên cứu lí thuyết đường song song” N I Lobachevski, xuất tiếng Đức năm1840:J Bolyai ngạc nhiên, thấy N I Lobachevski đến kết giống J Bolyai khâm phục tài N I Lobachevski
Cùng thời với J Bolyai, Cadan (Thủ đô nước cộng hòa tự trị Tacta thuộc Liên Bang Nga), xuất ngơi sáng, nhà toán học thiên tài Nicolai Ivanovich Lobachevski.1792 1856/:N I Lobachevski giáo sư xuất sắc, Hiệu trưởng cùa Trường Đại học Tổng hợp Cadan Ơng tìm cách chứng minh rằng, từ định đề tiên đề khác Hình học Euclid cổ điển, khơng thể suy Định đề5:
Để chứng minh điều đó, ơng giữ nguyên định đề tiên đề khác Hình học Euclid cổ điển thay Định đề 5bằng tiên đề phủ định tiên đề Euclid, phủ định Định đề5:Ngày nay, tiên đề nà gọi tiên đề Lobachevski Tiên đề có nội dung sau: “Trong mặt phẳng, qua một điểm không nằm đường thẳng cho trước, có hai đường thẳng khơng cắt đường thẳng cho” Rồi từ đó, Lobachevski xây dựng nên hình học khơng có mâu thuẫn Những kết hình học “trái mắt”, trái với trực quan hàng ngày chúng ta, trái với hình học Euclid quen thuộc
(179)Tạp chí Epsilon, Số 05, 10/2015
Trong thời đại Lobachevski, không hiểu tư tưởng ông, nhiều người chế nhạo ông Nhưng Lobachevski dũng cảm tin tưởng phát triển hình học Ơng kiên trì nghiên cứu cơng bố cơng trình nghiên cứu ngày chi tiết hơn, đầy đủ Một năm trước qua đời, Lobachevski bị mù Khi ơng cịn đọc cho học trị chép cơng trình sáng tạo mang tên “Hình học phẳng”, ông rõ Hình học Euclid trường hợp giới hạn Hình học phi Euclid ơng Lobachevski gửi cơng trình cuối cho Trường Đại học Tổng hợp Cadan – nơi đời sáng tạo ơng trơi qua
Ngày24 1856;Lobachevski qua đời, vài chục năm sau người ta cơng nhận tồn tư tưởng ơng
Cơng trình nghiên cứu Lobachevski J Bolyai Hình học phi Euclid thành tựu vĩ đại khoa học, mở kỉ ngun Tốn học, Vật lí nhiều ngành khoa học khác có liên quan
Vào năm1882;nhà toán học H J Poincare xây dựng mơ hình (gọi mơ hình Poincare) Hình học Lobachevski phẳng, sử dụng “vật liệu” lấy từ Hình học Euclid phẳng Trong mặt phẳng Euclid, lấy đường thẳngx nằm ngang, chia mặt phẳng thành2 miền, mà ta gọi “nửa trên” “nửa dưới” Ta có quy ước sau khái niệm Hình học Lobachevski phẳng: “Điểm” điểm Euclid thông thường thuộc “nửa trên” không thuộcx: “Đường tâm thuộcx;hoặc tia thông thường thuộc nửa trên, có gốc thuộcx
và vng góc vớix”
Tiếp tục, mơ hình này, xác định rõ ý nghĩa khái niệm khác, mà cụ thể tương quan sau : “Thuộc”, “ở giữa”, “bằng nhau” (cịn gọi “tồn đẳng”), “thuộc” “ở giữa” hiểu thơng thường
Người ta kiểm nghiệm tất tiên đề Hình học Lobachevski mơ hình nêu trên, thấy mơ hình thỏa mãn tất tiên đề
Thêm vào điều trên, ta có định lí sau Hình học Lobachevski : “Tổng ba góc trong tam giác nhỏ hai góc vng”
Việc xây dựng thành cơng mơ hình Hình học Lobachevski chứng minh : a/ Hình học Lobachevski phi mâu thuẫn.
b/ Từ tiên đề khác Hình học Euclid suy tiên đề Euclid.
Hình học Lobachevski khơng phải Hình học phi Euclid Hình học Riemann theo nghĩa hẹp Georg Friedrich Bernhard Riemann.1826 1866/người Đức Hình học phi Euclid Để có hệ tiên đề Hình học Riemann nghĩa hẹp, phải thay đổi hệ tiên đề Hình học Euclid nhiều thay đổi mà Lobachevski thực
(180)(181)GIỚI THIỆU MỘT SỐ ĐỀ THI CHỌN ĐỘI TUYỂN MƠN TỐN
NĂM 2015 - 2016
Ban Biên tập Epsilon
1 Một số đề thi
1.1 Đề thi chọn đội tuyển chuyên KHTN Hà Nội Ngày thi thứ 1
Bài 1. Cho dãy số.an/thỏa mãn
a0D1; anC1 D
q a2
nC1
Ca3n
:
Chứng minh rằng.an/hội tụ tìm giới hạn củaan Bài 2. Tìm tất sốnnguyên dương cho
3nC4nC5nj60n:
Bài 3. Cho tam giácABC cóE; F thuộc cạnhCA; AB choEF kBC Tiếp tuyến tạiE; F đường tròn.AEF /cắtBC tạiM; N Giả sửBE cắtF N tạiK vàCF cắt EM tạiL:
1 Chứng minh rằng∠KAB D∠LAC
2 Giả sửBE cắtCF tạiX vàEN cắtFM tạiY Chứng minh rằngX Y qua điểm cố định khiE; F thay đổi
Bài 4. Chox; y; zlà số thực dương có tổng bằng1:Tìm GTLN
P D s
x2y 4xC5y C
s y2z 4yC5z C
s z2x 4zC5x:
Ngày thi thứ 2
Bài 5. Tìm tất hàm sốf WR!Rthỏa mãn điều kiện:
(182)Bài 6. Cho dãy số.an/thỏa mãn (
a0 Da1 D5;
anC1 D7an an 1C44
; n1:
Chứng minh với mọinthìanlà tổng của2số phương
Bài 7. Cho tam giácABC đường tròn K/đi quaB; C các đoạn AC; AB E; F GọiM; N điểm đối xứng vớiB; C quaE; F theo thứ tự Tiếp tuyến tạiAcủa đường trịn.AMN /cắtMN; BC tạiP; Q Chứng minh rằngAlà trung điểmPQ: Bài 8. Cho bảng ô vuôngnnvớin N số nguyênk n:Điền vào ô bảng
nncác số thực thuộc đoạnŒ 1I1sao cho tổng số bảng conkkđều Tìm giá trị lớn tổng tất số bảng
1.2 Đề thi chọn đội tuyển PTNK Ngày thi thứ 1
Bài 1. Cho tập hợp
AD fn2 Nj1n2015; n; 2016/D1g:
Hỏi có số nguyêna A cho tồn số nguyênb mà aC2016b số phương?
Bài 2. Choa; b; c; d số thực thỏa mãn điều kiện
a2 1; a2Cb2 5; a2Cb2Cc2 14; a2Cb2Cc2Cd2 30:
1 Chứng minh rằngaCbCcCd 10: Chứng minh rằngad Cbc 10:
Bài 3. Tìm tất hàm sốf WR!Rthỏa mãn điều kiện
f x 2f y//D5f x/ 4x 2f y/
với mọix; y 2R:
Bài 4. Cho đường trònkvà điểmB; C thuộc đường trịn, khơng phải đường kính;I trung điểmBC ĐiểmAdi động cung lớnBC củak Gọii1là đường tròn quaI tiếp xúc vớiAB tạiB;i2 đường tròn quaI tiếp xúc vớiAC tạiC Các đường tròni1; i2 cắt tạiD(khácI)
1 Chứng minh đường trịn ngoại tiếp tam giácAIDln qua điểm cố định GọiK trung điểmAD,E tâm đường tròn quaK tiếp xúc vớiAB tạiA,F tâm
(183)Tạp chí Epsilon, Số 05, 10/2015
Ngày thi thứ 2
Bài 5. Dãy số.xn/được xác định cơng thứcxnD
ncos1n với mọin1:Tính giới hạn sau lim
n!C1
x1Cx3Cx5C Cx2n x2Cx4Cx6CsCx2n :
Bài 6. Tìm giá trị củab cho tồn tạiađể hệ phương trình sau có nghiệm.x; y/ (
.x 1/2C.yC1/2 Db y Dx2C.2aC1/xCa2
Bài 7. Cho n số nguyên dương, n X D f1; 2; 3; : : : ; ng Gọi A1; A2; : : : ; Am B1; B2; : : : ; Bm hai dãy tập khác rỗng củaX thỏa mãn điều kiện: Với mỗii; j f1; 2; 3; : : : ; ng,Ai \Bj D nếui Dj:
1 Chứng minh với hoán vị.x1; x2; : : : ; xn/củaX, có khơng q cặp tập hợp Ai; Bi/vớii D1; 2; 3; : : : ; nsao cho nếuxk Ai vàxl 2Bi thìk < l
2 Gọiai; bi số phần tử tập hợpAi; Bi vớii D 1; 2; 3; : : : ; m Chứng minh
m X
iD1 Cai
aiCbi 1:
Bài 8. Cho tam giácABC nhọn nội tiếp đường tròn tâmO:Đường tròn tâmI quaB; C cắt tiaBA; CAtạiE; F:
1 Giả sử tiaBF; CE cắt tạiDvàT tâm đường tròn.AEF / Chứng minh OT kID:
2 TrênBF; CE lấy điểmG; H choAG?CE; AH?BF:Các đường tròn ABF /; ACE/cắtBC tạiM; N (khácB; C) cắtEF tạiP; Q(khácE; F) GọiK giao điểm củaMP; NQ Chứng minh rằngDK vng góc vớiGH:
1.3 Đề thi chọn đội tuyển TP.HCM Ngày thi thứ 1
Bài 1. Chứng minh phương trình
3x3 24x2C60x 47D0
có3nghiệm thực và3nghiệm độ dài3cạnh tam giác có1góc lớn hơn120ı: Bài 2. Chox; y; zlà số thực thuộc đoạnŒ0I1và thỏa mãnxCyCz D2 Tìm giá trị lớn giá trị nhỏ
P D xyC1C
1 yzC1 C
(184)Bài 3. Cho tam giácABC vuông tạiAcó đường caoAH GọiK điểm đoạn AH Trên tiaKB lấy điểmM choCADCM, tiaKC lấy điểmN choBADBN Giả sửCM vàBN cắt tạiI Chứng minh rằngIM DI N:
Bài 4. Cho hàm sốf WN!Nthỏa mãn điều kiện
f 2/D5; f 2016/D2015vàf n/Df f n 1// vớin1:Tínhf 0/; f 1/vàf 2015/biết rằngf 2015/là số lẻ
Ngày thi thứ 2
Bài 5. Cho dãy số.an/xác định a1 D1; a2D
12; anC1 D
an2
an 1.5anC1/2:
Chứng minh rằngAD s
1 a2015 C
s
a2016 số nguyên
Bài 6. Cho tam giácABC nội tiếp đường tròn.O/với60ı < A < 90ı GọiB1 điểm đối xứng vớiB quaCAvàC1là điểm đối xứng vớiC quaAB GọiO1là điểm đối xứng với tâmO quaBC Chứng minh tâm đường tròn ngoại tiếp tam giácAB1C1nằm đường thẳng AO1
Bài 7. Tìm tất cặp số nguyên dương.a; b/sao cho a3Cbvàb3Ca lũy thừa của2với số mũ nguyên dương
Bài 8. Cho tập hợpAD f1; 2; 3; : : : ; 2015g Hỏi cần phải loại khỏiAít phần tử để phần tử cịn lại thỏa mãn tính chất khơng có phần tử tích hai phần tử khác?
Đề thi chọn đội tuyển Nghệ An
Bài 1. Cho số thực dươnga; b; c Chứng minh a
bCc C b cCa C
c aCb C
r a 2bC2c C
s b 2cC2a C
r c
2aC2b 3:
Bài 2. Tìm tất số tự nhiênasao cho tồn số nguyên dươngn > 1và thỏa mãnanC1 chia hết chon2
Bài 3. Cho tứ giácABCDnội tiếp đường trịn.O/
(185)Tạp chí Epsilon, Số 05, 10/2015
2 Các đường phân giác góc tứ giácABCD cắt tạo thành tứ giác MNPQ:GọiX; Y trung điểm củaMP; NQ Chứng minh rằngX; O; Y thẳng hàng
Bài 4. Viết số từ1đến2015lên bảng Ta chọn hai số bất kỳa; b bảng xóa chúng đi, sau viết thêm sốja bjlên Thực liên tiếp bảng lại số Gọi số làm
1 Hỏimcó thể bằng1được khơng? Tìm tập hợp giá trị có củam
1.4 Đề thi chọn đội tuyển ĐH Vinh Ngày thi thứ 1
Bài 1. Cho sốa > 0và dãy số.xn/xác định x1Dx2 Da; xnC1 DxnC
2pxn
n3 ; nD2; 3; : : : Chứng minh rằngxn< 2.aC3/vớinD1; 2; 3; : : :
Bài 2. Tìm tất hàmf W 0I C1/ !.0I C1/sao cho bất đẳng thức sau với số thực dươngx; y W
1 f xCy/f x/C2y; f f x//4x:
Bài 3. Cho tam giácABC điểmX nằm tam giác GọiH; K theo thứ tự hình chiếu củaX lên cạnhBC; BAvàA0; C0lần lượt điểm đối xứng củaX quaBC; BA:
1 GọiE DBC \XK; F DAB\XH Chứng minh điểmA0; B; C0; E; F thuộc đường tròn
2 GọiY giao điểm thứ hai đường trịn A0BC0/ với đường trịn đường kính BX: Chứng minh KX
K Y D HX H Y
Bài 4. Một lớp học cónhọc sinh tham gia trị chơi tơ màu bảng vngnnnhư sau: Trong phút đầu tiên,nhọc sinh tô màunô vuông không hàng cột Trong phút tiếp theo, học sinh tơ vng có cạnh chung với ô mà học sinh
đó tô phút liền trước
Một ô tô lần
Hỏi saunphút, bảng vng tơ kín hay khơng trường hợp sau đây: nD30?
(186)Ngày thi thứ 2
Bài 5. Cho số thực dươnga; b; c thỏa mãnaCbCc D a b C
b c C
c
a Tìm giá trị lớn biểu thức sau
P D aCbC1 a2Cb2C1 C
bCcC1 b2Cc2C1 C
cCaC1 c2Ca2C1:
Bài 6. Cho tứ giácABCD vừa nội tiếp vừa ngoại tiếp đường tròn GọiI tâm đường tròn nội tiếp tứ giác Đường thẳng quaI, song song vớiABcắtADvàBC tạiH vàK:Chứng minh độ dàiHKbằng phần tư chu vi tứ giácABCD:
Bài 7. Choplà số nguyên tố Chứng minh tồn số tự nhiênnsao chopncó chứa2015 chữ số liên tiếp
1.5 Đề thi chọn đội tuyển Đà Nẵng Ngày thi thứ 1
Bài 1. Cho dãy số.xn/xác định
x1 D1; xnC1 D3C xn
vớinD1; 2; 3; : : :Tìm số thực dương a cho dãy số.yn/xác định ynD a
n x1x2: : : xn có giới hạn hu hn v limynÔ0:
Bi 2. Cho tam giỏcABC nhọn khơng cân nội tiếp.O/có đường caoAH tâm đường trịn nội tiếp làI:GọiM điểm cung nhỏBC (O).D điểm đối xứng vớiAqua O:Đường thẳngMDcắt đường thẳngBC; AH theo thứ tự điểmP; Q
1 Chứng minh tam giácIPQvuông
2 Đường thẳngDI cắt.O/tại điểmE khácD:Hai đường thẳngAEvàBC cắt tạiF: Chứng minh nếuABCAC D2BC thìI trọng tâm tam giácAPF:
Bài 3. Tìm tất đa thứcP x/hệ số thực thỏa mãn điều kiện P x//3 3.P x//2 DP x3/ 3P x/ với mọix 2R:
Bài 4. Xác định tất số nguyên dươngn thỏa mãn tính chất tồn cách chia hình vng có độ dài cạnh lànthành năm hình chữ nhật cho độ dài cạnh năm hình chữ nhật số
(187)Tạp chí Epsilon, Số 05, 10/2015
Ngày thi thứ 2
Bài 5. Tìm tất hàm sốf WR!Rthỏa mãn
f x4Cf y/
DyCf4.x/ với mọix; y 2R:
Bài 6. Cho số hữu tỉa; b; cthỏa mãn điều kiện (
aCbCc 2Z
.2a 1/2C.2b 1/2C.2c 1/2D3 Chứng minh tồn số nguyênm; nthỏa mãn
.m; n/D1vàabc D m n3:
Bài 7. Cho tam giácABC nhọn nội tiếp đường tròn.O/:Đường tròn.J /tiếp xúc với O/tạiDđồng thời tiếp xúc với tia đối tiaBA; CAlần lượt tạiE vàF:
1 Chứng minh DB DC D
1CcosC 1CcosB
2 Giả sửAJ cắt.O/tạiT khácA: GọiP; Q điểm di động cung nhỏ AB; AC của.O/sao choPQsong song vớiBC:Các đường thẳngAP vàBC cắt tạiM: Gọi I; N trung điểm đoạn thẳng EF; IM:Chứng minh giao điểm đường thẳngN T vàIQluôn thuộc đường cố định
Bài 8. ChoP đa giác lồi2016cạnh Một cách chiaP thành tam giác đường chéo không cắt bên trongP gọi cách chia đẹpP
1 Chứng minh số đường chéo cần phải nối để chia đẹpP theo cách khác nhau
2 Một tam giác thu từ phép chia đẹpP nói gọi tam giác 3cạnh đường chéo củaP Hỏi có tất cách chia đẹpP mà có tam giác trong, cho biết hai cách chia khác có nhấu cặp tam giác không trùng nhau?
2 Lời giải số đề chọn lọc
2.1 Lời giải đề thi chuyên KHTN Hà Nội Ngày thi thứ 1
Bài 1. Cho dãy số.an/thỏa mãn
a0D1; anC1 D
q a2
nC1
Ca3n
:
(188)Lời giải. Ta thấy q
.a2 nC1/
3
Can3 > q
a6 nCa
3
nD janj
Can3
nên theo công thức cho, ta suy raan< 0;8n1: Xét hàm sốf x/D
7 q
.x2C1/3Cx3
vớix < 0, ta có
f0.x/D 7x
p
x2C1C3x
> 0;8x2 R:
Suy raf x/là hàm đồng biến Dễ dàng thấy rằnga1> a2nên quy nạp, ta có.an/là dãy tăng từ số hạng thứ Hơn nữa, dãy bị chặn nên có giới hạn Để tìm giới hạn đó, ta xét phương trình sau vớix 0:
3
q
.x2C1/3Cx3
Dx, q
.x2C1/3Cx3C 3x D0 ).x2C1/3Dx2
x2C
2
Đặty Dx2 0thì
.yC1/3Dy
y C7
2
,y3C3y2C3yC1Dy
y2C 14 yC
49 , 3y C 22
9 y 1D0,
6
y D y D
3
Do y nên ta chọn y D
3 có x D p
3 Vậy giới hạn dãy cho limanD p1
3
Nhận xétĐây giới hạn dạng bảnunC1 Df un/vớif x/là hàm đồng biến Đây tình bất ngờ tốn giới hạn dãy số đề thi KHTN ln mang tính thử thách định
Bài 2. Tìm tất sốnnguyên dương cho
3nC4nC5nj60n:
Lời giải. Do60nD22n3n5nvà3nC4nC5nlà ước của60nnên ta phải có 3nC4nC5nD2x 3y5z
trong đóx; y; z2 Nvà0x 2n; 0y; z n
(189)Tạp chí Epsilon, Số 05, 10/2015
1 Nếuy > 0thì3j4nC5nhay1C 1/n mod 3/ Suy ranlẻ ĐặtnD2kC1với k2 N:Khi
3nC4n 2/nC 1/n 1C2n/ mod 5/ Nếuz > 0thì5j3nC4n, dẫn đến 1C2n/0 mod 5/hay
22kC1 mod5/,2 1/k mod 5/
Tuy nhiên2 1/k mod 5/hoặc2 1/k mod 5/nên điều xảy Do đóz D0và ta có
3nC4nC5nD2x 3y: Vìnlẻ và2j.3C5/nên theo định lý LTE, ta có
v2.3nC5n/Dv2.3C5/Dv2.8/ D3:
Suy rav2.3nC4nC5n/Dminf3; 2nghayx Dminf3; 2ng:Ta lại có trường hợp: NếunD1thì3nC4nC5nD12và ta cóx D2; y D1thỏa mãn
NếunD3thì3nC4nC5nD216và ta cóx D3; y D3thỏa mãn
Nếun > 3thìx Dminf3; 2ng D Do đó3nC4nC5nD83y Tiếp theo, ta có v3.4nC5n/Dv3.n/Cv3.4C5/Dv3.n/Cv3.9/Dv3.n/C2:
Chú ý nếun > 3thìv3.n/C2 < nnên
y Dv3.3nC4nC5n/Dminfv3.n/C2; ng Dv3.n/C2: Suy
3nC4nC5nD83v3.n/C2,3nC4nC5nD72n: Dễ thấy phương trình vơ nghiệm vớin >
Do đó, trường hợp này, ta cónD1; nD3thỏa mãn Nếuy D0, ta có
3nC4nC5n D2x5z: Nếuz D0thì3nC4nC5nD2x 4n, vơ lý
Nếuz > 0thì5j3nC4n XétnD4kCr vớir f0; 1; 2; 3; 4g Theo định lý Fermat nhỏ
34kCrC44kCr 34/k3rC.44/k 4r 3r C4r mod5/ Thử trực tiếp, ta thấyr D2thỏa mãn Do đó,nD4kC2vớik 2N:
Ta có3nC5n 1C1 2.mod4/nên v2.3nC5n/D 1vàv2.3nC4nC5n/D minf1; 2ng D
Do đóx D1hay3nC4nC5nD25z Tiếp theo, ta có
(190)– NếunD2thì32C42C52D50D252 nênx D1; z D2, thỏa mãn – Nếun6thì dễ thấy rằngv5.2kC1/C2 < nnên
z Dv3.3nC4nC5n/Dminfv5.2kC1/C2; ng D v5.2kC1/C2:
Suy
3nC4nC5nD25v5.2kC1/C2
,3nC4nC5nD25n:
Dễ thấy phương trình vơ nghiệm vớin6 Do đó, trường hợp này, ta cónD2thỏa mãn
Vậy giá trịncần tìm D1; 2; 3:
Nhận xétĐịnh lý LTE sử dụng vơ hiệu tình Câu hỏi tương tự: Tìm sốnnguyên dương cho
2nC3nC5nj30n:
Bài 3. Cho tam giácABC cóE; F thuộc cạnhCA; AB choEF kBC Tiếp tuyến tạiE; F đường tròn.AEF /cắtBC tạiM; N Giả sửBE cắtF N tạiK vàCF cắt EM tạiL:
1 Chứng minh rằng∠KAB D∠LAC
2 Giả sửBE cắtCF tạiX vàEN cắtFM tạiY Chứng minh rằngX Y qua điểm cố định khiE; F thay đổi
Lời giải. 1) Giả sửBE; CF cắt đường tròn ngoại tiếp tam giácABC tạiS; T:Ta có ∠BF N D180ı ∠NFE ∠EFAD180ı ∠BAC ∠ABC
D∠ACB D∠ASB:
(191)Tạp chí Epsilon, Số 05, 10/2015
Mặt khác,∠FEB D∠EBC D∠S TF Từ đó, ta có tứ giácEF T S nội tiếp Suy ∠KAB D∠F SE D∠F TE D∠LAC:
2) Từ tính chất góc tiếp tuyến, ta dễ dàng có
∠BF N D∠ECM;∠CEM D∠FBN: Do đó, tam giácFBN vàCEM đồng dạng Chú ýF N DEM nên
BN CM D
BN ME
F N CM D
FB EC
FB EC D
F B2 EC2 D
AB2 AC2:
GọiP giao điểm củaX Y vàBC:Áp dụng định lý Menelaus cho tam giácFM C vớiX; Y; P thẳng hàng, ta có
PM P C
XC XF
Y F Y M D1:
Tương tự, áp dụng Menelaus cho tam giácENBvới điểmX; Y; P thẳng hàng, ta có PN
PB XB XE
YE Y N D1: Ta lại ý doEF kBC nên XC
XF D XB XE
Y F Y M D
YE Y N Từ ta thu PM
P C D PN
PB hay PM PN D
P C PB D
P C PM PB PN D
CM BN D
(192)không đổi nênP cố định Vậy đường thẳngX Y qua điểmP cố định Bài 4. Chox; y; zlà số thực dương có tổng bằng1:Tìm GTLN
P D s
x2y 4xC5y C
s y2z 4yC5z C
s z2x 4zC5x:
Lời giải. Theo bất đẳng thức Cauchy-Schwarz thì:
P2D X cyc
s x2y 4xC5y
!2
.xyCyz Czx/ X cyc
x 4xC5y
! :
Chú ý
X
cyc x 4xC5y D
X cyc 5y 4xC5y
D X cyc y 4xC5y
5
.xCyCz/2
5.x2Cy2Cz2/C4.xy Cyz Czx/ D
4
1
5.x CyCz/2 6.xyCyzCzx/ D
4
5
4Œ5 xyCyz Czx/
ĐặtaDxyCyz Czxthìa > 0vàa
3.x CyCz/
D
3, tức làa2
0I1 Khi X cyc x 4xC5y
3
5 4.5 a/ D
10 3a 4.5 a/: Do
P2 a.10 3a/
4.5 a/ Df a/:
Ta cóf0.a/D 3a
2 30aC50
4.5 a/2 > 0;8a2
0I1
nên hàm đồng biến
Suy raP f a/ f D Vậy giá trị lớn củaP
9, đạt khix Dy Dz D
Ngày thi thứ 2
Bài 5. Tìm tất hàm sốf WR!Rthỏa mãn điều kiện:
(193)Tạp chí Epsilon, Số 05, 10/2015
Lời giải. Gọi./là đẳng thức đề cho ThayxD1trong./;ta có
f 0/f y2/D0:
Nuf 0/Ô0thỡ vi miy 2R, ta phải cóf y2/D0 Suy
f x 1/0Dyf xy/ yf y/,yf xy/Dyf y/:
Thayy y2 > 0thìy2f xy2/ D y2f y2/ D 0nênf xy2/ D 0;8x vy Ô 0:iu ny cho thyf x/D0;8x õy l mt hàm số thỏa mãn đề
Nếuf 0/D0thì thayx D0vào./thì:
f 1/f y2/Dyf 0/ yf y/,f 1/f y2/D yf y/: Thayy D 1, ta cóf 1/f 1/Df 1/, dẫn đếnf 1/D0hoặcf 1/D1
Tuy nhiên, nếuf 1/D0thì dẫn đếnyf y/D0;8y nênf y/D0;8y ¤0, xét Ta xétf 1/D1, thayy D1vào./;ta đượcf x 1/Df x/ Do
f y2/D yf y/,f y2/Dyf y/: Từ kết này, ta thay vào (*) được:
.f x/ 1/yf y/Dyf xy/ yf y/ ,f x/f y/ f y/Df xy/ f y/ ,f x/f y/Df xy/;8x2 R;8y Ô0
ThayxDy vo ta cóŒf x/2 Df x2/, màf x2/Dxf x/nên Œf x/2 Dxf x/,f x/Dx:
Vậy hàm số cần tìm làf x/0;8x2 Rhoặcf x/Dx;8x 2R:
Bài 6. Cho dãy số.an/thỏa mãn (
a0 Da1D5;
anC1 D7an an 1C44
; n1 Chứng minh với nthìanlà tổng của2số phương
Lời giải. Ta tính số số hạng dãy cho:
a0D5D 2/2C 1/2 a1D5D12C22
a2D74D52C72 a3D409D142C192 a4D4869D372C502 Từ đây, ta dự đoán rằngan Du2nCv
2 nvới un/W
(
(194)và
.vn/W (
v1 D 1; v2 D2; vnC2 D3vnC1 vớin0:Ta chứng minh quy nạp
anDu2nCv n với mọin0: /Thật vậy,
VớinD0; nD1thì khẳng định./đúng Giả sử./đúng tớin, ta có
u2nC1Cvn2C1 D.3un un 1/2C.3un un 1/2 D7.u2nCv2n/ u2n 1Cv2n 1/C2.u2nCvn2/C C.u2n 1Cvn 12 / 6.unun 1Cvnvn 1/
Ta có
u2nCu2n 1 3unun Dun.3un un 2/Cu2n 1 3unun D unun 2Cu2n 1
D 3un un 2/un 2Cu2n 1 Du2n 1Cu2n 2 3un 1un Như thế, suy
u2nCu2n 1 3unun 1Du21Cu20 3u1u0 D11: Chứng minh tương tự, ta có
vn2Cv2n 1 3vnvn Dv12Cv02 3v1v0D11:
Do
2.u2nCv n/C.u
2 n 1Cv
2
n 1/ 6.unun 1Cvnvn 1/D44 hay
u2nC1Cv
nC1 D7.u nCv
2 n/ u
2 n 1Cv
2
n 1/C44D7an an 1C44DanC1:
Suy ra./đúng vớinC1và theo nguyên lý quy nạp thì./được chứng minh
Vậy ta cóan D u2nCvn2;tức với mọinthìanlà tổng của2số phương Đây điều phải chứng minh
Nhận xétĐoạn quy nạp phía sau khơng khó, chủ yếu đốn công thức dãy.un/; vn/ Ta ý cơng thức quan trọng dãy sai phân tuyến tính hay sử dụng: Cho dãy số.un/xác định theo công thứcunC2 DaunC1Cbunvớin1thì với mọin, ta ln có
(195)Tạp chí Epsilon, Số 05, 10/2015
Bài 7. Cho tam giácABC đường tròn K/đi quaB; C các đoạn AC; AB E; F GọiM; N điểm đối xứng vớiB; C quaE; F theo thứ tự Tiếp tuyến tạiAcủa đường trịn.AMN /cắtMN; BC tạiP; Q Chứng minh rằngAlà trung điểmPQ:
Lời giải. GọiB0; C0lần lượt điểm đối xứng vớiB; C quaA:Giả sửB0M; C0N cắt tạiD:Ta chứng minh rằngB0; C0; P thẳng hàng
Ta có AB
0
AC0 D
AB
AC , màB; E; C; F thuộc đường tròn nên ABAF DAC AE hay AB
AC D AE AF:
Chú ý rằngAE đường trung bình tam giácBB0M nênB0M D2AE Tương tựC0N D2AF Do AE
AF D B0M
C0N Kết hợp điều lại, ta
AB0
AC0 D
B0M
C0N
Ta lại có∠AB0M D∠BAC D∠AC0N nênAB0M AC0N Do AM AN D
B0M C0N
Mặt khác dễ thấy tứ giácAB0DC0là hình bình hành nên B0D
C0D D
AC0 AB0 D
AN AM:
VìAP tiếp tuyến của.AMN /nênAMP NAP PM
PN D SPAM SPAN D
AM AN
2 :
Suy
PM PN
B0D B0M
C0N C0D D
AM AN
2 AN
(196)nên theo định lý Menelaus đảo, ta cóB0; C0; P thẳng hàng
Hơn nữa,BCB0C0là hình hình hành nênBC kB0C0 Theo định lý Thales AP
AQ D AB0
AB D1nênAP DAQ: Ta có đpcm
Nhận xét.Bài tốn có liên quan đến tốn quen thuộc sau:
Cho tam giácABC cóB0; C0là điểm đối xứng vớiB; C quaAC; AB theo thứ tự Khi đó, Anằm đường nối tâm đường tròn Euler tam giácABC đường tròn ngoại tiếp tam giácAB0C0
Khi đối chiếu toán (dạng tổng quát), tính chất sau đúng: GọiH; K trung điểmAB; AC vàI tâm đường tròn ngoại tiếp tứ giácEFHKthìAI qua tâm đường trịn ngoại tiếp tam giácAMN
Bài 8. Cho bảng ô vuôngnnvớin N số nguyênk n:Điền vào ô bảng
nncác số thực thuộc đoạnŒ 1I1sao cho tổng số bảng conkkđều bằng0: Tìm giá trị lớn tổng tất số bảng
Lời giải. ĐặtnDakCr vớia2 ZCvà0r < k:Ta chứng minh2nhận xét sau:
Nhận xét 1. Nếu gọiT; S tổng số điền vào bảng vng có kích thước r r và.k r/.k r/nào
jTj t Dmin˚r2; k2 r2 vàjSj s Dmin˚.k r/2; k2 k r/2 :
Rõ ràng bảng vng kích thướcr r bị chứa bảng vng kích thước kknào mà tổng số bằng0:GọiT0là tổng số lại bảngkksau loại bảngr r chứa Ta có
T CT0D0) jTj DˇˇT0ˇˇ:
Số lượng ô bảngr r làr2nênjTj r2 số lượng cịn lại bảngkklà k2 r2nênˇˇT0ˇˇk
2
r2:Do đó:
jTj t Dmin˚r2; k2 r2 Hoàn toàn tương tự với bảng.k r/.k r/
Nhận xét 2. Nếu kích thước bảng tăng lênkđơn vị tổng số bảng tăng không quát Cs
(197)Tạp chí Epsilon, Số 05, 10/2015
Để tính tổng số điền vào phần tăng thêm, bao gồm phần ngang phần dọc (giao bảngkktại góc) Ta thấy phần nằm ngang chia thành số bảng vuôngkk không chồng lên dư lại hình chữ nhật kích thướck.k r/ở cuối Tương tự, phần nằm dọc chia thành số bảng vuôngkkkhông chồng lên dư lại hình chữ nhật kích thước.k r/k
Do đó, để tính tổng số phần thêm vào, trước hết, ta tính tổng số hình chữ nhật khơng chồng lên (theo giả thiết tổng bằng0) Phần hình vng kích thước k r/.k r/có màu đỏ giao của2hình vng cuối dãy ngang đầu dãy dọc, tính2lần nên cần phải trừ Cuối cùng, ta cộng số bảng vng góc có kích thướcrr cịn lại
Nói tóm lại, đặtT tổng số bảng vngkkở góc bên phải vàS tổng số phần chung bảng.k r/.k r/thì tổng tăng thêm là:T S
Khi đó, theo nhận xét thìT S t Cs Nhận xét chứng minh
Trở lại toán,
Ta thấy phần màu xanhr r ban đầu có tổng số khơng vượt qt nên tổng số bảng saualần tăng kích thước (từr r lênnnvớinDakCr) tổng khơng vượt q t Ca.t Cs/
Đến đây, ta cách xây dựng bảng thỏa mãn
Xét bảng ô vngkkở góc bên phải bảngnn, ta điền sau:
(198)Điền vào phầnB hình vng.k r/.k r/sao cho tổng số s (có thể điềnssố 1và phần cịn lại 0)
Điền vào phầnC; Dtùy ý cho tổng chúng bằngs t:Rõ ràng tồn cách điền giới hạn giá trị củat; s:
Đến đây, ta thực điền số vào bảng vng kề với cách tuần hồn theo chu kỳk:Nghĩa số điền ô.a; b/sẽ giống số điền ô
.aCk; b/; a; b Ck/; a k; b/; a; b k/:
Dễ thấy đó, tất hình vng kích thướckkđều có tổng bằng0và hình vng r r góc bên trái giống với hình vngr r góc bên phải, tức tổng bằngt: Do đó, cách điền cho ta bảng có tổng
t Ca.t Cs/:
Vậy giá trị lớn cần tìm làt Ca.t Cs/vớit; s; ađược xác định
2.2 Lời giải đề thi PTNK TP.HCM Ngày thi thứ 1
Bài 1. Cho tập hợpA D fn2Nj1n2015; n; 2016/D1g Hỏi có số nguyên
a2Asao cho tồn số nguyênbmàaC2016blà số phương?
Lời giải. Chon số nguyên dương lớn 1;ta quy ước gọi số nguyên dươnga gọi thặng dư phương theo modulo n nếu.a; n/ D 1và tồn số nguyên x cho ax2.modn/:Trong này, để đơn giản, ta quy ước xét thặng dư phương nhỏ n:
Đặts.n/là số số nhỏ hơnnvà thặng dư phương theo modulon Ta chứng minh hai bổ đề đây:
Bổ đề 1:Choplà số nguyên tố vàklà số nguyên dương Khi đó: Nếup D2thìs.2k/D2max.k 3;0/
2 Nếup > 2thìs.pk/D p
k pk Bổ đề 2:s.n/là hàm nhân tính
Thật vậy,
Trước hết, ta biết rằngs.p/D p
2 vớiplà số nguyên tố lẻ Ta tínhs.p k
/vớik2 ZC
(199)Tạp chí Epsilon, Số 05, 10/2015 ĐặtaDx2Cpqthì hiển nhiên
ax2Cpq.modpk/,a pq x2.modpk/
và đó, ta cópk cách chọnqđể sốa pq thặng dư phương modpk Suy
s.pk/Dpk 1s.p/ D p
k pk :
Xét số nguyên tốp D2, vớikD1; 2; 3;dễ dàng kiểm tra đượcs.2k/D1
Ta xétk 4, tương tự trên, bước chọnq, ta có cách nêns.2k/D2s.2k 1/ Từ quy nạp, ta có
s.2k/D2k 3; k4:
Tiếp theo, xét hai sốa; b nguyên dương và.a; b/ D Gọi Alà tập hợp thặng dư phương theo moduloabvàB tập hợp số thặng dư phương chung củaa; b: Nếux 2Athì tồn tạiysao choxy2.modab/ Rõ ràng đó,
xy2 mod a/; x y2 mod b/
(chú ý nếux > a, ta chọnx0sao chox0 < avàx x0 mod a/; tương tự vớib) Do đó,x 2B, tức làx2A)x 2B nênjAj jBj
Tiếp theo, xétx2 B Khi tồn tạir; s choxr2 moda/; x s2 mod b/ Theo định lý thặng dư Trung Hoa, tồn số nguyênz cho
z r mod a/; zs mod b/: Khi
x z2 mod a/; x z2 mod b/ nên
x z2:::abhayxz2.modab/: Do đó:x2 A, tức làx2 B )x2 AnênjAj jBj
Từ ta có
jAj D jBjhays.a/s.b/Ds.ab/: Vậys.n/là hàm nhân tính
Các bổ đề chứng minh Trở lại toán, ta thấy
2016D25327:
Rõ ràng toán yêu cầu đếm số thặng dư phương theo modulo2016 Theo bổ đề s.2016/ Ds.25/s.32/s.7/:
Theo bổ đề
s.25/D22 D4; s.32/D 3
2 D3; s.7/D
(200)Nhận xét.Bài tốn hồn tồn giải cách xét số dư cách tương đối "thủ công", không sử dụng kết thặng dư phương hay chí định lý thặng dư Trung Hoa
Bài 2. Choa; b; c; d số thực thỏa mãn điều kiện
a2 1; a2Cb2 5; a2Cb2Cc2 14; a2Cb2Cc2Cd2 30: Chứng minh rằngaCbCcCd 10:
2 Chứng minh rằngad Cbc 10:
Lời giải. 1) Dự đoán dấu xảy khiaD1; b D2; c D3; d D4nên ta có đánh giá sau
ˆ ˆ ˆ ˆ <
ˆ ˆ ˆ ˆ :
a2C12a b2C44b c2C96c d2C168d Do đó, ta có
24.aCbCcCd /3.d2C16/C4.c2C9/C6.b2C4/C12.a2C1/ D3d2C4c2C6b2C12a2C120
D3.a2Cb2Cc2Cd2/C.a2Cb2Cc2/C2.a2Cb2/C6a2C120 330C14C25C61C120D240
Suy raaCbCcCd 10: 2) Ta có:
16a2Cd2 8ad và9b2C4c2 12bc: Từ suy
24.ad Cbc/3.16a2Cd2/C2.9b2C4c2/
D3.a2Cb2Cc2Cd2/C5.a2Cb2Cc2/C10.a2Cb2/C30a2 330C514C105C301D240
Suy raad Cbc 10:
Bài 3. Tìm tất hàm sốf WR!Rthỏa mãn điều kiện
f x 2f y//D5f x/ 4x 2f y/ với mọix; y 2R:(*)
Lời giải. Trong./;thayx Dy D0, ta có
f 2f 0//D3f 0/:
người Hy Lạp, Vladimir Arnold, Simon Newcomb Frank Benford bài viết